X



トップページ数学
1002コメント417KB
分からない問題はここに書いてね450
レス数が1000を超えています。これ以上書き込みはできません。
0002132人目の素数さん
垢版 |
2018/12/30(日) 11:38:52.62ID:AVuA/6Xx
削除依頼を出しました
0003132人目の素数さん
垢版 |
2018/12/30(日) 20:48:48.94ID:Cu675cFc
他のスレで質問したらこちらを紹介されました。
 ↓↓↓
どなたか紳士か淑女または天才に質問です。
モジュラー方程式の解き方が分らないんですけど、教えてもらえないでしょうか。
modを使うあれです。
例えば、

7a + 5b =1 mod 12
12ab + 5b = 5 mod 7
12ab + 7a = 1 mod 5

のa,bを求めたいです。
modを12x7x5=420でまとめて計算しましたが、上手くいきません。
そのため、ユークリッドの互除法とか中国剰余定理とかいろいろ調べたんですけど、
なんとなくしかわからなくて困っています。
海外のサイトまで似たような問題を探してみたんですが、
未知数が2つになると全然ヒットしません。
それともナビエストークスみたいに現代の数学では解けない問題なんですか?
wolframeでは、一番上の一次式は解けるので、解法は存在するはずなんですが。
しかし、2番目と3番目の二次式はタイムアウトなのか、正確な回答が出ないようです。
勇志の方、どうかよろしくお願いします。
 ↑↑↑

それとNavier Stokes方程式の数値データはどこで見れますか。
速度と圧力、粘性係数、密度、外圧、時間、がデータとしてあればいいです。
OpenFoamをインストールしたのですが、使い方が分らないので、
まずはDNSの数値データで確認したいです。
0004132人目の素数さん
垢版 |
2018/12/30(日) 21:43:57.07ID:XvYA62fB
>>3
計算機でだしていいなら
Prelude> let n = 12*7*5 in [(a,b) | a<-[0..n-1],b<-[0..n-1],mod (7*a + 5*b) 12 ==1,mod (12*a*b + 5*b) 7 == 5,mod (12*a*b+7*a) 5 == 1]
[(1,102),(2,103),(3,380),(4,381),(7,288),(8,25),(9,26),(11,52),(12,293),(14,211),(16,117),
(17,58),(18,395),(19,216),(21,302),(22,123),(23,40),(24,401),(26,307),(28,225),(29,46),
(31,72),(32,73),(33,230),(36,137),(37,138),(38,415),(39,416),(42,323),(43,60),(44,61),
(46,87),(47,328),(49,246),(51,152),(52,93),(53,10),(54,251),(56,337),(57,158),(58,75),
(59,16),(61,342),(63,260),(64,81),(66,107),(67,108),(68,265),(71,172),(72,173),(73,30),
(74,31),(77,358),(78,95),(79,96),(81,122),(82,363),(84,281),(86,187),(87,128),(88,45),
(89,286),(91,372),(92,193),(93,110),(94,51),(96,377),(98,295),(99,116),(101,142),
(102,143),(103,300),(106,207),(107,208),(108,65),(109,66),(112,393),(113,130),
(114,131),(116,157),(117,398),(119,316),(121,222),(122,163),(123,80),(124,321),
(126,407),(127,228),(128,145),(129,86),(131,412),(133,330),(134,151),(136,177),
(137,178),(138,335),(141,242),(142,243),(143,100),(144,101),(147,8),(148,165),
(149,166),(151,192),(152,13),(154,351),(156,257),(157,198),(158,115),(159,356),
(161,22),(162,263),(163,180),(164,121),(166,27),(168,365),(169,186),(171,212),
続く
0005132人目の素数さん
垢版 |
2018/12/30(日) 21:44:12.59ID:XvYA62fB
続き

(172,213),(173,370),(176,277),(177,278),(178,135),(179,136),(182,43),(183,200),
(184,201),(186,227),(187,48),(189,386),(191,292),(192,233),(193,150),(194,391),
(196,57),(197,298),(198,215),(199,156),(201,62),(203,400),(204,221),(206,247),
(207,248),(208,405),(211,312),(212,313),(213,170),(214,171),(217,78),(218,235),
(219,236),(221,262),(222,83),(224,1),(226,327),(227,268),(228,185),(229,6),(231,92),
(232,333),(233,250),(234,191),(236,97),(238,15),(239,256),(241,282),(242,283),(243,20),
(246,347),(247,348),(248,205),(249,206),(252,113),(253,270),(254,271),(256,297),
(257,118),(259,36),(261,362),(262,303),(263,220),(264,41),(266,127),(267,368),
(268,285),(269,226),(271,132),(273,50),(274,291),(276,317),(277,318),(278,55),
(281,382),(282,383),(283,240),(284,241),(287,148),(288,305),(289,306),(291,332),
(292,153),(294,71),(296,397),(297,338),(298,255),(299,76),(301,162),(302,403),
(303,320),(304,261),(306,167),(308,85),(309,326),(311,352),(312,353),(313,90),
(316,417),(317,418),(318,275),(319,276),(322,183),(323,340),(324,341),(326,367),
(327,188),(329,106),(331,12),(332,373),(333,290),(334,111),(336,197),(337,18),
(338,355),(339,296),(341,202),(343,120),(344,361),(346,387),(347,388),(348,125),
(351,32),(352,33),(353,310),(354,311),(357,218),(358,375),(359,376),(361,402),
(362,223),(364,141),(366,47),(367,408),(368,325),(369,146),(371,232),(372,53),
(373,390),(374,331),(376,237),(378,155),(379,396),(381,2),(382,3),(383,160),(386,67),
(387,68),(388,345),(389,346),(392,253),(393,410),(394,411),(396,17),(397,258),
(399,176),(401,82),(402,23),(403,360),(404,181),(406,267),(407,88),(408,5),(409,366),
(411,272),(413,190),(414,11),(416,37),(417,38),(418,195)]
0006132人目の素数さん
垢版 |
2018/12/31(月) 06:03:25.69ID:7lD5urXw
>>4
早速のレスありがとうございます。

私も計算機は試しましたが、解答は一つになるはずなんです。
この>>4の結果は、恐らく420個くらいあると思うのですが、
これは12x7x5=420個と関連があると思います。
つまり、元の式を2倍すれば、解答の個数も2倍になり、
候補が膨張していきます。

そのため、一つに絞るためには最小レベルでまとめる必要があり、
今回の場合ではa=3+4mになるかもしれないし、b=2+3nになるかもしれません。
これは答えではないですが、イメージとしてはこのような形式の解答になり、
これくらい小さくまとまる可能性があるということです。

せっかく計算機でループしてもらってすいませんが、
少し期待する解答とは異なります。
ちなみに、>>4さんの解答の中には本当の答えが一つだけ存在します。
解答の形式としては、小さくまとまったa=3+4mのような形式と、
a=100、b=200というような形式の二つがあるということです。
後者の形式はmodが12のように低く設定されているので
求まらないのではないかと考えられます。
0007132人目の素数さん
垢版 |
2018/12/31(月) 07:17:20.76ID:FKhPkZl9
>>6
>解答は一つになるはずなんです。

そう考える根拠はなんなん?
どっかの問題なん?

短くするなら 5(b-a) ≡ 1 (mod 12) b-a ≡ 5 (mod12)となり
b = a+5+12k
とおける。
a^2 + 12(k+1)a -1 ≡ 0 (mod 5)
により
a ≡ -6(k+1) ± √(36k^2+72k+35) (mod 5)
であるから (k,a) ≡ (1,3),(2,2),(4,1),(4,4) (mod 5)。
それぞれのケースを12ab+5b≡5 (mod 7)に代入すれば短い表現はできるけど数学的には意味ある作業じゃない。
しょうもないだけ。
0008132人目の素数さん
垢版 |
2018/12/31(月) 07:40:08.73ID:5Zpn8kIg
>>6
>解答は一つになるはずなんです。
もしそういう前提なら問題を写し間違えてるのかもね

7にもあるように>>3の問題を簡単化すると
b-a≡5 (mod 12) → 解は12を法として12通り (a,b)≡(0,5),(1,6),(2,7),(3,8),(4,9),(5,10),(6,11),(7,0),(8,1),(9,2),(10,3),(11,4) (mod 12)
b(a+1)≡1 (mod 7) → 解は7を法として6通り (a,b)≡(0,1),(1,4),(2,5),(3,2),(4,3),(5,6) (mod 7)
a(b+1)≡3 (mod 5) → 解は5を法として4通り (a,b)≡(1,2),(2,3),(3,0),(4,1) (mod 5)

よって大雑把に言うと、解は12*7*5=420を法として12*6*4=288通り
4〜5 の回答とも一致するはず
0009132人目の素数さん
垢版 |
2018/12/31(月) 10:53:25.54ID:yaS+NxvF
うーん。やっぱりわかる人はいないか。

>>7
a ≡ -6(k+1) ± √(36k^2+72k+35) (mod 5)
であるから (k,a) ≡ (1,3),(2,2),(4,1),(4,4) (mod 5)。

kを平方根部分に代入しても整数にはならないみたいです。
√(36x1x1+72x1+35)=11.9582

イメージとしては12通り、6通り、4通りの3つのグループで共通する一つの解を見つける感じです。
だから、それぞれのグループで異なる形式をしていても、本質的には共通する一つの解を見出す必要があります。
modの数値を大きくして、候補を多くするのではなく、
反対にmodの数値を小さくして候補を一つに絞ります。
答えがmodを含む形でもいいのは、それが科学的に重要な意味があるからです。
数学的意味があるかどうかは関係ありません。
とはいえ、これは数学の問題なので、こちらに投稿させていただきました。
しかし、期待する回答は得られませんでした。残念。
0010132人目の素数さん
垢版 |
2018/12/31(月) 10:55:17.35ID:yaS+NxvF
書き忘れましたが、この問題について考えて頂いた勇志の方、
しょーもない問題に付き合わせてしまってすみませんでした。
もう一度自分の頭で考えてみます。
ありがとうございました。ではでは。
0011学術
垢版 |
2018/12/31(月) 12:36:43.96ID:O82e6c+A
数学を振り返ってみると、数学時代は一番いい時代だった。
0012132人目の素数さん
垢版 |
2018/12/31(月) 12:58:40.00ID:WhJD0x31
>>9
そら数学が論理的帰結を無視して期待通りにできたら、宇宙の法則だって乱れるさ
0013132人目の素数さん
垢版 |
2018/12/31(月) 14:06:29.11ID:d13PIu5N
いや、おそらく>>9が言いたいのは、mod420 で表される288通りの解を1通りの式で表しなさい、ってことかと。

例えば
st≡1 (mod 5)
uv≡1 (mod 7)
となるような媒介変数 s,t,u,v を使って

a≡f(s,u) (mod 420)
b≡g(t,v) (mod 420)

の形で表すことができたら>>9は満足なのかも。
0015132人目の素数さん
垢版 |
2018/12/31(月) 17:05:09.25ID:8viLx2WK
>>13
なにがいいたいのかわからないではないが

>>9
>kを平方根部分に代入しても整数にはならないみたいです。
>√(36x1x1+72x1+35)=11.9582

とか言ってるようではねぇ?
mod 5 での平方根の処理まで解説してられない。
0017132人目の素数さん
垢版 |
2018/12/31(月) 17:56:26.39ID:+vCode2O
K, E1, E2 を体とする。
K は E1, E2 の部分体とする。

E1 ∩ E2 は体になることを示せ。
0019132人目の素数さん
垢版 |
2018/12/31(月) 19:23:34.05ID:lZNtWIEW
■速報■

無限に続くと思われていた円周率がついに終りを迎えた
千葉電波大学の研究グループがこれまでの円周率演算プログラムに
誤りがあったことを発見
同大のスーパーコンピュータ「ディープ・ホワイト」を使って
改めて計算しなおしたところ、17327029桁目で割り切れたという

17327029桁目の最後の数字は「7」だった

千葉電波大学の研究グループの発表によると、
円周率計算に際し、改めて既存の円周率計算プログラムを
点検してみたところ、円周の誤差を修正する数値に
誤りがあることに気が付いた
この数値を正常値に直して計算しなおしてみたところ、
円周率は17327029桁で割り切れたという
0020132人目の素数さん
垢版 |
2018/12/31(月) 19:44:36.28ID:+vCode2O
部分環について質問です。


R を単位元をもつ環とする。

R' が単位元をもつ環であるとき、 R の部分環という。


とはなぜ定義しないのでしょうか?

つまり、 R' が単位元をもつ環であって、かつ、 R の単位元を含まない場合に、
R' を部分環からなぜ排除するのでしょうか?
0021132人目の素数さん
垢版 |
2018/12/31(月) 19:45:49.38ID:+vCode2O
訂正します:

部分環について質問です。


R を単位元をもつ環とする。

R の部分集合 R' が単位元をもつ環であるとき、 R の部分環という。


とはなぜ定義しないのでしょうか?

つまり、 R の部分集合 R' が単位元をもつ環であって、かつ、 R の単位元を含まない場合に、
R' を部分環からなぜ排除するのでしょうか?
0022132人目の素数さん
垢版 |
2018/12/31(月) 20:14:42.14ID:+vCode2O
今、上野さんのことだから「もしや?」と思い、上野健爾著『代数入門』を調べてみました。

「可換環 R が与えられたとき、 R の部分集合 S が R の和と積に関して閉じていて、この和と
積に関して可換環になるとき、 S を R の部分環(subring)であるという。」

などと書いてありました。

上野さんの本での「可換環」は乗法に関する単位元をもちます。

上野健爾さんは大丈夫な人なんでしょうか?
0023132人目の素数さん
垢版 |
2018/12/31(月) 21:12:31.33ID:AWqsdek+
これ解ける方おりますか?

nを自然数とする。A,Bはn次実正方対称行列でAの全ての固有値は正で、Bの全ての固有値は非負であると仮定する。

u=u(t)をR^n値の未知関数とする方程式
d^2u/dt^2+B*du/dt+Au=0
を考える。Aの任意の固有値λに対してKer(B)∩Ker(A-λI)={0}を仮定する。このとき上の方程式の任意の解u=u(t)はt→∞でu(t)=0となることを示せ。
0024132人目の素数さん
垢版 |
2018/12/31(月) 23:56:19.67ID:bZtX71gd
こんな質問するのもあれなんだが、気になったから質問する。

司法試験に合格するのと東京大学大学院数理科学研究科数理科学専攻博士課程修了するのはどっちの方が難しい?

ちなみに両方とも学部は東大。
0025132人目の素数さん
垢版 |
2019/01/01(火) 00:37:34.61ID:ZjsXOY9q
でもヒマラヤさんは東大生じゃないから関係ないんじゃないですか?
0027132人目の素数さん
垢版 |
2019/01/01(火) 06:22:07.65ID:i9MWj87p
ナンバーズ4は当たる確率が10000の一。

0000から9999の番号を買ったとしても当たる確率は10000の一。
1枚200円だから、ぜ〜んぶ買うと2,000,000円。
賞金の最高金額が100万円。当たっても、100万円の損。
何回も買うと確率論に組み込まれてしまう。

何とかの法則? 定理?で、最初の数枚買うなら当たる確率がある。
(ビギナーズラック)。たった一回で当たる確率は一億分の一。
日本人1億人が、ナンバーズ4を買うとして、
たった一人がたった一回で100万円を手にする確率。
だが、毎回一億人もナンバーズ4を買っていない。とてもあほらしい

以上が、自分が投稿しようと思っていた文章ですが、計算、あってますか?
0028132人目の素数さん
垢版 |
2019/01/01(火) 10:12:36.08ID:/kCNEN/H
寺田文行訳『ガロア理論入門』を読んでいます。

K, E1, E2 を体とする。
K は E1, E2 の部分体とする。

ある問題の中で、寺田さんは、 E1 ∩ E2 が体になるということを使っています。

E1, E2 を部分体として含む体 E が存在すれば、
E1 ∩ E2 が体になることは簡単に分かります。

そのような体 E の存在を仮定しない場合、

a, b ∈ E1 ∩ E2 としたとき、

a *1 b ∈ E1
a *2 b ∈ E2

ですが、

a *1 b = a *2 b

でないとまずいですよね?

このあたりはどう考えればいいのでしょうか?
0029132人目の素数さん
垢版 |
2019/01/01(火) 10:55:46.24ID:bylOGYZS
三角形の形状決定問題は辺の関係式に直してまとめていくと思うんですが、写真のような二倍して因数分解の方式に落とし込んで、=より、正三角形と出せるのはどういった考えを念頭に置いて計算してるのでしょうか?
こういった類いの計算は暗記しておくべきですか?

https://dotup.org/uploda/dotup.org1734347.jpg
https://dotup.org/uploda/dotup.org1734349.jpg
https://dotup.org/uploda/dotup.org1734351.jpg
https://dotup.org/uploda/dotup.org1734352.jpg
0030132人目の素数さん
垢版 |
2019/01/01(火) 12:15:58.08ID:pKc5cC4D
>>29
この変形は、よく使う変形なので覚えておくと便利。
a^2 + b^2 + c^2 - ab - bc - ca = 1/2((a-b)^2 + (b-c)^2 + (c-a)^2)
a=b=cの時に両辺とも0になるから、因数定理じゃないけど直観的におもいつきやすいかも。

a^3+b^3+c^3-3abc = (a+b+c)(a^2+b^2+c^2-ab-bc-ca)
の右辺にも出てくる
0031132人目の素数さん
垢版 |
2019/01/01(火) 12:24:21.27ID:/kCNEN/H
a^2 + b^2 + c^2 - ab - bc - ca = 1/2((a-b)^2 + (b-c)^2 + (c-a)^2)


2 * [a^2 + b^2 + c^2 - ab - bc - ca]

[a^2 + b^2 + c^2 - ab - bc - ca] + [a^2 + b^2 + c^2 - ab - bc - ca]

=

[a^2 - 2*a*b + b^2] +[b^2 - 2*b*c + c^2] + [ c^2 - 2*c*a +a^2]

=

(a - b)^2 + (b - c)^2 + (c - a)^2

∴ a^2 + b^2 + c^2 - ab - bc - ca = (1/2) * [(a - b)^2 + (b - c)^2 + (c - a)^2]
0034132人目の素数さん
垢版 |
2019/01/01(火) 21:35:45.40ID:bylOGYZS
>>33
素数が無限の証明と同じちゃうの?
theorem2 が何か分からんから知らんけど。
Π[f:irr] 1/(1-q^(-deg f)) div.s⇔ Σ [f:irr] q^(-deg f) div.s
を示しておく。
左辺は Σ [f] q^(-deg f) とおなじだから←は明らか。
Σ [f:irr] q^(-deg f が収束するとすれば
-Σ [f:irr] log(1-q^(-deg f))はそれより小さいから収束し、よって Π[f:irr] 1/(1-q^(-deg f)) も収束する。
0035132人目の素数さん
垢版 |
2019/01/01(火) 21:39:04.29ID:bylOGYZS
あ、下の方のリンクがTheorem 2 へのリンクなのか。
反転公式使う証明ね。
0036132人目の素数さん
垢版 |
2019/01/01(火) 22:08:44.27ID:zweBPVgg
                   ◎
   >┴<             /_________
 -( ゚∀゚.)-旦         /| ◎ 《,《,《,《,《,《,《,《,《/Σ
   >┬<            /\|  ノミ《,《,《,《,《,《,《,《,《\Σ
   .        ∧,,∧  /  . ̄ ̄ ̄ ̄ ̄ ̄ ̄ ̄ ̄
          (^ω^ =)/
           O┬O )シャカシャカシャカ
          ◎┴(())'◎===== 
;;⌒::.;;.⌒⌒/   /| ̄ ̄ ̄ ̄ ̄/   /::. :; ;⌒⌒:.:⌒:;⌒;;⌒   
..  ,::.;  /   /| ̄ ̄ ̄ ̄ ̄/   /.., ,; .:   ,,。,.(◯)   ::
  : :::., /   /| ̄ ̄ ̄ ̄ ̄/   /,,;  (◯)  ::: ヽ|〃  ;;:
.  ,:.; /   /| ̄ ̄ ̄ ̄ ̄/   /.., ,; :ヽ|〃  ,,。,    ::;;,

ラファエルに直接会ってディスる歌「ラファエルクッキング最高」
http://www.youtube.com/embed/fw-iNonBsi0?playlist=Gjl7zquvb9s,PG41nKiQMFA,6qcwzd8Rgko,BWCMsp9hNdM,uzhvVvhUa3U,a872cPnnjhk,34Wg0HWPVlo,xZrZW2JS4hE&;autoplay=1&loop=1

パシャ パシャ  パシャ パシャ パシャ  パシャ パシャ パシャ パシャ パシャ
   パシャ パシャ パシャ パシャ パシャ  パシャ パシャ パシャ  パシャ  パシャ
 ∧_∧      ∧_∧     ∧_∧  ∧_∧    ∧_∧     ∧_∧
 (   )】      (   )】    (   )】 【(   )    【(   )    【(   )
 /  /┘ .   /  /┘.    /  /┘ └\ \   └\ \   └\ \
ノ ̄ゝ     ノ ̄ゝ      ノ ̄ゝ     ノ ̄ゝ    ノ ̄ゝ旦    ノ ̄ゝ
パシャ パシャ  パシャ パシャ パシャ  パシャ パシャ パシャ パシャ パシャ
   パシャ パシャ パシャ パシャ パシャ  パシャ パシャ パシャ  パシャ  パシャ
 ∧_∧      ∧_∧     ∧_∧  ∧_∧    ∧_∧     ∧_∧
 (   )】      (   )】    (   )】 【(   )    【(   )    【(   )
 /  /┘ .   /  /┘.    /  /┘ └\ \   └\ \   └\ \
ノ ̄ゝ     ノ ̄ゝ      ノ ̄ゝ     ノ ̄ゝ    ノ ̄ゝ旦    ノ ̄ゝ
0038132人目の素数さん
垢版 |
2019/01/01(火) 22:11:12.47ID:zweBPVgg
⊞ ⊟ ⊠ 〼 ⊡ ◆
⊕ ⊖ ⊗ ⊘ ⦿ ●
0040132人目の素数さん
垢版 |
2019/01/01(火) 22:57:26.85ID:hB2NEMBr
>>34
うーん、それと同じですかね
定理2(メビウスの反転公式)のやつを使えとあったのでそこが引っかかっています
0041132人目の素数さん
垢版 |
2019/01/01(火) 23:05:55.26ID:hB2NEMBr
>>34
定理2というのが誤植な気がしてきました。
解説ありがとうございます。
素数が無限にある証明がどんなやつなのかわかってないので調べてきます。ありがとうございます。
0042132人目の素数さん
垢版 |
2019/01/01(火) 23:40:22.96ID:XKtNhiWs
位相空間の問題が分からない。解ける人がいたら頼む。

(X,d)をコンパクトな距離空間とする。連続写像f:X→Xがあるとする。いま互いに交わらない空でない閉集合K_1、K_2があって

f(K_1)⊃K_2、f(K_2)⊃K_1∪K_2

が成立しているとする。

(1)集合列M_1、M_2、M_3、・・・、M_m、・・・が2条件
(i)各自然数mに対してM_m=K_1またはM_m=K_2
(ii)もしM_k=K_1ならばM_(k+1)=K_2
を満たすとする。この時、あるx∈K_1∪K_2があってf^n(x)∈M_n(nは自然数)とできることを示せ。

(2)(1)の集合列{M_m}_m=1^∞の取り方の全体の濃度は♯(R)となることを示せ。

f^2(x)=f(f(x))、f^3(x)=f(f(f(x)))のように、f^n(x)は自分自身との合成写像を示しています。
https://ja.wikipedia.org/wiki/%E5%86%99%E5%83%8F%E3%81%AE%E5%90%88%E6%88%90
0043132人目の素数さん
垢版 |
2019/01/02(水) 00:26:54.92ID:ozp04/Hn
>>42
まず列が有限の場合には帰納法。
列の長さ1で明らか。
長さ<Nで成立として長さがNのとき
y∈K_1∪K_2があってf^n(y)∈M_(n+1) 2≦n≦N をみたすものがとれる。
(i) M_1 = K_1 のとき
仮定により f(y)∈M_2 = K_2 であるから z∈K_1 を f(z)=y と選べる。
さらに x∈K_1∪K_2 を f(x)=z を満たすようにとれてこれが条件を満たす。
(ii) M_1 = K_2 のとき
仮定により f(y)∈M_2 ⊂ K_1∪K_2 であるから z∈K_2 を f(z)=y と選べる。
さらに x∈K_1∪K_2 を f(x)=z を満たすようにとれてこれが条件を満たす。
以上により長さ有限の場合の証明が終わった。
次にx[N]を
f^n(x[N])∈M_n (1≦n≦N)
を満たすようにとれる。
K_1∪K_2 はコンパクト距離空間だから、必要なら部分列をとってlim x[N]=xが存在するとして良い。
この x が条件を満たす。
0044132人目の素数さん
垢版 |
2019/01/02(水) 01:29:14.53ID:kmE/oyRb
ルービック・立方体(キューブ)の組み合わせ数は
1801439850948198で、あってる? 正しい?
0045132人目の素数さん
垢版 |
2019/01/02(水) 01:29:39.24ID:nsTEkruF
すいません
方べき定理で出てきた答えって結局なんなんですか?
0046132人目の素数さん
垢版 |
2019/01/02(水) 01:34:48.64ID:1XTwIlaG
答えです
0047132人目の素数さん
垢版 |
2019/01/02(水) 01:53:56.86ID:nsTEkruF
そういうことじゃ無い
結局その答えはなんの答えなんですか?
問題の答えとか言う回答は待ってません
0048132人目の素数さん
垢版 |
2019/01/02(水) 03:02:24.95ID:7wxHl3VN
そういうことじゃないならどういうことなのかはっきり説明してくれ

定理には答えなんて存在しない
そう答えればいいのか?
0049132人目の素数さん
垢版 |
2019/01/02(水) 09:26:20.21ID:UKsYtLJs
f_1(x)=x^2 とし、n=1,2,3,...に対して
f_(n+1)(x)=|f_n(x)-1|
と定める。
(1)y=f_2(x), y=f_3(x)の概形を書き,
(2)0≦x≦√(n-1) において 0≦f_n(x)≦1 を,
√(n-1)≦x において f_n(x)=x^2 -(n-1) を示せ。
また,
(3) n≧2 として, y=f_n(x) のグラフとx軸で囲まれた図形の面積をS_nとした時に S_n + S_(n+1) を求めよ。

これを教えてください。
0050132人目の素数さん
垢版 |
2019/01/02(水) 11:15:50.98ID:3rBsZ6cK
∫(1/1+x^2)dx = arctan xですよね?

tany = xとおくとdx/dy=1/cos^2=1+x^2
よってdy/dx=1/1+x^2
この式変形の手順自体は理解できるのですが

でもtanxは周期2πの周期関数だからy=arctan x はxに対して無限個のyを返す多値関数ですよね?
そうすると例えば定積分は定まらなくないですか?

これはどう解釈したらいいのでしょうか?
2πの周期は積分定数のようなものだから〜という説明も考えたのですがやはり納得いきません。

どなたかスッキリ解決できる方はいらっしゃらないでしょうか?
0051132人目の素数さん
垢版 |
2019/01/02(水) 11:31:35.20ID:GvDO1SFm
>>29

137
次の等式が成り立つとき、△ABCはどのような形の三角形か
 aa+bb+cc = bc(1/2 + cos(A)) + ca(1/2 + cos(B)) + ab(1/2 + cos(C)),

(略解)
第二余弦定理
 cos(A) = (bb+cc-aa)/(2bc),
 cos(B) = (cc+aa-bb)/(2ca),
 cos(C) = (aa+bb-cc)/(2ab),
を与えられた式の右辺に代入すると

bc(1/2 + cos(A)) + ca(1/2 + cos(B)) + ab(1/2 + cos(C))
 = ・・・・
 = ・・・・
 = (1/2)(bc+bb+cc-aa) + (1/2)(ca+cc+aa-bb) + (1/2)(ab+aa+bb-cc)
 = (1/2)(aa+bb+cc + ab+bc+ca),
よって
 aa+bb+cc = (1/2)(aa+bb+cc + ab+bc+ca),
整理すると aa+bb+cc -ab -bc -ca = 0,
しかし
 aa+bb+cc -ab -bc -ca = (1/2){(a-b)^2 + (b-c)^2 + (c-a)^2}
であるから
 (1/2){(a-b)^2 + (b-c)^2 + (c-a)^2} = 0,
よって a-b = b-c = c-a = 0,
すなわち a = b = c,
ゆえに,△ABC は正三角形。 ・・・・ 答
0052132人目の素数さん
垢版 |
2019/01/02(水) 11:32:40.25ID:cKCL6IJm
この飛行機までのおよその距離を知りたい
https://i.imgur.com/tHtQSUb.jpg
トリミング無し
レンズ35mm版換算3000mmの水平画角0.687541°
35mm版の横幅を36mmとする
飛行機はB737-800 全長39.5mとする
進行方向への角度やセンサーサイズの誤差は無視でお願いします

計算のしかたと答えを教えて下さい(飛行機の全長から距離を測る計算)
よろしくお願いします
0054132人目の素数さん
垢版 |
2019/01/02(水) 11:55:44.76ID:GvDO1SFm
>>33
qが素数の場合は上の画像のとおり。
qが合成数の場合は素因数に分解してTheorem2(メビウス反転公式)を使え、ということか?
0055132人目の素数さん
垢版 |
2019/01/02(水) 12:06:24.63ID:3rBsZ6cK
>>53
置換したらそうなるのは理解できるのですが、
最初の逆関数の微分の変形だけでtan-1 xなのは導出できますよね?

そこからなぜ連続した部分に対応するものを選ばなければならないというのが分かるのでしょうか?
0056132人目の素数さん
垢版 |
2019/01/02(水) 12:16:09.63ID:1XTwIlaG
>>55
>∫(1/1+x^2)dx

定積分できないとか言ってますけど、これは定積分ではないですね
これは何を意味しているつもりなんですか?
0057132人目の素数さん
垢版 |
2019/01/02(水) 12:46:07.76ID:3rBsZ6cK
>>56
原始関数の導出のつもりでした。
aからbまでの定積分って原始関数(b)-原始関数(a)ですよね?
原始関数が多値関数ってどういうことなんだろうと思ったのです
0058132人目の素数さん
垢版 |
2019/01/02(水) 13:10:01.29ID:1XTwIlaG
>>57
原始関数とはなんですか?
0059132人目の素数さん
垢版 |
2019/01/02(水) 13:22:47.17ID:5hoS1qxn
>>52
飛行機の長さが画像の水平方向の長さに対してどれだけに
なるか比率を測ればいいだけだろ。中学生でもわかる問題。
tan(水平画角×比率) = 飛行機長÷距離
なんだから、
距離=飛行機長/tan(水平画角×比率)
この写真だと、比率は260px/1200px =0.217なので
距離=39.5m/tan(0.687541° x 0.217)=15200m
0061132人目の素数さん
垢版 |
2019/01/02(水) 14:37:44.74ID:dpB9xwpW
>>50
値域を選択すれば(例えば(-π/2,π/2)など)arctan(x)は多価でない関数になります
(一般にarctan(x)を使うときは多価関数と見なすよりもこのようにして普通の関数として扱うことが多いと思います)
値域の選択によって関数は変わりますが、定数しかズレないので、原始関数としては同じものを表しています
多価関数といってもmonodromyがあるわけではないので、それほど複雑な話ではありません
0062132人目の素数さん
垢版 |
2019/01/02(水) 14:42:07.13ID:ppDCuzQe
Qの条件の曲線と直線x+y=aで囲まれる図形を直線を軸に回転させた時の回転体の体積を求めよという問題なのですが、
この解答はどこが間違っていますでしょうか?

https://i.imgur.com/UJkw39m.jpg
0063132人目の素数さん
垢版 |
2019/01/02(水) 14:45:08.93ID:3rBsZ6cK
>>61
回答ありがとうございます。

最初の原始関数の導出は値域とかについてなんの仮定も置いてないですよね?

なのになぜarctanxのとりうる値はどこか適当な幅πの間であることにすると限定してしまうことができるのでしょうか?
0064132人目の素数さん
垢版 |
2019/01/02(水) 15:09:30.89ID:3j9fwFeq
>>62
最後から5行目の積分の中身
(a - a sin^4θ)^2
の左の定数aって何?
0065132人目の素数さん
垢版 |
2019/01/02(水) 15:13:30.58ID:Tjk3JLSV
>>63
(少なくとも各点の周りの適当な連結開集合上で)引数を連続的に変化させれば値も連続的に変化するから
0066132人目の素数さん
垢版 |
2019/01/02(水) 15:18:16.94ID:ne8HBguh
>>63
原始関数とはなんですかと聞いてますね
なぜ答えないのですか?
0067132人目の素数さん
垢版 |
2019/01/02(水) 15:43:31.88ID:7wxHl3VN
>>63
最後の最後で計算ミスしてるやん…
目が悪いんだからこんな細かい計算チェックやらさんといて…
0069132人目の素数さん
垢版 |
2019/01/02(水) 15:57:09.00ID:dpB9xwpW
>>63
「tany = xとおくとdx/dy=1/cos^2=1+x^2
よってdy/dx=1/1+x^2」

2行目でdy/dxとありますが、この時点で暗にx=tan(y)の逆関数の存在を仮定しています
しかし、逆関数が存在する為には単射でないといけません
x=tan(y)は単射ではないので、定義域を(-π/2,π/2)等に制限して単射にしているわけです
0070132人目の素数さん
垢版 |
2019/01/02(水) 16:06:12.66ID:3rBsZ6cK
>>69
なるほど!ありがとうございます。
頭が悪いので怪しいですが理解できたような気がします
>>67
すみません、よくわかりません
0072132人目の素数さん
垢版 |
2019/01/02(水) 16:15:42.37ID:ppDCuzQe
>>67
本当だ、とりあえず最後の積分計算で最後に足している1は1/2の誤りですね…ありがとうございます

ただここを修正してもまだ模範解答と答えが合わないのですが、他どこがおかしいか分かる方いらっしゃらないでしょうか
wolframでは複雑すぎて処理できませんでした……早大の過去問です。
0073132人目の素数さん
垢版 |
2019/01/02(水) 16:18:59.49ID:ppDCuzQe
>>64
上のQの仮定にある正の実数の定数です
x=a*cos^4θ、y=a*sin^4θ、で書く曲線をx+y=aで切り取った部分を軸に回転させています
0074132人目の素数さん
垢版 |
2019/01/02(水) 16:33:12.66ID:ppDCuzQe
すいません、誤りに気づきました………
スレ汚しすみません死んできます
0075132人目の素数さん
垢版 |
2019/01/02(水) 19:22:56.44ID:ohr+DqS6
座標平面上の原点をOとし、y軸の正の方向をOから見て真北の方向と定める。
時刻0において点Pは原点にある。
nを自然数とし、各時刻nにおいて4つの数1,2,3,4から無作為に一つを選ぶ。
1が選ばれたときは点Pを東に、2が選ばれたときは西に、3が選ばれたときは南に、4が選ばれたときは北に、それぞれ(1/2^n)だけ移動する。
以下の問に答えよ。

(1)点Pのx座標をa、y座標をbとする。n→∞としたときのa,bの期待値を求めよ。

(2)時刻1において選ばれた数が1であったときの、n→∞としたときのa,bの期待値を求めよ。
0077132人目の素数さん
垢版 |
2019/01/02(水) 23:36:32.22ID:ohr+DqS6
aは0でない複素数、b,cは複素数である。
xについての2次方程式
ax^2+bx+c=0
が以下のような解を持つとき、a,b,cが満たすべき条件を述べよ。

(1)2解(以下、重解も2解とする)がともに実数である。

(2)2解がともに実数でない。

(3)2解の一方が実数で、他方が実数でない。
0080132人目の素数さん
垢版 |
2019/01/03(木) 10:18:05.89ID:clk1Joik
多項式について質問です。

(a_0 + a_1*x + … + a_n*x^n) + (b_0 + b_1*x + … + b_m*x^m)

上の式で多項式の項の間にある「+」と多項式と多項式の和の「+」を同じ記号で
表わしていますが、悪い習慣でしょうか?
0082132人目の素数さん
垢版 |
2019/01/03(木) 10:35:40.84ID:1clk5DIm
微分方程式の質問です。

以下の問題が分かりません。

Aをd次の正方行列で、g=g(t,ξ)を写像g:R×R^d → R^dでξに関して全微分可能でδg(t,ξ)/δξも連続であるとする。 いま、uをR^d値の未知関数とする方程式
du/dt=Au+g (t,u) (※)
を考える。あるK > 0があって|g(t,ξ)| ≦ K (t ∈ R,ξ ∈ R)が成立するとする。

(1)この時任意のa ∈ Rに対して(※)の−∞ < t < ∞における解でu(0) = aとなるものが存在することを示せ。

(2)さらにAを実対称行列で全ての固有値は負であるとする。このとき、aを適当に選ぶことでu(t)は−∞ < t < ∞で有界になることを示せ。

(1)は解けたけど、(2)がギブアップ。(2)を解ける人、解説お願いします。
0084132人目の素数さん
垢版 |
2019/01/03(木) 13:50:02.11ID:clk1Joik
>>81

ありがとうございます。

多項式とは何かということに関して、多くの代数学の本では、説明が全くないですよね。

あまり評判は芳しくないようですが、Serge Lang著『Undergraduate Algebra』という本に
ちゃんとした説明がありました。松坂和夫著『代数系入門』にも書いてありますね。
0085132人目の素数さん
垢版 |
2019/01/03(木) 15:19:51.98ID:tKhMtALm
論理学に詳しい方に質問です。

背理法のネストは数学的に正しいのでしょうか?
例えば、「A(偽な命題)を仮定する。……ここで、B(偽)を仮定すると、矛盾が生じる。よって、Bは誤り。つまりnotBが正しい。……すると、矛盾が生じる。よってAは誤り。」という議論は正しいのでしょうか?
Bを誤りだと断定した矛盾が、本当にBによるものなのかそれともAによるものなのかが分からないし、場合によってはAかBの単体を仮定するだけではその矛盾は導かれないが、A,Bを共に仮定した時にのみその矛盾が導かれる、っていうことがありそうな感じがして…
0086132人目の素数さん
垢版 |
2019/01/03(木) 15:22:08.65ID:9wtPVJ3r
矛盾というのは、前提が正しいなら何をしても起こらないはずなんです
それが起こったということは前提が間違えだったということで、これが背理法ですね
いいんですよそういうことしても別に
0087132人目の素数さん
垢版 |
2019/01/03(木) 15:28:30.29ID:tKhMtALm
>>86
すみません、質問の仕方が悪かったんですが、これでnotAが正しいことを示せたのかってことです
これだとnot(A∧B)しか示せてない気がして

すみません、ここまで書いて自己解決しました…お目汚し失礼しました…
0088132人目の素数さん
垢版 |
2019/01/03(木) 16:26:29.04ID:rGbJGMG0
背理法は、A→B と A→¬B がともに成り立つことを示して、そこから ¬A を導く方法なのかと
0089132人目の素数さん
垢版 |
2019/01/03(木) 18:46:32.32ID:FhdwjoyF
お願いします

aは0でない複素数、b,cは複素数である。
xについての2次方程式
ax^2+bx+c=0
が以下のような解を持つとき、a,b,cが満たすべき条件を述べよ。

(1)2解(以下、重解も2解とする)がともに実数である。

(2)2解がともに実数でない。

(3)2解の一方が実数で、他方が実数でない。
0090132人目の素数さん
垢版 |
2019/01/03(木) 20:33:09.11ID:clk1Joik
松坂和夫著『代数系入門』を読んでいます。


整数 a の標準分解を

a = p_1^α_1 * p_2^α_2 * … * p_k^α_k

とする。



d を a の約数とし、 a = d * q とすれば、 d および q の素因数はもちろん a の素因数であるから、

d = p_1^β_1 * p_2^β_2 * … * p_k^β_k

q = p_1^γ_1 * p_2^γ_2 * … * p_k^γ_k

と書くことができる。


と書かれています。

d, q がそのように書けるということを導くのに素因数分解の一意性が使われているのに、そのことが
書いてありませんね。松坂さんは素因数分解の一意性がここでも使われていることに気づいていま
せんね。

↑の下の行で再び素因数分解の一意性が使われているのですが、そこでは、一意性が使われている
ことが書かれています。
0091132人目の素数さん
垢版 |
2019/01/03(木) 20:40:41.42ID:AYL6n86W
先頭車両から順に1からnまでの番号がついたn両編成の列車がある。ただしnは2以上とする。各車両を赤色、青色、黄色のいずれか1色で塗るとき、隣り合った車両の少なくとも一方が赤色となるような色の塗り方は何通りか。
この問題は漸化式を使わずに解く方法はありますか?
0098132人目の素数さん
垢版 |
2019/01/04(金) 01:46:52.79ID:4y9oavEG
>>89
(1) b/a,c/a が実数で、(b/a)^2 - 4(c/a) ≧ 0.

>>91
(n+1)両目に赤を追加すると → 「赤」「青赤」「黄赤」を並べたものになる。
0102132人目の素数さん
垢版 |
2019/01/04(金) 13:45:39.54ID:4y9oavEG
>>98
つまり
x/(1-x-2xx) = x/{(1-2x)(1+x)} = (1/3){1/(1-2x) - 1/(1+x)} = (1/3)納n=1,∞] {2^n - (-1)^n} x^n,

答 {2^n - (-1)^n}/3.
0103132人目の素数さん
垢版 |
2019/01/04(金) 13:57:39.89ID:4y9oavEG
>>98
nが2つずれた...orz
{1/(1-x-2xx) - 1}/x = (1+2x)/{(1-2x)(1+x)} = (1/3){4/(1-2x) - 1/(1+x)} = (1/3)Σ[n=1,∞] {2^(n+2) - (-1)^n} x^n,

答 {2^(n+2) - (-1)^(n+2)}/3.
0104132人目の素数さん
垢版 |
2019/01/04(金) 17:29:53.61ID:xOMYmhWY
以下の性質を満たす集合 E は存在するか?

(1)
E ⊂ R × {0} ⊂ R × R
E は R × R で閉集合、 R × {0} で閉集合ではない。

(2)
E ⊂ R × {0} ⊂ R × R
E は R × R で閉集合ではない、 R × {0} で閉集合。
0105132人目の素数さん
垢版 |
2019/01/04(金) 18:58:57.05ID:s68Y7dWN
リチャード・テイラーっていうイギリスの数学者はどのくらいのレベルの数学者ですか?
現役ではそこそこ上位の方に入るぐらいの学者ですか?
0106132人目の素数さん
垢版 |
2019/01/04(金) 23:19:10.56ID:UTaC5hnL
>>105

底辺のものが語るべき話題にあらず
0107132人目の素数さん
垢版 |
2019/01/04(金) 23:23:11.55ID:N1T5KklW
3 5 11 21 43 85 171 341 683 1365 2731 5461 10923 21845 43691

を一つの式で表すとどうなりますか?

末尾で1 1 3 5を繰り返すようです
0109132人目の素数さん
垢版 |
2019/01/04(金) 23:37:47.72ID:H2FFTYS5
>>107
(2^(n+2)-(-1)^n)/3
0110132人目の素数さん
垢版 |
2019/01/05(土) 02:07:58.36ID:vJrRtyuT
{2^(n+2) - (-1)^(n+4)}/3

{2^(n+2) - (-1)^(n+664)}/3でも結果は同じ

実部と虚部の関係がわからない
0111132人目の素数さん
垢版 |
2019/01/05(土) 12:53:50.54ID:k/cacouY
3^30 ≡ 1 + 17*31 (mod 31^2)

を証明せよ。

松坂和夫著『代数系入門』の問題です。

松坂さんの意図として、(直接)計算の練習として出題したのか、何かうまい解法があるのかが判断できません。

うまい解法を思いついた人は解答してください。
0114132人目の素数さん
垢版 |
2019/01/05(土) 14:07:06.33ID:k/cacouY
>>113

Fermatの小定理はこの問題が出題されたセクションよりも後で登場します。

>>111

の問題は合同式の定義が説明されているセクションの問題です。

「松坂さんの意図として、(直接)計算の練習として出題したのか、何かうまい解法があるのかが判断できません。」

と書きましたが、おそらく単なる計算問題ではないと推測します。

なぜそう推測したかというと、

(1)そのセクションには11問の問題があるのですが、その最後の問題であること。
(2)「証明せよ」と書いていること。(単なる計算問題だったら「示せ」が自然。)
0115132人目の素数さん
垢版 |
2019/01/05(土) 15:55:52.86ID:TXJNULY7
神ですら無には敵いませんか?
0116132人目の素数さん
垢版 |
2019/01/05(土) 16:08:07.36ID:k/cacouY
>>111

松坂和夫さんのことなので、おそらくアメリカの初等数論の教科書かなんかに載っている問題を
コピー&ペーストしたのではないかと思います。
0117132人目の素数さん
垢版 |
2019/01/05(土) 19:34:30.13ID:wZ3XBthC
冪nの反復合成写像を f^n(x) で表す。
f^({2^(f(x))}+1)(x) ≦ f^(2^x)(x)
f^(2^x) = g(x)
(xは自然数)
のどちらも満たすf(x)を全て求めよ。g(x)は定数でないxの多項式関数とする。
0118132人目の素数さん
垢版 |
2019/01/05(土) 22:55:30.05ID:ujon6Ql+
3^30
= 243^6
= (31×8 - 5)^6
≡ 31×8×(-5)^5×6 + (-5)^6
= -31×40×30×125 + 125^2
≡ -31×9×(-1)×(31×4 + 1) + (31×4 + 1)^2
≡ 31×9 + 31×4×2 + 1
= 31×17 + 1
0119132人目の素数さん
垢版 |
2019/01/06(日) 00:09:32.41ID:2xrPMjAH
>>118

ありがとうございます。

やっぱり試行錯誤するしかないということですね。
0120132人目の素数さん
垢版 |
2019/01/06(日) 01:24:05.51ID:pYT+uHU8
【1】-a-b+(b*q)-c+(c*r)-d+(d*s)-e+(e*t)-p-q-r-s-t+(a*p)+(a*q)+(a*r)+(a*s)+(a*t)+(b*p)+(b*q)+(b*r)+(b*s)+(b*t)+(c*p)+(c*q)+(c*r)+(c*s)+(c*t)+(d*p)+(d*q)+(d*r)+(d*s)+(d*t)+(e*p)+(e*q)+(e*r)+(e*s)+(e*t)

【2】-b-c+(c*r)-d+(d*s)-e+(e*p)+(e*t)-q-r-s-t+(a*t)+(b*q)+(b*r)+(b*s)+(b*t)+(c*q)+(c*r)+(c*s)+(c*t)+(d*q)+(d*r)+(d*s)+(d*t)+(e*q)+(e*r)+(e*s)+(e*t)

【3】-c-d+(d*p)+(d*s)-e+(e*p)+(e*q)+(e*t)-r-s-t+(a*s)+(a*t)+(b*t)+(c*r)+(c*s)+(c*t)+(d*r)+(d*s)+(d*t)+(e*r)+(e*s)+(e*t)

【4】-(a*r)-(c*p)-d-e+(e*p)+(e*q)+(e*r)+(e*t)-s-t+(a*t)+(b*t)+(c*t)+(d*s)+(d*t)+(e*s)+(e*t)

【5】-(a*q)-(a*r)-(a*s)-(b*p)-(b*r)-(b*s)-(c*p)-(c*q)-(c*s)-(d*p)-(d*q)-(d*r)-e-t+(e*t)

【6】-a+(a*p)-b+(b*q)-c+(c*r)-d+(d*s)-e+(e*t)-1+a+b+c+d+e

【1】=【2】=【3】=【4】=【5】=【6】のときa,b,c,d,e,p,q,r,s,tの値をそれぞれ求めよ。

これって答え出ますかね・・・?
mathematicaとか使ってもエラー出てくるんですよね・・・
0122132人目の素数さん
垢版 |
2019/01/06(日) 01:41:05.55ID:RGE3MgWR
微分方程式に詳しい人、これ解説してください。

g = g(ξ) , h = h(ξ)はR上のC1級関数として2階の常微分方程式

d^2w/dt^2 + g(w)(dw/dt) + h(w) = 0

を考える。正定数δ > 0 , κ > 0が存在し

てg , hは次の条件を満たすと仮定する。

g(ξ) ≧ κ , h(0) = 0 , h'(ξ) ≧ δ (ξ ∈ R)

とする。このとき、任意のa,b∈Rに対してw(0) = a , w'(0) = bとなるような解w(t)がt ≧ 0で存在することを示せ。またlim[t→∞]w(t) = 0を示せ。

ヒント:H(ξ) = ∫[0→ξ]h(η)dηとして

(d/dt){(1/2)(dw/dt)^2 + H(w(t))} = (dw/dt)((d^2w/dt^2) + h(w(t)))
0123132人目の素数さん
垢版 |
2019/01/06(日) 03:18:37.21ID:XR08xOo7
Uをd*d次元のユニタリ行列として,U^a = Iである.
ここに,aは2より大きな整数であり,Iはd*d次元単位行列である.
このとき,Uの固有値はそれぞれどのように表されるか?

Uがユニタリであることから,絶対値が1なのは明らかですが,それ以上に具体的に書き表すことができるようですが,どのように書けるのでしょうか?
0125M_SHIRAISHI
垢版 |
2019/01/06(日) 03:57:50.88ID:tGRzsmjc
>>88
>背理法は、A→B と A→¬B がともに成り立つことを示して、そこから ¬A を導く方法なのかと

いいや違う。

背理法とは、命題:「P(x) ならばQ(x) である」を証明したい場合に、
P(x) & not-Q(x) から xに関しての矛盾を導く方法を言う。

尚、これから論理的に導かれる方法として、命題A(α) を証明したい場合に
not-A(α) から α に関しての矛盾を導く方法もある。
0126132人目の素数さん
垢版 |
2019/01/06(日) 04:34:36.36ID:Jxai4k6k
ブッ
0128132人目の素数さん
垢版 |
2019/01/06(日) 16:34:35.86ID:hAkRPfB2
ある一次変換fがある。
どのような整数p,qについても、座標平面上の点(p,q)をfにより移すと、移った先の点のx座標またはy座標が整数であるという。
fとしてあり得るものをすべて、2×2行列の形で表せ。
0129132人目の素数さん
垢版 |
2019/01/06(日) 17:40:42.16ID:KhHSkFCq
ω>4を定数とする。
u=u(t),v=v(t)を未知関数とする方程式

du/dt=−ωv+(3−2u^2−v^2)u
dv/dt=ωu+(3−u^2−2v^2)v

を初期条件として(u(0),v(0))=(a,b)≠(0,0)を与える時、t≧0で解が存在することを示せ。また、周期解(U(t),V(t))も存在することを示せ。
0130132人目の素数さん
垢版 |
2019/01/06(日) 18:24:54.36ID:U1uFvPRA
次の不定方程式の整数解(x,y)を1組求めてください:
(2^2019)*x + ((2^10)-1)*y =1

たくさんありそうですけど、どれくらい解き方ありますかね?
0131132人目の素数さん
垢版 |
2019/01/06(日) 18:55:52.26ID:HXfdm1di
(2^10-1)(2^9(2^2010-1)/(2^10-1)x + y) + 2^9 x = 1
の整数解を求めればいい。
すなわち z=2^9(2^2010-1)/(2^10-1)x + y とおいて
1023 z + 512 x = 1
の整数解を求めれば良い。
0132132人目の素数さん
垢版 |
2019/01/06(日) 19:30:06.98ID:hU1YSmaF
2進数で考える。
@2^2019は1の後ろに0が2019個続き、これをa乗すると1の後ろに2019a個の0が続く数を作ることができる。
x=2^2019^(a-1)
A2^10-1は、1が10個続く数である。これにΣ(k=0 to b)(2^(10k))をかけると、1が10(b+1)個続く数を作ることができる。
y=-Σ(k=0 to b)(2^(10k))

そして、1の後ろに0がn個続く数から、1が(n-1)個続く数を引くと1になる。

よって次を満たすa,bを見つける。
2019a=n
10(b+1)=n-1
つまり、2019a-10b=11
a=9,b=1816

x=2^2019^8=2^16152
y=-Σ(k=0 to 1816)(2^(10k))

力技なんだけどね。エレガントな解き方ないの?
0133132人目の素数さん
垢版 |
2019/01/06(日) 20:11:37.45ID:AUBxw3Wu
f(n)をnの各桁の和として
f(n^2)=f(n)-7となるnは分かりますか?
出典が怪しいので解答可能かはわかりません
0139132人目の素数さん
垢版 |
2019/01/07(月) 08:01:11.18ID:LBx6GyyK
>>137 >>138
すみません
言葉足らずでした
形として、そのようになるのはわかるのですが、整数mはどこまで動くのでしょうか?
必ず1からaまで動くのでしょうか?
それとも、形としてexp(2πim/a)とはなりますが、mはどのような値をとるのかは分からず、場合によってはm=1がd個重複するということもあるのでしょうか?
0143132人目の素数さん
垢版 |
2019/01/07(月) 22:38:50.09ID:wWg0R9jm
x,yについての連立方程式
(s-t)x-ty=1
tx+(s-t)y=0
が-1<x<1かつ-1<y<1の範囲に解を持つとき、実数s,tが満たすべき条件を述べよ。
0144132人目の素数さん
垢版 |
2019/01/08(火) 02:26:53.87ID:WdFpB4mR
>>143
x + iy = 1/(s-t +it),

x = (s-t)/[(s-t)^2 + t^2],  y = -t/[(s-t)^2 + t^2],

(s-t)^2 ±(s-t) + t^2 > 0, (s-t)^2 + t^2 ±t > 0,

(s-t+1/2)^2 + t^2 > 1/4, 
(s-t-1/2)^2 + t^2 > 1/4,
(s-t)^2 + (t+1/2)^2 > 1/4,
(s-t)^2 + (t-1/2)^2 > 1/4,
の共通部分。(4楕円合併領域の外部)
0145132人目の素数さん
垢版 |
2019/01/08(火) 04:04:55.17ID:ZNcycGg9
煽りじゃなくて本当に疑問なんだけど、>>144の解法で複素数を引っ張り出してきたのはなぜ?
普通に連立方程式でx,yを求めるよりなにかいいことがあるの?
0148132人目の素数さん
垢版 |
2019/01/08(火) 16:44:31.80ID:/70NE4Ur
>>144
ありがとうございます。この問題の形がいかにも行列×ベクトルの形をしているのですが、一次変換に関係ある問題なのでしょうか?
0150132人目の素数さん
垢版 |
2019/01/08(火) 18:24:44.92ID:wfVVLLbn
広義積分 ∫ 1/(x(log(1+x^2))^β)dx from 1 to ∞ (β>0)
が収束するための条件はβ>1であることを示せ
をどなたか教えてください
0151132人目の素数さん
垢版 |
2019/01/08(火) 19:27:23.97ID:5EJbWTrs
12345→51234のように、十進表記の末尾一桁を先頭にもってくる操作を考える。
ただし末尾は0ではないとし、全桁が同じ数字からなる数も考えないものとする。
ある数Mをこの操作でLへと置き換えた。
MがLで割り切れるMのうち、桁数が最も小さいものを全て求めよ。

という問題なのですが、20点満点で5点しかもらえませんでした。

解答のどのあたりがまずいでしょうか?

https://i.imgur.com/KVfJSEP.jpg
0152132人目の素数さん
垢版 |
2019/01/08(火) 19:29:00.02ID:5EJbWTrs
結構自信あったのですが?で全部バツになってしまいました
言葉が足らずケタ数からの議論だということが伝わらなかったということでしょうか?
議論自体に不備がありますか
0153132人目の素数さん
垢版 |
2019/01/08(火) 20:54:22.86ID:iVwpyA+C
>>151>>152
t=4のときは互いに素ではないのに除外してるのはなぜ?
結局はt=7と同様の理屈で除外されるけども
どういう意味で同様の議論なのかを明確にすべき
「t=5,6,8,9のとき10-tとt*10^(n-1)-1は互いに素となるので、同様の議論で不適」といった感じに

あとは左端に詰めて書いてないので単純に読みづらい
0154132人目の素数さん
垢版 |
2019/01/08(火) 21:56:00.85ID:HcyYoAVi
Q上でd_pをp進距離関数、d_QをEuclid距離の部分距離関数として
距離空間(Q,d_p)と(Q,d_Q)を考えたときにこの2つが同相でないことを示せ

感覚的には分かるのですが証明方法が浮かばないのでお願いします……
0155132人目の素数さん
垢版 |
2019/01/08(火) 22:23:08.41ID:tzmf51gl
Walter Rudin 著『Principles of Mathematical Analysis』を読んでいます。

https://imgur.com/oT3qaqa.jpg

↑の赤線を引いた条件を満たすような V_(n+1) が存在することの証明ですが、↓であっていますよね?


V1 ∩ P ∋ x1 だから、 V1 ∩ P ≠ φ である。

V_n ∩ P ≠ φ と仮定する。

(A) x_n ∈ V_n の場合

V_n に含まれるような x_n を中心とする開球 B1 が存在する。

x_n ∈ P であり、 P は perfect set だから x_n は P の limit point である。

よって、 B1 は x_n 以外の P の点 x_k を含む。

V_(n+1) を x_k を中心とし、半径が min(B1の半径 - |x_k - x_n|, |x_k - x_n|) よりも小さい開球とすると、

(i) closure(V_(n+1)) ⊂ V_n
(ii) x_n ∈ closure(V_(n+1)) ではない。
(iii) V_(n+1) ∩ P は空ではない。(x_k を含む。)

(B) x_n ∈ V_n ではない場合

V_n ∩ P ≠ φ だから x_k ∈ V_n となるような k が存在する。

V_(n+1) を x_k を中心とし、半径が V_n の半径 - 「V_n の中心から x_k までの距離」 より小さい開球とすると、

(i) closure(V_(n+1)) ⊂ V_n
(ii) x_n ∈ closure(V_(n+1)) ではない。
(iii) V_(n+1) ∩ P は空ではない。(x_k を含む。)
0156132人目の素数さん
垢版 |
2019/01/08(火) 23:34:48.43ID:lM2d1hEM
数論幾何学とブロックチェーン技術ってどっちの方が独学するの難しいですか?
0157132人目の素数さん
垢版 |
2019/01/09(水) 00:06:19.10ID:3v0fKymE
>>150
 log(1+xx) = u,   (log(2) < u < ∞)
とおく。
 2x/(1+xx) dx = du,
 I[β] = ∫[log(2),∞] {(1+xx)/(2xx)} (1/u)^β du
一方、x≧1 より
 1/2 < (1+xx)/(2xx) ≦ 1,
 (1/2)∫[log(2),∞] (1/u)^β du < I[β] ≦ ∫[log(2),∞] (1/u)^β du,
かな・・・・
0158132人目の素数さん
垢版 |
2019/01/09(水) 00:27:00.66ID:h9JOR4rM
fはℝ上で定義された十分滑らかな関数とし,
a_n= {n Σ[k=1,n] f((2k-1)/2n)}-{n^2 ∫[0,1] f(x) dx}
について, lim[n→∞] a_nは収束するか?
その場合は極限値を求めよ。
0160132人目の素数さん
垢版 |
2019/01/09(水) 02:38:13.07ID:nRRCVLX/
>>154
(Q,d_p)においては
sup {d(a,x)|d(b,x)≦d(a,b)} = d(a,b) (∀a,b)
だけど(Q,d_Q)においては上は成立しない。
0161132人目の素数さん
垢版 |
2019/01/09(水) 03:34:34.38ID:zxxgNHWU
数学板利用したことがないのでここで質問させてください。
ボロノイ図のプログラムを組んでいるのですが、ランダムな点9個ほどからドロネー三角形の三点の座標と外接円の中心座上を求めることはできました。
そこからドロネー三角形からボロノイ図をつくりたいのですが、
ドロネー三角形に隣接している(辺を共有している)ドロネー三角形の外接円の中心を結んでいけばいように思ったのですが、あってますかね?
0162132人目の素数さん
垢版 |
2019/01/09(水) 09:29:08.86ID:p03RkPfT
実はこれからお話しすることは
公にはできない秘匿性の高い秘密なので
一度しかお話しできないということを
予めご理解ください・・・
0165132人目の素数さん
垢版 |
2019/01/09(水) 11:05:09.80ID:y0UYmX7x
んなわけないやん。
出題ミスかなんかしらんけど明らかに発散してる。
答えとしては発散する例あげておしまいだからf(x) = xで終わり。
0166132人目の素数さん
垢版 |
2019/01/09(水) 11:32:42.98ID:Iqt0a3E9
なんでこのスレって上から目線でレスするやつほど数弱なんだろうね
0167132人目の素数さん
垢版 |
2019/01/09(水) 12:25:07.86ID:VGWbLLfc
>>165
f(x)=xだと一次きえるのかorz。
ならf(x)=x^2で
n Σ[k=1,n] f((2k-1)/2n)} = 1/3(n+1)(4n^2-4n+1)×n/(4n^2)
n^2 ∫[0,1] f(x) dx} = n^2/3
で2次ローラン多項式。
1次は消えず。
0169132人目の素数さん
垢版 |
2019/01/09(水) 13:10:25.97ID:Ydwhnpwq
K_ε(x)平均0、分散εの正規分布の確率密度関数ですにおいて、δ>0とするとき、

∫[|x|≧δ]K_ε(x)dxはε→+0の時0に収束することを示せ。

ε→0とするとK_ε(x)はディラックのデルタ関数に近づいて感覚的に0になりそうなのは分かりますが、証明の仕方が分かりません。
0170132人目の素数さん
垢版 |
2019/01/09(水) 13:43:45.43ID:J7VIp4mr
正規分布でなくても積分が有限なだけで自明やん
分布固定で積分範囲を広げて収束するのと同じ
0172132人目の素数さん
垢版 |
2019/01/09(水) 16:19:20.96ID:ENgVnsAP
>>158
「区分求積 中点法 誤差」
で検索すると出てくる
愛知工業大学の講義資料に
誤差の評価計算があって
ほぼそのまま使える

結論だけ書くと
収束して値は −(1/2){f'(1)−f'(0)}
0173132人目の素数さん
垢版 |
2019/01/09(水) 16:20:07.97ID:hkMGGmBS
6x2+39x+54の因数分解

(6x+27)(x+2)
(3x+6)(2x+9)

どちらも正解ですか?それとも何か決まりがあってどちらかになる?
0174132人目の素数さん
垢版 |
2019/01/09(水) 16:22:47.99ID:oFfp0/ea
>>173
3(x+2)(2x+9)
まで分解しないと不正解とされそう
0175132人目の素数さん
垢版 |
2019/01/09(水) 17:58:58.38ID:fz5F2D1Y
>>173
じゃあテメエは3x+6を因数分解せよって言われたら3x+6って答えるのかよウスラハゲが。
0176132人目の素数さん
垢版 |
2019/01/09(水) 20:24:17.82ID:32+XtSYy
Y(ω)を求めるためにsinc関数と三角関数の積をフーリエ変換したいのですが解き方が分かりません。畳み込み積分を使うとは思うのですが、解法(出来れば途中式も)お教え頂けないでしょうか?
https://i.imgur.com/m4su9hx.jpg
0177132人目の素数さん
垢版 |
2019/01/09(水) 20:50:09.20ID:tWOuRuZ0
Pをカントール集合とすると、Pは((3*k+1)/3^m, (3*k+2)/3^m))の形の開区間とは交わらないことを証明せよ。
k, mは正の整数とする。
0180132人目の素数さん
垢版 |
2019/01/10(木) 03:23:20.50ID:4mlVGqNc
>>158
x_k = (2k-1)/(2n), とおく。
テイラーの定理
 f(x) = f(x_k) + f '(x_k)(x-x_k) + (1/2)f "(x_k)(x-x_k)^2 + (1/6)f '''(ξ)(x-x_k)^3,
を使う。ただし |ξ - x_k| ≦ |x - x_k|,  |f '''(ξ)| ≦ M,

∫[x_k -1/(2n), x_k +1/(2n)] f(x)dx
= (1/n)f(x_k) + (1/2)f "(x_k)∫[x_k -1/(2n), x_k +1/(2n)] (x-x_k)^2 dx + O(M/n^4)
= (1/n)f(x_k) + (1/nn)f '(x_k)・0 + (1/24n^3) f "(x_k) + O(M/n^4),

∴ nΣ[k=1,n] f(x_k) - nn∫[0,1] f(x)dx
= - (1/24n)Σ[k=1,n] f "(x_k) + O(M/n)
→ - (1/24)∫[0,1] f "(x)dx   (n→∞)
= - (1/24){f '(1) - f '(0)}    >>172

>>167
Σ[k=1,n] (2k-1)^2 = (1/3)(4nn -1)
1次の項は消える。
0182132人目の素数さん
垢版 |
2019/01/10(木) 08:37:15.01ID:0TNXVAe/
0 1 2 2 1 -2 -7 -15 -26 -41 -60 -84 -113 -148 -189

を一つの数式にするとどうなりますか?
0183132人目の素数さん
垢版 |
2019/01/10(木) 10:25:06.45ID:2GD8CjST
>>178
こんな問題すら解けないならホモトピーを学ぶ前に位相空間論の勉強をやり直すことをオススメします
0185132人目の素数さん
垢版 |
2019/01/10(木) 11:28:02.02ID:esm1ke7I
Sn=Σ[k=1,n] f((2k-1)/2n)}(2/2n) , S=∫[0,1]f(x)dx
とすると積分の定義から
ε=1/n³ に対して、∃n₀, n>n₀
|S-Sn|<ε=1/n³

a[n]=n²Sn-n²S だから
|a[n]|=n²|S-Sn|<n²ε=1/n
したがって
a[n]→0

>>180
これじゃダメなの?
0186132人目の素数さん
垢版 |
2019/01/10(木) 12:45:13.67ID:ViNWiWfk
ある本に書いてあった問題です。
この問題を解ける人は居ますか?

>大きな数はごく単純な状況でも簡単に生じる。
一例を挙げよう。
今度何かの退屈な委員会に出たときに考えるとよい問題だ。
その会議に出ている人々で構成しうる下部委員会が何通りあるか、すべて数え上げて、下部委員会にありうる対をすべて考える。
対のそれぞれを、二つの集団の一方に割り当てる。
どういう割り当てにしようと、四つの下部委員会について、すべての対が必ず同じ集団にあるという四つ組が必ずあって、全員が必ず偶数の下部委員会に属するようにする、元の委員会の最小の人数はいくらか。
0187132人目の素数さん
垢版 |
2019/01/10(木) 13:15:49.10ID:mS1xfsbs
>>182
((-1)^n-1)/16+(13n-n^3)/12+n^2/8
ではどうか
0188132人目の素数さん
垢版 |
2019/01/10(木) 15:33:53.59ID:4mlVGqNc
>>185
ε = 1/n^3 に対してある自然数 N が存在して

 |S_N - S| < ε,

| a[N] | = NN |S_N - S| < NNε = NN/n^3,

N ≦ O(n^{3/2}) なら収束するが…
0189132人目の素数さん
垢版 |
2019/01/10(木) 15:38:28.42ID:0aS1RbSq
質問です。

平均μ,分散共分散行列狽フ多変量正規分布をN(μ,)としたとき、狽ヘ非負定値行列であることを示せ

誰かわかる方お願いします。
0190132人目の素数さん
垢版 |
2019/01/10(木) 17:05:21.82ID:aFUafC37
準対角行列ってどんなものですか?
三角行列とは違うものですか?
0191132人目の素数さん
垢版 |
2019/01/10(木) 17:15:04.51ID:oQ6VnKQl
>>169
2乗可積分なX1〜Xnで分散共分散行列なるもの作ったらいわゆるグラミアンになるからじゃね?
0192132人目の素数さん
垢版 |
2019/01/10(木) 17:46:34.49ID:usfuY8n0
>>186
4人で成り立つ
証明は全通りを列挙すると場合の数が爆発するので
問題を言い換えてから論理学で解けばよい
例えば以下の問題と同値になる

問:
2進数でN桁以下の数 1, 2, ..., 2^N-1 を、2つの集合
P, Q のいずれかに振り分けて分割する。
どのような分割に対しても P, Q のどちらかに
4つの数 a, b, c, d が存在し、排他的論理和
XOR(a, b, c, d)=0となるとき、
Nの最小値を求めよ。
0193132人目の素数さん
垢版 |
2019/01/10(木) 17:50:48.61ID:dGlyJUHh
(X,d)を空でないコンパクト距離空間、f:X→Xを連続写像とする。

Xの空でないコンパクト部分集合Yでf(Y)=Yとなるものが存在することを示せ。

Zornの補題を使いそうな気はするけど、どうすれば解ける?
0198132人目の素数さん
垢版 |
2019/01/10(木) 23:29:09.20ID:4mlVGqNc
>>158
>>180 の改良)
x_k = (2k-1)/(2n), とおく。
テイラーの定理
 f(x) = f(x_k) + f '(x_k)(x-x_k) + (1/2)f "(ξ)(x-x_k)^2,
を使う。ただし (k-1)/n < ξ(x) < k/n,

∫[(k-1)/n, k/n] f(x) dx
= (1/n)f(x_k) + (1/2)∫[(k-1)/n, k/n] f "(ξ(x))(x-x_k)^2 dx
= (1/n)f(x_k) + (1/nn)f '(x_k)・0 + (1/2)(1/12n^3) f "(y_k)  (←平均値の定理)
ここに (k-1)/n < y_k < k/n,

∴ nΣ[k=1,n] f(x_k) - nn∫[0,1] f(x) dx
= - (1/24n)Σ[k=1,n] f "(y_k)
→ - (1/24)∫[0,1] f "(x) dx   (n→∞)
= - (1/24){f '(1) - f '(0)}    >>172
0199132人目の素数さん
垢版 |
2019/01/10(木) 23:33:36.16ID:2GD8CjST
>>195
それぞれの完備化が同相でないから
(実数体とp進数体、前者は連結空間だが後者は完全不連結)

というのはどうでしょうか
0200132人目の素数さん
垢版 |
2019/01/11(金) 00:28:54.65ID:6oXTloqV
lim[n→∞] Σ(k=1〜n) [{k^n}/{n^n}]
を求めたいのですが
Σ(k^a) がnについてのa+1次式(で、最高次の係数が正)になることを示せば(帰納法?)、Σ(k^n)がnについてのn+1次式になるから、求める極限は∞で良いですか?
0203132人目の素数さん
垢版 |
2019/01/11(金) 04:08:34.04ID:SKsrg5eA
>>200
Σ(k=1〜n) k^a 〜 {1/(a+1)}(n+1/2)^(a+1)   ・・・・ Faulhaberの公式
本問では a=n なので分母が 1/(n+1) となり nのn次式です。
それを n^n で割るので有限値に収束しそう。

2項公式から
 k^n ≒ {(k+1)^(n+1) - (k-1)^(n+1)}/(2n+2) - (n/12){(k+1)^(n-1) - (k-1)^(n-1)} - ・・・・,

Σ(k=1〜n) k^n ≒ {(n+1)^(n+1) + n^(n+1)}/(2n+2) - (n/12)(n+1)^(n-1) - ・・・・
 = {(n+1)^n + n^(n+1)/(n+1)}/2 - (n/12)(n+1)^(n-1) - ・・・・,

Σ(k=1〜n) (k/n)^n ≒ {(1+1/n)^n + n/(n+1)}/2 - (1/12)(1+1/n)^(n-1) - ・・・・
 → (e+1)/2 - (1/12)e - ・・・・   (n→∞)
 = e/(e-1)
 = 1.58197670687

∵ (1+1/n)^(n+1/2) → e  (n→∞)
0204132人目の素数さん
垢版 |
2019/01/11(金) 04:25:23.08ID:SKsrg5eA
>>203 訂正

Σ(k=1〜n) k^n ≒ {(n+1)^(n+1) + n^(n+1)}/(2n+2) - (n/12){(n+1)^(n-1) + n^(n-1)}- ・・・・
 = {(n+1)^n + n^(n+1)/(n+1)}/2 - (1/12){n(n+1)^(n-1) + n^n} - ・・・・,

Σ(k=1〜n) (k/n)^n ≒ {(1+1/n)^n + n/(n+1)}/2 - (1/12){(1+1/n)^(n-1) + 1} - ・・・・
 ≒ (e+1)/2 - (1/12)(e+1) - ・・・・
 = e/(e-1)
 = 1.58197670687    (n→∞)
0205132人目の素数さん
垢版 |
2019/01/11(金) 05:21:48.91ID:SKsrg5eA
>>190
 quasi-diagonal matrix

固有値に重根がある場合、それに対応するブロックは対角化できないことがある。
その場合でも、対角線上に固有値、その1つ上(下)に1が並ぶ形に変換することは可能。
(ジョルダンの標準形)

互いに複素共役な固有値 a±bi がある場合、それらに対応するブロックを
[a+bi, 0]
[0, a-bi]
から実表示
[a, -b]
[b, a]
に変換することがある。
0206132人目の素数さん
垢版 |
2019/01/11(金) 06:48:11.97ID:Dq90IXlW
>>201
元の問題の
「全員が偶数の下部委員会に属する」
の偶数は0を含まないのでは、
という指摘と理解しました

自分は0を含むとして解きましたが
これを排除すると
答えが変わるかもしれませんね

投稿者はヤフー知恵袋にも
投稿しているようなので
証明の掲載はいったん控えます
0208132人目の素数さん
垢版 |
2019/01/11(金) 11:45:12.82ID:KwypqMKz
二乗可積分な確率変数 X,Y に対して内積 (X,Y) を E(XY) で定めた場合のグラミアンが分散共分散行列。
なので有限次元のヒルベルト空間のグラミアンが任意の基底で固有値正を示せば良い。
0210132人目の素数さん
垢版 |
2019/01/11(金) 13:22:30.95ID:xPuURV4g
サイコロを振り、1,2,3の目が出たときは文字列AABを書き、4のときは文字Bを、5のときはCを、6のときはDを書く。
このことを繰り返し、既にある文字列の右側につなげて新しい文字列を作る。
例えばサイコロを3回投げ、順に5,1,4の目が出たときは、文字列CAABBが得られる。

(1)nを正整数とし、サイコロをn回投げて文字列を作る。
文字列の一番左からn番目の文字がAとなる確率P_A[n]、文字列の一番左からn番目の文字がBとなる確率P_B[n]をそれぞれ求めよ。

(2)極限 lim[n→∞] P_B[n]/P_A[n] を求めよ。
0211132人目の素数さん
垢版 |
2019/01/11(金) 14:16:14.48ID:1RH5/468
>>193
x(1)=f(X), x(n+1)=f(X(n)) として Y=∩{X(n) | n∈N} にすれば
Y≠φ はコンパクトから言えるからZornの補題は要らんだろ
0212132人目の素数さん
垢版 |
2019/01/11(金) 15:55:34.59ID:5J5uogTQ
>>199
よく考えたらユークリッド距離の(0,1)とRが同相でも完備化して同相でなくなる例でしたね
0213132人目の素数さん
垢版 |
2019/01/11(金) 16:51:41.30ID:r1mV13ck
n次元アフィン空間の平行でない2つのn-1次元アフィン部分空間の交わりはn-2次元アフィン部分空間になることの証明を教えてください
0216132人目の素数さん
垢版 |
2019/01/11(金) 21:59:51.35ID:hiaVndzB
>>214
先頭から 0,1,2 と続くので、この問題の場合は初項を a_0 としたほうが綺麗かなと個人的には思う
まあ、趣味の問題でしかないですが
0218132人目の素数さん
垢版 |
2019/01/11(金) 22:28:08.13ID:XTVF40zW
0
0221132人目の素数さん
垢版 |
2019/01/12(土) 00:11:38.32ID:V06NzB18
>>219

「われは過程をつくらず」 (Hypotheses non fingo)
--- Isaac Newton "Principia Mathematica, Philosophiae Naturalis" 2nd ed.(1713)
0222132人目の素数さん
垢版 |
2019/01/12(土) 00:14:42.36ID:V06NzB18
>>220
ぢゃあ生姜ねぇ

・解1
sin(x) < x < tan(x) より
0 < 1/sin(x) - 1/x
 < 1/sin(x) - 1/tan(x)
 = {1-cos(x)}/sin(x)
 = sin(x)/{1+cos(x)}
 → 0 (x→0)
http://detail.chiebukuro.yahoo.co.jp/qa/question_detail/q1036757970 (2010/02/15)

・解2
lim (x→0) {1/x - 1/sin(x)}
= lim (x→0) {sin(x)-x}/{x・sin(x)}
= lim (x→0) {cos(x)-1}/{sin(x)+x・cos(x)}  (← l'Hospital)
= lim (x→0) {-sin(x)}/{2cos(x)-x・sin(x)}  (← l'Hospital)
= 0
http://detail.chiebukuro.yahoo.co.jp/qa/question_detail/q1363539883 (2011/06/01)
0224132人目の素数さん
垢版 |
2019/01/12(土) 14:02:35.27ID:2CN4f8o0
>>210
誰かこれ解けませんか?
大学入試問題らしいので特別難しくはないと思います
私は漸化式作ろうとして作れませんでした
0226132人目の素数さん
垢版 |
2019/01/12(土) 16:07:25.11ID:F/0/MLYr
高校数学のスレで誰も解いてくれないので誰か教えてください。高校数学の問題です。
xyz空間において
C1, x^2+y^2=1,x≧0,y≧0,z=0

C2, x^2+z^2=1,x≧0,y=0,z≧0

C3, z^2+y^2=1,x=0,y≧0,z≧0 を考える。

点Pがx軸の0≦x≦1の部分を動くとき、Pを通りx軸に垂直な平面とC1,C2の交点を順にQ,R として、三角形PQRが通過してできる立体をK1とする。

同様に、点P' がy軸の0≦y≦1 の部分を動くとき、P'を通りy軸に垂直な平面とC1,C3との交点を順にQ',R'として、三角形P'Q'R'が通過してできる立体をK2とする。
このとき、K1とK2の共通部分K の体積を求めよ。
0227132人目の素数さん
垢版 |
2019/01/12(土) 21:10:07.56ID:ilBN4PEM
>>226
線分QR, Q'R' が削り出す立体は 平面 x=y に関して対称的である。
例えば QR が削り出す側 ( y ≧ x ) の体積を2倍すればよい。
よって
底辺: a(x) = √(1-x^2) - x
高さ: h(x) = √(1-x^2) - x
この三角形面積を積分して2倍すればよい。

V{K} = 2 ∫ [ 0, 1/√2 ] dx a(x) h(x) / 2 = ∫ [ 0, 1/√2 ] dx ( 1 - 2x √(1-x^2) )
= 1/√2 - ∫ [ 0, 1/2 ] dt √(1-t)
= √2 /2 +2/3 ( (1-1/2)^{3/2} - (1-0)^{3/2} )
= √2 /2 +√2 /6 - 2/3 = 2/3 ( √2 - 1 )

てか本当に高校数学か?
0228132人目の素数さん
垢版 |
2019/01/12(土) 23:30:46.84ID:F/0/MLYr
>>227ありがとうございます。この問題は東大の添削問題として学校で出された問題なのですが全然わからなかった問題です。
0231132人目の素数さん
垢版 |
2019/01/13(日) 02:33:31.12ID:RQPCLdmu
>>230
原点を選ぶことで全空間をベクトル空間と見なす
適当な平行移動をすることによりアフィン部分空間はどちらも(ベクトル空間としての)部分空間としてよい
こう考えれば主張は明らかでしょう
0235132人目の素数さん
垢版 |
2019/01/13(日) 09:28:18.41ID:OocgZ7X3
>>210
a_1 = 1
a_2 = 1/2
a_3 = 1/4
a_n = (1/2)*a_(n-3) + (1/2)*a_(n-1)

P_A[n] = (1/2)*a_(n-1)+(1/2)*a_n

じゃ間違いですか?
0236132人目の素数さん
垢版 |
2019/01/13(日) 15:52:05.16ID:MydbhMAn
>>210
AAB を “A1”, “A2”, “B1” の並びに, 4のときのBを “B2” に解釈するとよさそう
A1[n] = 1/2 . (1 - A1[n-1]-A2[n-1])
A2[n] = 1 . A1[n-1]
B1[n] = 1 . A2[n-1]
B2[n] = 1/6 . (1 - A1[n-1] - A2[n-1]) = 1/3 . A1[n]
C[n] = 1/6 . (1 - A1[n-1] - A2[n-1])
D[n] = 1/6 . (1 - A1[n-1] - A2[n-1])
(総和が1 になる事を確認)

∴ A1[n] = 1/2 . (1-A1[n-1]-A1[n-2])

A1’[n] := A1[n] - 1/4 と置くと
A1’[n] = - 1/2 ( A1’[n-1] + A1’[n-2] ). [漸化式]
二次方程式: 2x^2 + x + 1 = 0 の解
α= {-1 + √(-7) }/4 , β= {-1 - √(-7) }/4
|α| = |β| = 1/√2 < 1
α^n , β^n は漸化式を満たす (特解)

[漸化式]が線形なので一般解は
a1 [n] = a1’[n] + 1/4 = p α^n + q β^n + 1/4

初期条件
a1[1] = 1/2, a1’[1] = 1/4
a1[2] = 1/4, a1’[2] = 0
p α + q β = 1/4
p α2 + q β2 = 0. より p=…, q = … (略)

A[n] = A1[n] + A2[n] =( p α^n+q β^n) + ( p α^{n-1}+q β^{n-1}) + 1/2 → 1/2
B[n] = B1[n] + B2[n] = A2[n-1] + 1/3 . A1[n] = A1[n-2] + 1/3 . A1[n]
= ( p α^{n-2}+q β^{n-2} ) + 1/3. ( p α^n + q β^n ) + 1/3 → 1/3

∴ B[n]/A[n] → 2/3
0237132人目の素数さん
垢版 |
2019/01/13(日) 15:53:08.32ID:MydbhMAn
後半は単純に見積もる事も可能 (厳密ではない)
n回平均で 1/2. 3n + 1/2. n = (2n)文字が生成される
そのうち (3n/2)文字は {A1,A2,B}のどれか (n/2)文字は {B2,C,D} のどれか
nが大になるほど、およそ中央の n文字目は特徴がなくなるはず.
A[n] → 3/4. 2/3 = 1/2
B[n] → 3/4. 1/3 + 1/4. 1/3 = 1/3

∴ B[n]/A[n] → 2/3
0238132人目の素数さん
垢版 |
2019/01/13(日) 17:02:38.39ID:bVbmlIcM
>>231
直感的に、部分空間の共通部分の基底を、それぞれの部分空間の基底の同じ方向を向いているやつたちから選べると思いますが、これは厳密に証明として書くにはどのようにすればいいでしょうか?
0240132人目の素数さん
垢版 |
2019/01/13(日) 21:31:11.63ID:RQPCLdmu
>>238
基底を書き下すことによって証明をするのは不可能ではないと思いますが、手間がかかってしまうと思います

Vを全空間、AとBを部分空間として、線形写像
f:A×B→V ;(a,b)→a-b
に対し次元定理を適用すると楽に示せます
(Im(f)=V, Ker(f)=A∩Bです)
0243132人目の素数さん
垢版 |
2019/01/14(月) 00:19:46.78ID:61q6TCzp
>>189, >>239
> 平均μ,分散共分散行列狽フ多変量正規分布をN(μ,)としたとき、狽ヘ非負定値行列であることを示せ
狽フ, 狽ヘ が何だか分からんが

f(x) を確率密度関数とする.
分散共分散行列 M{i,j} := E[ (xi-μi)(xj-μj) ] = ∫ dx1...∫ dxn (xi-μi).(xj-μj). f(x) は対称行列 ★
ゆえに適当な回転行列 R により対角化可能
(R.M.R^t){i,j} = Σ[k=1,n]Σ[m=1,n] ∫ dx1...∫ dxn (xk-μk).(xm-μm).R{i,k} R{j,m} f(x) = δ{i,j} λi = Λ{i,j}
λi = Σ[k=1,n]Σ[m=1,n] ∫ dx1...∫ dxn (xk-μk).(xm-μm).R{i,k} R{i,m} f(x)
= Σ[k=1,n]Σ[m=1,n] ∫ dx1...∫ dxn | R.(x-μ) |^2 P(x) ≧ 0
よって Λ = R.M.R^t は非負定値行列 → M は 非負定値行列 ★

あるいは...
https://mathtrain.jp/tahenryogauss この多変量正規分布関数に現れる対称行列 Σ について考える.
適当な対角化により Λ= R.Σ.R^t, y = R.(x-μ )
f(x) ∝ exp( - (1/2). (x-μ)^t. Σ^{-1}. (x-μ) ) = exp( - (1/2). y^t. Λ^{-1}. y ) = exp( - (1/2). Σ[i=1,n] yi^2 / λi )
確率密度関数の積分が有限となるように λi > 0 となっている必要がある.
よって Λ は 正定値行列 → Σ は 非負定値行列 ★
∫ dx1...∫ dxn yi. yj. f(x) = ∫ dy1...∫ dyn yi. yj. f(...y...) = ... = δ{i,j} λi = Λ{i, j}
(∵ det Σ = λ1.λ2....λn , ∫ dy ye^-y^2=0, ∫ dy e^-y^2 = √(π),  ∫ dy y^2 e^-y^2 = Γ(3/2) = √(π) /2 )
よって Σ = R^t.Λ.R = ∫ dx1...∫ dxn (R^t.y){i}.(R^t. y){j} f(x) = ∫ dx1...∫ dxn (xi-μi).(xj-μj) f(x)
確かに Σ は "分散共分散" 行列である. ★
0244132人目の素数さん
垢版 |
2019/01/14(月) 08:36:56.29ID:iN2QCevu
nを自然数とする。
fn(1) = ∫[0 to 1] 1/(1+x^2n) dx
について、n=1,2,...に対する増減を調べよ。
0245132人目の素数さん
垢版 |
2019/01/14(月) 17:07:24.04ID:VbLIcfRT
>>244
 nについて単調増加。
1 - x^(2n) < 1/{1+x^(2n)} < 1
より
1 - 1/(2n+1) < I[n] < 1

∴ I[n] →1  (n→∞)

I[1] = ∫[0,1] 1/(1+x^2) dx = [ arctan(x) ](x=0,1) = π/4 = 0.7853981634

I[2] = ∫[0,1] 1/(1+x^4) dx = {π + 2log(1+√2)}/(4√2) = 0.86697298734

I[3] = ∫[0,1] 1/(1+x^6) dx = {π + (√3)log(2+√3)} / 6 = 0.90377177375
0246132人目の素数さん
垢版 |
2019/01/14(月) 19:33:04.92ID:1ixFLnb2
大学入試の過去問程度の問題で、
むちゃくちゃ難しい立体求積の問題を集めたものはありますか?
オンラインで利用できるものを探しております。お願いいたします
0247132人目の素数さん
垢版 |
2019/01/14(月) 19:38:47.90ID:1ixFLnb2
「むちゃくちゃ難しい」は「平均的受験生にとって、数学がある程度得意でもきつい」というような意味です。
なにとぞよろしくお願いします
0248132人目の素数さん
垢版 |
2019/01/14(月) 21:13:05.39ID:JExPOrqO
昔質問した内容について、再度教えてください。

●質問内容
(x1, y1), (x2, y2), …, (xn, yn)とする、n個のxとyの値が分かっているペアがあります。これらが以下の方程式
y = a * {sinh(bx)}^c
を満たす場合、最小二乗法を使って係数a, b, cを求める方法を教えてください。

●当時の回答
b=b0 を固定する。
log(y) = log(a) + c・log|sinh(bx)|より
 X = log(sin(bx)),Y = log(y)
とし、(X,Y)データを最小二乗法で直線回帰する。
 Y = log(a) + c・X
ただし、(a,c) は b0 に依存する。
次に、
Z = sinh^(-1){(y/a)^(1/c)}
とし、(x,Z)データを最小二乗法で直線回帰する。
Z = b・x + d,
ただし、(b,d)は(a,c)に依存する。
これを (a,b,c,d) が収束するまで繰り返す。

●質問内容
回答のZ = b・x + dのbとdは、どのように分解され、Y= の式に当てはめればよいのでしょうか。
また、可能であれば、数回繰り返した結果を示していただけないでしょうか(係数a,b,cは任意でかまいません)。
0249132人目の素数さん
垢版 |
2019/01/14(月) 23:15:18.08ID:9eUtE+7q
初めて来ました。
私立の経済学部に通ってるものです。
画像の例題の解説に疑問があります。
(1)でμをそれぞれ正と置いたため解なしになるのは理解できるのですが、
(4)では解なしにならない理由がわかりません。
https://i.imgur.com/s3feGFL.jpg
0250132人目の素数さん
垢版 |
2019/01/14(月) 23:16:38.05ID:9eUtE+7q
>>249
教えていただけたら幸いです。
0252132人目の素数さん
垢版 |
2019/01/14(月) 23:38:23.92ID:mRxaT4D4
>>249-250
(1)でも(4)でもg1=0、g2=0を解いて
(x1,x2) = (±√2,1)
までは同じ。
ここから
(1)では
μ0(-1,-1) + μ1(∓2√2,2) + μ2(±2√2,2) = 0、μ0=0
を解くと
(μ0, μ1, μ2) = (0,0,0)になり条件に反する。
(4)では
(-1,-1) + λ1(∓2√2,2) + λ2(±2√2,2) = 0
を解くまでもなく
(μ0, μ1, μ2) = (1,λ1,λ2)だからλ1,λ2が何であろうと条件に適する。
0253132人目の素数さん
垢版 |
2019/01/14(月) 23:43:23.26ID:1ixFLnb2
>>251
うーん、問題をジャンル別に分けてる人が一人もいないと断定できる理由が分かりかねます。
市販の問題集でも過去問をジャンル別に分けますよね
0254132人目の素数さん
垢版 |
2019/01/14(月) 23:52:24.89ID:R6IO2Dcx
自分で検索したらいいじゃん。
それで見つからないなら、オンラインで手に入るものはないと思っていた方が精神衛生上、吉。
0255132人目の素数さん
垢版 |
2019/01/14(月) 23:55:40.42ID:1ixFLnb2
ここの数学通の皆様なら何か良サイトをご存知かと思いましたがそういうものでもないのですね。
0256132人目の素数さん
垢版 |
2019/01/15(火) 00:10:21.27ID:FAag7PrR
>>252
μ0がゼロではないことを忘れていました。
2λ1+2λ2=0だと勘違いしていました。
解決しました。有難うございます。
0257132人目の素数さん
垢版 |
2019/01/15(火) 00:40:46.17ID:Vp0pEVqH
行列[[cosθ,-sinθ],[sinθ,cosθ]]の最小多項式と標準形はどのように求めればよいでしょうか
場合分けが必要らしいのですが上手くいかず困っています
0258132人目の素数さん
垢版 |
2019/01/15(火) 01:42:04.67ID:uJxU5NAt
>>248

Z(i) = sinh^(-1){(y(i)/a)^(1/c)}
としたので、これは(a,c)に依存します。

(x, Z) で最小二乗法により直線回帰すると
b = {n(Σx・Z) - (Σx)(ΣZ)}/{n(Σxx) - (Σx)^2}
d = {(Σxx)(ΣZ) - (Σx)(Σx・Z)}/{n(Σxx) - (Σx)^2}

b0 としてこの b を使い、(X,Y) = (log|sinh(b0・x)|, log(y)) で直線回帰する。
log(a ') = {(ΣXX)(ΣY) - (ΣX)(ΣXY)}/{n(ΣXX) - (ΣX)^2}
c ' = {n(ΣXY) - (ΣX)(ΣY)}/{n(ΣXX) - (ΣX)^2}
0259132人目の素数さん
垢版 |
2019/01/15(火) 03:56:00.38ID:uJxU5NAt
>>248

(a,b,c) を一度に最適化するのはムズいようなので、
(a,c) と b を交互に最適化しようというわけです。
0260132人目の素数さん
垢版 |
2019/01/15(火) 04:05:58.19ID:uJxU5NAt
>>258
そうすると
 y = a・{sinh(bx+d)}^c
の形に最適化することになりますな。

d=0 に固定するなら簡単に
 b = (ΣZ)/(Σx)
でいいかも・・・
0262132人目の素数さん
垢版 |
2019/01/15(火) 12:51:26.94ID:RzwAs1D2
さいころを繰り返し振って,出た目を足してゆくとき, いつか和が1995となる確率は?

これ解ける方いますか?
大数の問題だそうなので結構難しいのではという気がしますが

適当に大きいNに対して
6P(N)=P(N-1)+P(N-2)+P(N-3)+P(N-4)+P(N-5)+P(N-6)という漸化式が成り立つのは分かりますが、
これ現実的に解ける数値になるんですかね?
0263132人目の素数さん
垢版 |
2019/01/15(火) 14:21:14.03ID:i71VFB19
>>262
大数の法則って言うんだったら、おおよそ1/6でいいのでは
0264132人目の素数さん
垢版 |
2019/01/15(火) 15:28:07.31ID:6HF/6uz4
a[n] = 0 (if n < 0)
. 1 (if n=0)
. (a[n-1] + a[n-2] + … + a[n-6])/6 (otherwise)
の1995項。
0265sage
垢版 |
2019/01/15(火) 15:44:17.83ID:vrOKAv8c
>>262
1-P(N) = P(N-1)*(5/6)+P(N-2)*(4/6)+P(N-3)*(3/6)+P(N-4)*(2/6)+P(N-5)*(1/6)
なので、Nが十分大きい時にはP(N-5)〜P(N)はほぼ等しい(=p)とみなすと
1-p = ((5+4+3+2+1)/6)*p より
p=2/7
0266132人目の素数さん
垢版 |
2019/01/15(火) 15:48:26.28ID:zvmq7Hj6
ああ、漸化式はわかってんのか。
概算値なら一回平均7/2回進むのでn回振って止まるマスの割合は
2/7 = 0.2857142857142857。
明示的な解がほしいなら特性方程式
x^6 = 1+x+x^2+…+x^5
の解のn乗和をt[n]としてNewtonの漸化式
t[1] = 1/6, t[2] = t[1]^2 - 2t[2], …
でt[0]〜t[10]まで求めて
p[n] = a t[n] + b t[n+1] + c t[n+2] + … + f t[n+5]…(*)
とおいてn = 0〜5代入して得られる連立方程式とけばすくなくとも(*)の形の表示なら得られる。
ちなみに
Prelude> import Data.Ratio
Prelude Data.Ratio> let ps = map head $ iterate (¥x-> ((sum $ take 6 x)*(1%6):x)) [1,0,0,0,0,0,0]
Prelude Data.Ratio> ps!!1995
でHaskellは答えだしてくれる。
レス長すぎで答え書き込めないけど。
数値的には 0.2857142857142857 でさっきの数値と合う。
ちなみに 6/(6-(x+x^2+x^3+…+x^6)) のマクローリン展開のx^1995の係数なのでコーシーの定理で数値解出す手もあるとは思う。
でもほぼほぼ2/7なので面白くともなんとも。
0269132人目の素数さん
垢版 |
2019/01/15(火) 17:32:21.70ID:lgsxjTiv
次の式を0≦n≦13の範囲で出力するとどうなりますか?

(-4n^14+355n^13-14222n^12+339911n^11-5395962n^10+59933445n^9-477806186n^8+2758860533n^7
-11489843794n^6+33915953500n^5-68528878392n^4+89002832256n^3-65833050240n^2+20597068800n
+56043187200)
――――――――――――――――――――――――――――――――――――――――――――――
(-7n^14+754n^13-35243n^12+955526n^11-16846401n^10+204374742n^9-1755671489n^8+10800459098n^7
-47463039052n^6+146597126104n^5-307636013568n^4+412169486976n^3-312533130240n^2
+99632332800n+224172748800)
0270132人目の素数さん
垢版 |
2019/01/15(火) 22:12:54.41ID:vA3smCvo
m次多項式h_m(x)を
h_m(x)=(1-x)^m
と定める。
また、あるn次多項式f(x)に対し、g_m(x)を
g_m(x)=h_m(x)f(x)
とおく。
以下の問に答えよ。

(1)任意のmに対してg_m(x)の各項の係数が整数ならば、f(x)の各項の係数はすべて整数であることを示せ。

(2)任意のmに対してg_m(x)の各項の係数が整数であることは、f(x)の各項の係数がすべて整数であるための必要十分条件か。
0275132人目の素数さん
垢版 |
2019/01/16(水) 00:43:04.54ID:tTsmHG60
すみません
これどこがダメなんですか

-1=i×i
=√(-1)×√(-1)
=√((-1)×(-1))
=√1
=1
つまり、-1=1
0278132人目の素数さん
垢版 |
2019/01/16(水) 01:08:04.79ID:Se1z58YA
√-x*√-yのように、負数の根号をそのまま計算してはいけない。
正の数で成り立つ結合法則が成り立たないから。
すぐi√xの形に戻してから計算すること。


√-x * √-y = i√x * i√y = -√xy

結合法則を適用してしまうと、下のような誤答になる。
√(-x)*(-y) = √xy

なぜこうなるかというと、√は、「2つの根号のうち、正の方を取る」という人間が勝手に付けてる不自然な機能があるから。
0286132人目の素数さん
垢版 |
2019/01/16(水) 04:46:14.37ID:lOjtUToz
>>269
 A = n(n-1)・・・・(n-12)
 B = n(n-1)・・・・(n-12)(n-13)
とおくと
(与式) = (9(13! - A) - 4B)/(36・13! +117A-7B),
nが整数ならば B=0
 = 1/4    (n=0,1,・・・・,12 のとき)
 = 0,     (n=13 のとき)
0287132人目の素数さん
垢版 |
2019/01/16(水) 05:16:39.99ID:lOjtUToz
>>282 [問題25]

第二種ルジャンドル関数
 v(x) = x・log((1+x)/(1-x)) -1
の方だろ。
出し方は、ロンスキアンを
 Wr(x) = det{[u(x), v(x)] [u '(x), v '(x)]}
として
 dWr/dx = -{2x/(1-xx)}Wr,
 Wr(x) = Wr(0)(1-xx),
を解くと思う。

u(x) = x は問題文中にある。
0288132人目の素数さん
垢版 |
2019/01/16(水) 06:21:44.98ID:lOjtUToz
>>263
いや、ここは数セミの法則でござる。

特性多項式
 6t^6 + 5t^5 + 4t^4 + 3t^3 +2t^2 + t = 0
の根(特性根)を α、β、β~、γ、γ~、0 として
P(N) = 2/7 + a・α^N + {b・β^N + b~・(β~)^N} + {c・γ^N + c~・(γ~)^N}

 α = -0.670332
 β = -0.375695 + 0.570175i = 0.682822 e^(2.15341i)
 γ = 0.294195 + 0.6668367i = 0.7288497 e^(1.15530i)
どんぐりの背比べでござる。
0289132人目の素数さん
垢版 |
2019/01/16(水) 08:28:08.11ID:Omn5Op3o
Min. x^2 + 2xy + 2y^2 + 4x - 5y
s.t. 2x^2 + 3x + 4y^4 <=10

を、補助変数を用いて、ベクトルvに関する
Min. 1/2(v^t)Q[0]v + (p[0]^t)v
s.t 1/2(v^t)Q[i]v + (p[i]^t)v + r[i] <= 0
i=1,...,n
という形に書き直すことって可能でしょうか?
(v^t)はベクトルvの転置です。
0290132人目の素数さん
垢版 |
2019/01/16(水) 09:20:21.83ID:sDktBDlu
>>282, >>287
y = x g(x) と置いて ( u(x) = x は g(x) くくり出しのヒントかも? )
(1-xx) (2 g' + x g'') - 2x (g + x g') + 2 (x g) = 0
x(1-xx) g'' + 4(1-xx) g' - 2 g' = 0
g''/g' = -4/x + (1/x)( 1/(1+x) + 1/(1-x) ) = 0
g''/g' = -4/x + (1/x - 1/(1+x)) + (1/x + 1/(1-x)) = 0
log(g') = - log(xx) - log(1+x) - log(1-x) + log(C1)
g' = C1 (1/xx)( 1/(1-xx) ) = C1 (1/xx + 1/(1-xx)) = C1 ( 1/xx + (1/2)(1/(1+x)+1/(1-x)) )
g = C1 ( -1/x + (1/2) log( (1+x)/(1-x) ) + C2
y = x g = C1 ( -1 + (1/x)log( (1+x)/(1-x) ) + C2 x
0291132人目の素数さん
垢版 |
2019/01/16(水) 12:35:29.29ID:cRnFoAvw
>>273
こちら分かる方いませんか?
Zの閉包がs/t(tとpは互いに素)の全体の集合となることは分かったのですがそこから困っています
0292132人目の素数さん
垢版 |
2019/01/16(水) 12:45:15.86ID:62PxuIhP
>>269
■正の整数nに対して

α=(n^2-13n)^6+182(n^2-13n)^5+13468(n^2-13n)^4+516360(n^2-13n)^3+10752768(n^2-13n)^2+114341760(n^2-13n)+β

β=479001600

とおいて

与式は(117β-4αn^2+43αn)/(468β-7αn^2+208αn)

出力は0≦n≦13の範囲で

1/4
1/4
1/4
1/4
1/4
1/4
1/4
1/4
1/4
1/4
1/4
1/4
1/4
0293132人目の素数さん
垢版 |
2019/01/16(水) 13:10:48.70ID:30GFXTVf
f(x)をn次多項式とする。

(1)任意の正整数mに対し、多項式f(x)*(1-x)^mの各項の係数がすべて整数ならば、f(x)の各項の係数はすべて整数であることを示せ。

(2)任意の正整数mに対してf(x)*(1-x)^mの各項の係数が整数であることは、f(x)の各項の係数がすべて整数であるための必要十分条件か。
0296132人目の素数さん
垢版 |
2019/01/16(水) 16:19:27.05ID:Omn5Op3o
>>289
解決しました
お騒がせしました
0298132人目の素数さん
垢版 |
2019/01/16(水) 18:13:14.55ID:lOjtUToz
>>292

α・n = n(n-1)(n-2)・・・・(n-12) = A
α・n(n-13) = B,
β = 12!
13β = 13!
なので
(与式) = {9(13β-α・n) -4(n-13)α・n} / {36(13β) -(7n-208)αn}
 = {9(13!-A) - 4(n-13)A} / {36(13!-A) -7(n-13)A +153A}
 = {9(13!-A) - 4B} / {36(13!-A) -7B +153A}    >>286
0301132人目の素数さん
垢版 |
2019/01/16(水) 22:12:26.95ID:PyxAa7VX
斎藤毅の「集合と位相」181ページA7.1.3.2が分かりません

N^N :={自然数列(0を含む) (a_n) | {n|a_n ≠0} は有限集合}とおく
g : N^N → N,をa=(a_n)に対して
(a_n) = ゼロ列 の時は、g(a) = 0,
(a_n) ≠ゼロ列の時は、m:=max{n|a_n≠}として、
m=0 なら g(a) = 2^{a_0}
m>0 なら g(a) = 2^{a_0} ( 1 + 2^{a_1 + 1} (1 + … (1+2^{a_{m-1}+1}(1+2^{a_m}))…))
で定義する。
この時gは可逆写像であることを示せ

なのですが、その証明がイマイチ理解できません
単射であることは容易に検証できますが
全射であることが分かりません

例えばg^{-1}(1) は何になるのでしょうか?
0302132人目の素数さん
垢版 |
2019/01/16(水) 23:33:34.02ID:PyxAa7VX
>>301の写像gについて3日ほど考えてるんですが、やっぱりこのgって作り方間違ってると思います
証明も間違ってると思います。穴があるというか…
このgでは全射が成り立ちません
0303248
垢版 |
2019/01/16(水) 23:55:15.30ID:CyFUI9pW
>>258
ありがとうございます。週末に考えて見ようと思います。
0304132人目の素数さん
垢版 |
2019/01/16(水) 23:59:33.07ID:sDktBDlu
>>301
g(a)の表式からパターンを見出してその意を汲むと...

本当はこんな風にしたいはず.
a {10進} → g(a) {2進}
0, → 0
1, → 1
2, → 10
3, → 100
0,1, → 11
0,2,→ 101
1,1, → 110
1,2, → 1010
1,2,3, → 10010010
1,0,3,4,2,3, → 100100100001000110

m が 2進桁に 1 が立つ総数に対応,
下位桁からの連続する0のカウントが数列の各値に対応する (ただし最上位の1は 「0のカウント」に含める).
これなら g が 全単射なのは明らかですよね?
そして g^{-1}(1) = {1,0,0,0,...} です.

その表式は
g(0)=0
m=0: g(a)=2^{a[0]-1}
m≧1: g(a)= 2^a[0](1 + 2^{a[1]+1}....(1 + 2^{a[m-1]+1}(1+2^a[m]) )...))
となります.
まあ1週間で1ページも進まない...とかよくあるので 3日くらい大した事ないない.
0305132人目の素数さん
垢版 |
2019/01/17(木) 00:48:41.29ID:srr7Syej
>>304
詳しい説明ありがとうございます。

gによって何がしたいのかという意図は僕自身でも読み取ってはいましたが、境界的な微妙な値をどういう風にしたら全射性まで担保できるのか
をあれこれ悩んでいました。
やっぱりm=0ならg(a)=2^{a_0-1}ですよね

で、その本における証明の骨子は、写像f:N^N→F(N) が全単射で対応しているということです。(F(N)={A⊆N|Aは有限})
感覚的には自明ですが、もう少し詳しく言うと、N^Nの元aに対して、単にaの像を考えてしまうと重複した値がキャンセルされてしまうので、
重複の無いようにaの値を”ずらして”F(N)の元に対応させる、という考えです:
つまり、f(a)={a[0]+0,a[0]+a[1]+1,a[0]+a[1]+a[2]+2,a[0]+a[1]+a[2]+a[3]+3,…Σ_{l=0}^{m-1}a[l]+(m-1),Σ_{l=0}^{m}a[l]+(m-1)} です。
(最後の項(要素)だけプラス1されていないのが注意点)

数日前からこのfの逆写像を具体的に定義しようと、A∈F(N)に対して|A|=0,1,2以上の場合でどうしたら良いのかあれこれ考えていました。

しかし、>>304さんの回答がクリアすぎますね。その説明を聞くだけで自明感ありありです。
実際の細部までの証明となるとしんどそうですが
0306132人目の素数さん
垢版 |
2019/01/17(木) 02:11:02.54ID:w+hJ6CW4
>>289 >>296

(x,y) = (-2.342794584464302552555148029, 1.109026655618377417435608178)
のとき
x^2 + 2xy + 2y^2 + 4x - 5y = -12.1641881908282504073292411817392892
0307132人目の素数さん
垢版 |
2019/01/17(木) 02:47:08.46ID:R3Er0Hp2
f(x)をn次多項式とする。

(1)任意の正整数mに対し、多項式f(x)*(1-x)^mの各項の係数がすべて整数ならば、f(x)の各項の係数はすべて整数であることを示せ。

(2)以下の命題が真となるような正整数kが存在するならば、それをnを用いて表せ。
「k以下の任意の正整数mに対し、f(x)*(1-x)^mの各項の係数がすべめ整数ならば、f(x)の各項の係数がすべて整数である。」
0308132人目の素数さん
垢版 |
2019/01/17(木) 04:33:21.27ID:7Y/wl+xk
>>307
g(x)は恒等的に0ではない整数係数多項式で、
g(x)の最高次の係数は1または−1とする。
f(x)は複素数係数多項式で、f(x)g(x)は整数係数多項式になるとする。
このとき、f(x)自体が整数係数多項式になることを示せ。
0310132人目の素数さん
垢版 |
2019/01/17(木) 09:29:52.21ID:ooa+yzJx
■正の整数nに対して

(1/4){1-n(n-1)(n-2)(n-3)(n-4)(n-5)(n-6)(n-7)(n-8)(n-9)(n-10)(n-11)(n-12)/13!}

出力は0≦n≦13の範囲で

1/4
1/4
1/4
1/4
1/4
1/4
1/4
1/4
1/4
1/4
1/4
1/4
1/4
0311132人目の素数さん
垢版 |
2019/01/17(木) 10:13:15.48ID:fWvrUb8c
すいません、計算方法を教えて欲しいです。

仰角固定で回転する場合に、回転角度に応じた真横から見た見た目の仰角の角度を計算したいです。

例えば仰角30度で回転角度ゼロの場合に真横から見て、見た目の仰角が30度とした場合に、
回転角度が90度に近づくにつれ、同じ位置からの見た目の角度も90度に近づきますが、
この場合にいくつかの固定の回転角度での見た目の仰角を計算したいのです。

説明が分かりにくかったら質問して頂けると助かります
0312132人目の素数さん
垢版 |
2019/01/17(木) 11:36:54.75ID:LNA270ZM
>>311
仰角θ、回転角ρ、見た目の仰角φだったら、横から見て(cosθ,sinθ)の位置の物体は回転によって(cosθcosρ,sinθ)の位置に移動する
よって、cotφ=cosθcosρ/sinθの関係式になる。これをφについて解けば良い

ってことで合ってます?
0314132人目の素数さん
垢版 |
2019/01/17(木) 17:08:44.21ID:R3Er0Hp2
nを3以上の整数とする。
0<q<p<nを満たすように整数p,qを動かすとき、方程式
x^n-x^p-x^q+1=0
が持つ実数解の個数の最大値を求めよ。
0315132人目の素数さん
垢版 |
2019/01/17(木) 17:13:06.40ID:w+hJ6CW4
>>287
第二種ルジャンドル関数は
 v(x) = (1/2)x・log((1+x)/(1-x)) -1

>>292
最後の行は
y = x g(x) = C_1 { -1 + (1/2)x log|(1+x)/(1-x)| } + C_2 x
0316132人目の素数さん
垢版 |
2019/01/17(木) 17:53:51.17ID:w+hJ6CW4
>>314
 x=1 はつねに解だが

n=3 … 2個
 x^3 -x^2 -x -1 = (x+1)(x-1)^2  2個(±1)

n=4 … 2個
 x^4 -x^2 -x +1 = (x^3+x^2-1)(x-1) 2個
 x^4 -x^3 -x +1 = (x^2+x+1)(x-1)^2 1個(1)
 x^4 -x^3 -x^2 +1 = (x^3-x-1)(x-1) 2個

n=5 … 3個
 x^5 -x^2 -x +1 = (x^3 +x-1)(x+1)(x-1) 3個
 x^5 -x^3 -x +1 = (x^4+x^3-1)(x-1)   3個
 x^5 -x^3 -x^2 +1 = (x^2+x+1)(x+1)(x-1)^2 2個(±1)
 x^5 -x^4 -x +1 = (x^2+1)(x+1)(x-1)^2 2個(±1)
 x^5 -x^4 -x^2 +1 = (x^4-x-1)(x-1)   3個
 x^5 -x^4 -x^3 +1 = (x^3-x^2-1)(x+1)(x-1) 3個
0317132人目の素数さん
垢版 |
2019/01/17(木) 18:36:16.79ID:kkyC/fOc
D_n は、

σ^n = e
τ^2 = e
τ * σ = σ^(-1) * τ

を満たす2つの元 σ, τ をもち、

D_n = {σ^i * τ^j | i ∈ {0, 1, …, n-1}, j ∈ {0, 1}}

と書けるような群とする。

D_n の中心は、 n が奇数ならば単位元のみより成り、 n が偶数ならば単位元以外の元を含むことを示せ。
0318132人目の素数さん
垢版 |
2019/01/17(木) 18:40:18.91ID:kkyC/fOc
D_n は、

σ^n = e
τ^2 = e
τ * σ = σ^(-1) * τ

を満たす2つの元 σ, τ をもち、

D_n = {σ^i * τ^j | i ∈ {0, 1, …, n-1}, j ∈ {0, 1}}

と書けるような位数 2*n の群とする。

D_n の中心は、 n が奇数ならば単位元のみより成り、 n が偶数ならば単位元以外の元を含むことを示せ。
0319132人目の素数さん
垢版 |
2019/01/17(木) 19:01:39.83ID:srr7Syej
数理論理学以外の分野ではなぜ自然数nにたいして n+1=n∪{n} が意識されないように議論されてるんですか?
0320132人目の素数さん
垢版 |
2019/01/17(木) 19:47:28.99ID:JyGsQA8q
そのように定義する必要がないから
0321311
垢版 |
2019/01/17(木) 23:56:45.35ID:fWvrUb8c
>>312
ありがとうございます
そしてすいません
cotって何ですか?
0322311
垢版 |
2019/01/18(金) 01:47:20.76ID:ZgTJXNbC
>>321
すいません
ググって解決しました

計算して実験してみます
ありがとうございました
0324132人目の素数さん
垢版 |
2019/01/18(金) 06:56:29.59ID:/RRmg93h
>>314
f(x)=x^n-x^p-x^q+1とおくと f(1)=0, f(0)=1
f(-1) は n,p,q の偶奇により -2,0,2,4 のいずれか

偶奇の全パターンで増減表を書いて調べると
n,p,q がすべて偶数のとき f(1)=f(-1)=0
ここで n≠p+q ならば f'(1)≠0, f'(-1)≠0
f(x) は x→-∞, x=0, x→∞ で正だから
f(x) は x=1, x=-1 の近傍でさらに零点をもつ

よって f(x) の実数解は最大で 4 個.
(例:x^8-x^4-x^2+1=0)
0325132人目の素数さん
垢版 |
2019/01/18(金) 11:22:34.07ID:/GZoaJ6g
おそらくここのレベルではすごく簡単ですみません…

ここでいいのかわからないけど質問します
趣味で小説書いてるんですけどそのネタで書きたいものがあって答えをそこに書いてもいいという奇特な方お願いします
ttps://dotup.org/uploda/dotup.org1750146.png
正三面体を横から見たときに見える辺なんですけどこちら普通の辺を1pとしたとき
どのくらいの長さなんでしょうか
高さhとして何やら導くような気がするのですが…理系だったのにもうわけわからない
0326132人目の素数さん
垢版 |
2019/01/18(金) 11:26:33.58ID:/GZoaJ6g
あ、すいません
上図のように作図したいのが目的なんですけど
きれいに星形にするには上下反転した後どのくらいずらすのが良いのかもご回答いただけるとありがたいです

あと、1cmと書きましたけど分かりやすくするためなのでxとかでもいいですし2pとかでもいいです
0329132人目の素数さん
垢版 |
2019/01/18(金) 13:15:07.52ID:JgVx1B29
星形多面体をググったほうが早そう
0330132人目の素数さん
垢版 |
2019/01/18(金) 13:40:21.87ID:/GZoaJ6g
>>327-329
わーありがとう!
正四面体なのか。
すばらしいです。やっと星形になったよー!名称も助かりました。
どの角度からも星形かと思ってたけど違うのね
こんなに早く回答もらえると思わなかった
あざす!!がんばりやす
0331132人目の素数さん
垢版 |
2019/01/18(金) 14:25:21.00ID:nVZ4g14z
>>324
ありがとうございます、f(-1)の偶奇で場合分けするとは考えつきませんでした。答えは無限個ではないと聞いていましたが、意外と少ないです
0333132人目の素数さん
垢版 |
2019/01/18(金) 17:09:02.93ID:Bfs/XC23
東大出版の線形代数入門を読んでて分からないところが出てきてしまいました。
第4章線形空間の§3.基底および次元、定理[3.10]の証明のところで、「n=m-1と仮定すると、ふたたび帰納法の仮定により、F'はS'の極大線形独立系となるから」とされているのですが、
F'のベクトルの線形結合でS'の任意のベクトルが表されるということには触れられていません。
これはf_1,f_2,・・・,f_m-1という線形独立なベクトルの線形結合で表わされないベクトルがS'に存在するとすると、そのようなベクトルをfと置いたとき、
f,f_1,f_2,・・・,f_m-1が線形独立となり、帰納法の仮定である"n個より多くのSのベクトルは線形従属となる"ことに矛盾するから、F'がS'の極大線形独立系となる、という解釈でよろしいのでしょうか。スレチであれば申し訳ありません。
0334132人目の素数さん
垢版 |
2019/01/19(土) 01:42:33.97ID:teKbRMU/
>>333
その解釈で問題ありません
ちなみに証明の本筋には関係ありませんが、テキストの証明ではnを固定してSの濃度kに関する帰納法を用いているようなので、k=1ではなくk=nから始めるべきですね

専門書でも証明にギャップがあったり誤りが含まれていることは多々あります
行間を埋めるのも勉強だと思って受け入れましょう
0335132人目の素数さん
垢版 |
2019/01/19(土) 07:26:59.70ID:4F1RB9EE
「四捨五入して上から2桁の概数にしましょう。」

という問題で、

1.852をやると、1.9だが、

0.852をやると、0.9かと思ったら、答えは0.85だった。

小学生の教科書にどこにも書いてないから、ネットで調べたら
「上から2桁は0は含めないで、初めて出てきた整数から数える」と教えてもらって初めて理解した。

なんで小学生の教科書に、その大事な決まりごとを書かないのか理解できない。

「上から2桁は0は含めないで、初めて出てきた整数から数える」という事は、
生まれた時点で誰から数えられることなく、本能的に知ってなきゃダメなんですか?

なんで教科書にその前提を書かない?

答えろや!!
0336132人目の素数さん
垢版 |
2019/01/19(土) 07:43:41.18ID:4F1RB9EE
いいや、わかってるぞ。
なぜ「上から2桁の概数は、0は含まず最初に出てきた整数から数える」
という大事な決まりごとを、あえて小学生の教科書に書かない理由が。
作り手がわざと、小学生の段階から数学につまづいてもらおう、苦手になってもらおうという戦略だ。
数学が苦手な人間が増えれば、相対的に理数系の人間の価値が高まる。
作り手は、「この決まりごとはあえて書かない。理数系の人間なら説明不要。この問題がノーヒントで分からないやつは、所詮素質がない(笑)」と
小学生たちを振るいにかけるため、わざと教科書を作成している。
いい加減にしとけよ。
0337132人目の素数さん
垢版 |
2019/01/19(土) 07:47:46.39ID:4F1RB9EE
そういう教科書づくりをして、今まで日本の何百万人の子供たちが、数学嫌いになったかわかるか?
わかるまい!
0340132人目の素数さん
垢版 |
2019/01/19(土) 08:17:46.35ID:4F1RB9EE
>>339

>書いてないはずないと思うんですけど

その通り。私も書いてないはずがないと、目を皿のようにしてみましたが、
しかし見つかりませんでした。

「わかるやつだけがついて来い。分からないやつは捨て置く」
という「選民思想」に基づいた教科書づくりなんですよ。
0344132人目の素数さん
垢版 |
2019/01/19(土) 08:25:33.07ID:4F1RB9EE
私は書いてない、と主張している。
書いてないものをどうアップするというのでしょうか?
0345132人目の素数さん
垢版 |
2019/01/19(土) 08:26:14.43ID:uBbXZkfZ
表紙だけとか概数の説明が書いてあるページ貼ればいいだけですよね

貼れないということは持ってないということですね
0346132人目の素数さん
垢版 |
2019/01/19(土) 08:36:12.62ID:4F1RB9EE
いや、意味ないでしょう。
私が意図的に、当該解説ページを外してアップしたらどうしますか?
0348132人目の素数さん
垢版 |
2019/01/19(土) 08:37:26.50ID:uBbXZkfZ
それでもいいですよ

あなたが妄想で物を語っているのではないことが証明されますね

私はあなたが教科書を読まずに文句を言ってるのだと思ってますから
0349132人目の素数さん
垢版 |
2019/01/19(土) 08:40:09.31ID:4F1RB9EE
>>347
そうですよ。
小5の小数のわり算。
概数自体は小4でならいますがね。
いずれも書いてありません。
0350132人目の素数さん
垢版 |
2019/01/19(土) 09:12:22.17ID:lyGBZ0Hj
とりあえずその問題とその問題が書かれている章をアップすればいいだろう
それもしないと妄想と言われてもしょうがないんじゃ?
0353132人目の素数さん
垢版 |
2019/01/19(土) 10:35:57.17ID:4F1RB9EE
教科書の作り手は「選民思想」ではなく「数学が苦手な子もわかる」教科書づくりに励んでもらいたい。
0354132人目の素数さん
垢版 |
2019/01/19(土) 10:44:29.57ID:uBbXZkfZ
でも、あなた教科書読んでないですよね?
なぜ読んでないのにそんなこと言えるんですか?
0355132人目の素数さん
垢版 |
2019/01/19(土) 10:52:01.36ID:4F1RB9EE
読んでますよ。
読んでないのに腹を立てて、このような場所に書き込むわけないでしょう。
0357132人目の素数さん
垢版 |
2019/01/19(土) 11:09:07.17ID:4F1RB9EE
むしろ「上から二桁の概数を求める際には、0は含まず最初の整数から数える」
という部分が記載されている教科書があればアップしてもらいたいぐらいです。
しかし、
教科書は著作物なので勝手にアップは違法なのでは。
出版社名を明かすのも、出版社にご迷惑がかかる。
私がアップを拒む理由はそれもあります。

ゆえに、先程からアップをあおるお方の「遵法意識」はどのようなものか、疑わざるを得ない。
0359132人目の素数さん
垢版 |
2019/01/19(土) 11:30:00.76ID:4F1RB9EE
左様でございます。
そもそも此度、教科書に書いてあるか、ないかを証明する事が目的で書き込みをしたわけではござらん。
ずさんな解説が横行している昨今の教科書を憂い、義憤にかられて糾弾したまで。
0361132人目の素数さん
垢版 |
2019/01/19(土) 11:50:26.35ID:uBbXZkfZ
藁人形論法という奴ですね

自分で仮想敵を作り上げて、糾弾する

まずはちゃんと調べることから始めましょうよ
0362132人目の素数さん
垢版 |
2019/01/19(土) 13:51:10.22ID:LEg0zy7I
>>335
概数を有効数字の意味で使ってるよな。
1000,100,10,1,0.1,0.01,0.001,0.0001どれも1桁の概数w
0363132人目の素数さん
垢版 |
2019/01/19(土) 14:17:51.31ID:x3ursV4d
教科書を読まん奴が教科書を馬鹿にする
改めて読んでみたら感心したと言う人しか会ったことがない
0364132人目の素数さん
垢版 |
2019/01/19(土) 15:35:45.65ID:8oM1lSXz
予想通り、始めに結論ありきだよなw

「教科書が悪い!教師が悪い!」っていいたいだけ
それをいうために都合のいい二桁の概数とかいうのを
ひっぱりだしてるだけ

悪いのはオメ―の頭だっつうのw
0365132人目の素数さん
垢版 |
2019/01/19(土) 16:52:36.63ID:H/XLPvYP
「とどきませんでした。」と聞こえてきました。何度も聞かされて、非常に不愉快なので
もうその意味不明を聞かせるのを止めろ
0366132人目の素数さん
垢版 |
2019/01/19(土) 17:44:42.92ID:WRHkPuIU
M.リードの初等代数幾何講義に、
「幾何学的環A=k[V]に対しては、スペクトルSpecAは多様体Vと丁度同じだけの情報を含んでいて、かつ、それ以上の情報は含んでいないことを理解しておくことは大切である。」
と書かれています。
(ここで、Vはアフィン代数多様体、k[V]はVの座標環、幾何学的環とは有限生成k代数で整域なもの)
これは、幾何学的環のスペクトラムから幾何学的環(またはアフィン代数多様体)を構成する関手が圏同値になるということだと思ったのですが、この理解は正しいでしょうか?
また、正しいならどのように構成できるか教えてください。
0367132人目の素数さん
垢版 |
2019/01/19(土) 18:31:27.74ID:HiRUTxEy
すいません>>325です
正四面体は理解できたのですが
正四面体を水平に置いたときに視覚的に見える長さって計算で出せるのでしょうか

何度もすみません
0368132人目の素数さん
垢版 |
2019/01/19(土) 18:33:00.31ID:HiRUTxEy
なんというか…地面についている三角形についている三角形は奥に向かっている…ので
視覚的には小さく見える気がするんですけど
それを数値化できるのかなって
0369132人目の素数さん
垢版 |
2019/01/19(土) 21:51:17.60ID:teKbRMU/
>>366
手元にリードの本がないので推測になりますが、アフィン代数多様体の定義は「多項式系の零点集合として定まるアフィン空間の部分集合」程度で考えているものと思いますので、その前提で書きます
まずSpecについて述べます
通常、環Aに対しSpec(A)は「Aの素イデアル全体の集合にある位相と構造層を合わせた局所環付き空間」として定義されます
Spec(A)の形で表される局所環付き空間をアフィンスキームと呼びます
局所環付き空間としての射を考えることにより、アフィンスキームのなす圏が考えられます
Spec(A)はこの圏の対象と考えるのが自然であると思います
実は、アフィンスキームのなす圏は環のなす圏と(反変)圏同値です
(証明は難しくない)
よって、アフィン代数多様体のなす圏は、アフィンスキームのなす圏の充満部分圏となります
なお、X=Spec(k[V])に対し
V……Xの中で閉点(極大イデアル)のなす部分集合
k[V]……構造層の大域切断のなす環
を取ることによりVやk[V]が得られます
代数多様体のなす圏はスキームのなす圏の充満部分圏である為、初めから代数多様体を特別なクラスのスキームとして定義するのが現代的です

もし疑問が残っている場合や分からない用語があれば追加で質問するか、スキーム論の解説書(Hartshorneの第2章など)を読むと良いと思います
0370132人目の素数さん
垢版 |
2019/01/20(日) 01:52:44.29ID:ShQpNSzK
>>369
ありがとうございます。
Specと言うだけで位相と構造層まで込めて考えるのですね。Reidの本にはSpecに位相と構造層を込めてアフィンスキームと呼ぶと書いてあったので、構造層は考えなくてももしかしたらいけるんじゃないかと思ったりしていました。少し考えれば普通に無理ですね。

先の質問とは少し離れるのですが、アフィン代数多様体の圏と圏同値になるようなスキームの圏の部分圏の、スキームの性質としての特徴付けはありますか?
0372132人目の素数さん
垢版 |
2019/01/20(日) 12:03:15.27ID:02AEpTUM
>>370
文献によって細かな定義は異なります
特に、可換環論の解説書や、代数幾何であってもスキーム論を避けたい状況では、SpecAを単なる集合と見なすことが多いかもしれません

アフィン代数多様体のスキームとしての特徴付けをかくと
integral affine scheme of finite type over k
(k上有限型の整アフィンスキーム)
となります
各性質の定義は割愛しますが、これは言い換えると単に
「k上有限生成な整域RによってSpecR
とかけるスキーム」
というのと同じことです
(これが質問に対する回答)

これは「対応V→k[V]がアフィン代数多様体のなす圏とk上有限生成な整域のなす圏の間の反変圏同値を与える」「対応A→SpecAが環のなす圏とアフィンスキームのなす圏の間の反変圏同値を与える」ことを認めればほとんど当たり前ですね

ちなみに、より一般に代数多様体のスキームとしての特徴付けは
integral separated scheme of finite type over k
(k上分離的かつ有限型である整スキーム)
となります
0373132人目の素数さん
垢版 |
2019/01/20(日) 12:04:42.72ID:02AEpTUM
>>371
この定理はアフィンスキームの特徴付けであって、アフィン代数多様体の特徴付けではありません
0374132人目の素数さん
垢版 |
2019/01/20(日) 14:02:25.79ID:b75/2Ym4
銀河群とD-加群はどっちの方が重要ですか?
0377132人目の素数さん
垢版 |
2019/01/20(日) 14:42:42.79ID:qioqZQhr
>>376
因数分解ですよね
それはわかるんですがその後です
ルート外して(x-2)−(x-3)これがなんで2x-5になるのかがわかりません
2<x<3が関わると思うんですが考え方が見当つかない
0378132人目の素数さん
垢版 |
2019/01/20(日) 14:43:57.53ID:F4+3Tnu3
>>377
その範囲の時、ルートを外してそうならないだろ
Aが負の数のときA^2の√ってAか?
0379132人目の素数さん
垢版 |
2019/01/20(日) 14:46:58.22ID:qioqZQhr
攻撃的な人はいらないのでもうレスしないでください
普通に教えてくれる方、途中式を教えてください
途中式見ながら自分で考えます
0380132人目の素数さん
垢版 |
2019/01/20(日) 14:51:26.45ID:H/9snw60
>>375
平方根を外したときの値は絶対値がつきます
絶対値の中身はxによって正負が変わるので
そこに気をつければいい
0384132人目の素数さん
垢版 |
2019/01/20(日) 19:23:36.82ID:EljfxviT
高校数学入門書を買ったのですが因数分解の”たすきがけ”にて

ax2-(a2+2a-1)x-a-2

ax2-(a2+2a-1)x-(a+2) にして"たすきがけ"するように書いてあるのですが
a-2が(a+2)になる理由が分かりません。

基礎が抜けてるかもしれませんが、答えでもヒントでもいいので教えてくれませんか?
0386132人目の素数さん
垢版 |
2019/01/20(日) 20:30:56.72ID:EljfxviT
>>385
凄い初歩に戻ると思うのですが
=(-1)*(a+2)の(a+2)に付いてるカッコはどうしてついているのでしょうか?

カッコのルールが理解できてないみたいなのでググったのですが上手く出てこなくて困ってます;
0387132人目の素数さん
垢版 |
2019/01/20(日) 21:03:51.44ID:1f4erZ8s
幾つかの数が幾つかの演算記号(足す+、引く-、掛ける*、割る/)で結びつけられているとき
計算の順序を示すためにカッコを使います。

5*6+3 は 掛算と足し算の計算順の定めから 答は33になります。
これに対して 5*(6+3) は ( )の中を最初に計算してから5をかけるので答は45になります。

(-1)*a+1 は -a+1 ですが
(-1)*(a+1) は 掛算と足し算の間の分配法則から (-1)*a+(-1)*1 =-a-1 になります。
0388132人目の素数さん
垢版 |
2019/01/20(日) 21:25:45.84ID:k2tsfbM3
http://i.imgur.com/RM4d3Va.jpg

この画像を数学的に説明するとどうなりますか?
0391132人目の素数さん
垢版 |
2019/01/20(日) 23:35:14.08ID:k0e/DM0q
>>335
おお、たしかにそなたの言う通り、小学生の教科書にはその重要な部分を説明する箇所がなかったな。

わしか?
わしは日本の小学生教科書を司るグランドマスターじゃ。
そなたの指摘、もっともじゃ。改善しようぞ。
しかと担当する行政機関に伝え、以後、小学生の教科書に「上から二桁の概数を求める場合は、0は含めず、初めての整数から数える」と注釈を入れるように働きかけてみようぞ。
そなたこそ、真の勇者じゃ。
そなたを誹謗中傷した奴は、ろくな死に方をしないであろう。
そなたが正しい。
心から礼を言う。
0392335
垢版 |
2019/01/20(日) 23:39:10.00ID:9R55F4BQ
>>391
お聞き届けいただきありがとうございます。
よろしくお願いいたします。
これで算数につまづく小学生も減る事でしょう。
0394132人目の素数さん
垢版 |
2019/01/21(月) 05:39:10.69ID:eRVhwv7g
確率の問題です。教えてください。

ボタンを押すと1から100までの数字のうち一つがランダムに表示される装置を使い、二人でゲームをします。

ルール
親役と子役に別れます。
子→親→子→親…という順番でボタンを押し、出た数字の十の位か一の位の好きな方(10が出れば1か0)を「30」から引いていき、最初に「0」以下にしてしまった方の負けです。
途中どちらかが1か100を出した場合は親の勝ちとなります。

このとき親の勝率は何%でしょうか。
また、親は序盤は小さな数字を選択するほうが有利だと思うのですが、
「30」がいくつ以下になるときは大きな数字を選択するほうが良いでしょうか。

よろしくおねがいします。
0395132人目の素数さん
垢版 |
2019/01/21(月) 07:11:44.41ID:s0Etp/8a
>>379
>378のレスが理解できない自分を恥じるべき
そして考えるヒントを答えてくれた人に謝辞を述べるべきだね。
0399132人目の素数さん
垢版 |
2019/01/21(月) 08:41:04.59ID:vUGyFC3R
>>398
そういうことか
それぞれ30から引くと思っていたので
> 「30」がいくつ以下になるときは大きな数字を選択するほうが良いでしょうか
これの意味がわからなかった

しかしそうなると確率の問題ではなくなっちゃわないかな?
確率の問題ということだったから常に小さい方を選択してそれぞれに30から引くのだと思ってたわ
0400132人目の素数さん
垢版 |
2019/01/21(月) 08:49:23.71ID:eRVhwv7g
>>398
>>399
すみません説明不足でした。
「30」は共有でそこから順番で数字を減らしていきます。

後半の意味は
例えば子から渡された数字が「13」の時に、
親が16を出した場合「12」か「7」どちらを選択するほうが勝率があがるのか
という意味です。
0401132人目の素数さん
垢版 |
2019/01/21(月) 08:56:04.67ID:vUGyFC3R
常に勝率が高くなる方を選ぶとしてもどの場面でどういう選択をすればよいのかをまず確定させないと確率計算出来ないだろう
相当大変なんじゃないか?
0402132人目の素数さん
垢版 |
2019/01/21(月) 09:05:25.51ID:s0Etp/8a
残り数字が6のときに15が出たらはランダムに1か5を選ぶ
75が出たらは5を選ぶようにすればシミュレーションできそう。
0403132人目の素数さん
垢版 |
2019/01/21(月) 09:36:12.79ID:TK+8KOsR
>>393
y ∈ A をピックアップして A' := { x ∈ A ; d(a,x) ≦ d(a, y) } を作れば 、A' は 有界閉集合
Euclid空間では 有界閉集合 ⇔ コンパクト集合
d(a,x) は連続写像で, 連続写像による コンパクト集合の像は コンパクト集合 ( = 有界閉集合 )
実数の有界閉集合は 下限値 ξ を持ち(∵下に有界) , それが最小値である (∵ 閉集合) .
f(A') の最小値 ξ は f(A) の最小値である.

後半は A (コンパクト集合 = 有界閉集合) をそのまま使って 像の下限値, 上限値 を取れば良い.
0404132人目の素数さん
垢版 |
2019/01/21(月) 09:48:40.61ID:35kZ+EFy
方程式2y^2+3x+4y+5=0の表す放物線の焦点の座標は(ア)であり、準線の方程式は(イ)である。すいませんわからないので助けてください
0405132人目の素数さん
垢版 |
2019/01/21(月) 10:03:31.32ID:TK+8KOsR
>>404
2y^2+3x+4y+5=0
⇔ 2(y + 1)^2 + 3 (x+1) = 0
⇔ x+1 = (-2/3) (y + 1)^2

放物線: x = (-2/3) y^2 の 焦点/準線 を求めて
(-1, -1) だけ平行移動すればよい.
0406132人目の素数さん
垢版 |
2019/01/21(月) 10:42:24.09ID:LT67NZkH
>>394
コンピュータ向きの問題よね

それぞれが自分の手番では勝つ確率の高い方の手を選択する前提で、
親と子のそれぞれの手番で、場の数字が1の場合の勝率から始めて、場の数字が30の場合までの確率を順番に求めるのがまっとうなやり方かな。
0407132人目の素数さん
垢版 |
2019/01/21(月) 15:03:05.17ID:LT67NZkH
>>394
数値計算をしてみたところ、「子の手番で場の数字=30」における親の勝率は約56.85% (※誤差あるかも。どなたか検証お願いします)
親の手番で親の勝率が極小になるのは、場の数字が1,14,25、極大になるのは場の数字が10,22のとき
子の戦略としては、まず場の数字を25に近づけ、次に14に近づけ、10を下回ったらなるべく減らすようにする
子の手番で子の勝率が極小になるのは、場の数字が1,14、極大になるのは場の数字が10,21のとき
親の戦略としては、まず場の数字を21に近づけないように進め、あとは同様に14に近づけ、10を下回ったらなるべく減らすようにする
いずれも序盤の確率変動は少ないので、20以下になったら勝負をかけるのでもよいのではないかと思います

興味深いのは、親の総取りルールを無くして、1が出たときに1を減らし、100が出たときは0か1かを選べるルールとすると、「子の手番で場の数字=30」における親の勝率が約50.00%となるところ。
このゲームでは、場の数字が十分大きければ、後手にハンデをつけなくても公平性は保てるのではないかと思われます
0408132人目の素数さん
垢版 |
2019/01/21(月) 17:00:27.19ID:eRVhwv7g
>>407
とても詳しい解説ありがとうございます。

「親は10を下回ったらなるべく減らすようにする」
例えば>>400の場合「7」を選択するほうが有利ということでしょうか。
0410132人目の素数さん
垢版 |
2019/01/21(月) 17:25:25.62ID:LT67NZkH
>>408
>例えば>>400の場合「7」を選択するほうが有利ということでしょうか。

そこはそう単純でもないです。
子の手番で12のときの親の勝率は52.50%ですが、7のときは43.72%と低くなります。
どの局面でも確実に有利なほうを選ぶには、確率の表を持ってないと厳しいかなと思います。
0411132人目の素数さん
垢版 |
2019/01/21(月) 17:32:06.38ID:iPiFil5w
30での勝率が出てるということは「親もしくは子に1〜29が回ってきた場合の親の勝率」も計算されてると思うので、それを用いれば微妙な状況での最善手も分かりますね
というか最善手が分かっていなければそもそも勝率は出せてないはず
自分も計算しようと少し考えてたのですが2〜8あたりの計算が大変だと思ってやめてしまいました
0412132人目の素数さん
垢版 |
2019/01/21(月) 17:35:12.32ID:eRVhwv7g
>>410
ありがとうございます。
もしご迷惑でなければ確率の表を教えていただけませんか。
0413132人目の素数さん
垢版 |
2019/01/21(月) 18:02:08.05ID:ShFojwfW
nを奇数、n=2t+1として、


a(t)=Σ(p=0→t) C(n-p,p)*2^p

この級数の一般項は多分、a(t+1)=4a(t)+1となっている
(数値を当てはめて調べて成立するのでおそらく)ようなのですが、
このことをどうやったら簡潔に示せますか?


nが偶数、n=2sの時の
b(s)=Σ(p=0→s) C(n-p,p)*2^p

こちらはb(s+1)=4*b(s)-1を満たしているようなのですが、できればこちらもお願いします。
0414132人目の素数さん
垢版 |
2019/01/21(月) 18:46:01.69ID:wPIq5atL
>>412
データ出しておきます。計算間違ってたらご容赦。

場の数字 1   2    3    4    5    6    7    8    9    10   11   12   13   14   15   16   17   18   19   20   21   22   23   24   25   26   27   28   29   30
先手(子) 8.08% 22.17% 33.52% 41.99% 48.28% 52.93% 56.28% 58.61% 60.08% 60.79% 52.86% 47.50% 44.57% 43.48% 43.64% 44.30% 45.04% 45.69% 46.17% 46.44% 46.49% 46.29% 45.70% 44.94% 44.26% 43.75% 43.42% 43.24% 43.17% 43.15%
後手(親) 10.27% 24.77% 36.57% 45.48% 52.22% 57.26% 60.98% 63.59% 65.25% 66.06% 58.55% 53.66% 51.30% 50.88% 51.63% 52.79% 53.96% 54.96% 55.70% 56.16% 56.31% 56.33% 56.17% 55.87% 55.68% 55.73% 55.97% 56.31% 56.67% 56.99%

http://i.imgur.com/wlXuG2H.png

>>411
0の手が打てるところが計算しにくいですね。
最善手が分かっていなければそもそも勝率は出せないですが、0の手が打てることで、
勝率を出すには最善手が分かっていなければならないというジレンマに陥ります。

1と2を手で計算したところであきらめて、最終的には計算機を使いましたが、
アルゴリズムが間違っている可能性もあります。
0415132人目の素数さん
垢版 |
2019/01/21(月) 21:06:57.28ID:LQwvPK0m
f(x)=i|x| (iは虚数単位)
L^2([-π,π])
とした時
f∈L^2([-π,π])
を示せ

教えてください
0416132人目の素数さん
垢版 |
2019/01/21(月) 21:25:37.70ID:5/4ByZoN
すげー低レベルな質問していい?
四捨五入して()内の位を概数にしなさい
8730(百の位)
これの答えって9000だよな?
0417132人目の素数さん
垢版 |
2019/01/21(月) 22:22:02.09ID:bB6fQa5A
>>416
百の位で四捨五入しろという意味と取ったということだと
常に百の位は0になってしまうから
「百の位を概数にしなさい」という文章の意味が全く無くなってしまう

これは、8730は大体、何百くらいですか?という意味で
8700
になる
0418132人目の素数さん
垢版 |
2019/01/21(月) 22:24:15.92ID:bB6fQa5A
>>415
問題文が変
0419132人目の素数さん
垢版 |
2019/01/21(月) 23:25:12.89ID:iPiFil5w
>>414
p[n]を「nが親に回ってきた場合の親の勝率」
q[n]を「nが子に回ってきた場合の親の勝率」
とおく
(親子が互いに「相手が最善手を選択する」という想定で最善手を選ぶものとする)
p[1]~p[n-1],q[1]~q[n-1]を既知とし、p[n]とq[n]を求めることを考える
p[n]及びq[n]は、ランダムに与えられる数字100通りに対する行動の選択を確定することで計算ができるようになる
∀k≦0 p[k]=q[k]=0 としておく

計算の手順は
・0を候補に含まない数(10,20,……,90を除く91通り)が選出された場合の最善手はそれまでの計算結果から決まる
((1,100,一桁を除く)10k+lに対してはq[n-k]及びq[n-l]のうちより大きい方が選ばれるような選択をする)
・この時点での計算結果により、p_n及びq_nのとりうる値の範囲が絞れる
・この範囲を用いると10,20,……,90のうち最善手が決まるものがあるので、それにより計算が進んでp_n,q_nの範囲をさらに狭められる
・これを全ての手が確定するまで繰り返す

といった手順を考えておりました
しかし、0が絡んだところの処理がこれだけで上手くいくのか心配で、またコードを書くのも面倒だったので放置してしました
0420132人目の素数さん
垢版 |
2019/01/21(月) 23:41:52.30ID:LT67NZkH
>>419
考え方はそのような感じになると思います。なお、その定義の場合、
∀k≦0 p[k]=1,q[k]=0 ということになります。
0421132人目の素数さん
垢版 |
2019/01/21(月) 23:44:03.31ID:5XDj55BD
>>413

c(n) = Σ(p=0→[n/2]) C(n-p,p)・μ^p
とおくと
c(n+1) - c(n) -μ・c(n-1) = 0,
c(0) = c(1) = 1,
特性多項式 tt-t-μ の根(特性値)を α, β とおくと一般項は
c(n) = (β^{n+1} - α^{n+1})/(β-α)
= (2^{n+1} - (-1)^{n+1}) /{2 - (-1)}
= (2^{n+1} - (-1)^{n+1}) /3,

∵ μ=2 より α=-1, β=2.
0423132人目の素数さん
垢版 |
2019/01/22(火) 05:19:25.30ID:rCrBfS+J
aを負の整数とし、数列{a[n]}を
a[0]=a
a[n+1]=p*a[n]+1
により定める。

(1)pを整数とする。ある自然数kが存在してa[k]=0となるようなpを決定せよ。

(2)(1)において、kを最大にするpを求めよ。
0425132人目の素数さん
垢版 |
2019/01/22(火) 11:02:26.69ID:HdzIXYyZ
μにいろいろな数字を入れても成り立つので感動しました。
でも頭が悪くて分からないので証明が知りたいです…
0426132人目の素数さん
垢版 |
2019/01/22(火) 11:06:18.55ID:HdzIXYyZ
あ、理解できました!
Σをずらしてμの次数を揃えるだけですね……

やっぱりここで聞くと鮮やかに解決するなあ。ありがとうございますm(_ _)m
0427132人目の素数さん
垢版 |
2019/01/22(火) 13:12:14.66ID:iWbOX1m1
384 53760 8755200 1805690880 471092428800 153043438141440 


384との関連を式にするとどうなりますか?
0429132人目の素数さん
垢版 |
2019/01/22(火) 17:39:17.91ID:Eu/ejJJl
コーシーの積分定理の範囲なんですけど次の曲線Cに沿ってf(z)の積分を計算しなさい
f(z)=z +1、C:z(t)=t +it^2(0≦t≦1)
お願いします
0430132人目の素数さん
垢版 |
2019/01/22(火) 17:41:04.83ID:Eu/ejJJl
コーシーの積分定理の範囲なんですけど次の曲線Cに沿ってf(z)の積分を計算しなさい
f(z)=z +1、C:z(t)=t +it^2(0≦t≦1)
お願いします
0436132人目の素数さん
垢版 |
2019/01/22(火) 21:57:37.06ID:BehDmfKO
>>419
>p[n]を「nが親に回ってきた場合の親の勝率」
>q[n]を「nが子に回ってきた場合の親の勝率」

計算手順の考え方として、以下のように考える
* 各nについて、p[n-9]〜p[n-1]は既知で、それらの順序もわかっているが、さらに
 p[n]を含めたp[n-9]〜p[n]の10個におけるp[n]の順位は、同率である場合も含め、1位から10位までのいずれかである。
* qについても同様のことが言えるので、p[n]とq[n]の順位について、高々10×10=100パターンを考慮するだけでよい。
* それぞれのパターンで計算して求めたp[n]とq[n]の値が、仮定したp[n]とq[n]の順位と合致するパターンは
 各nについてそれぞれ1通りのみ。(のはず)

このように考えると、高々30×10×10=3000回のループを回せばn=30のときの値を求めることが可能で、以下となる。

q[30]=

1,255,840,831,286,410,344,553,190,947,158,076,917,732,266,749,165,730,083,853,325,236,106,658,980,529,840,407,925,066,436,903,263,853
──────────────────────────────────────────────────────────────────────────
2,209,088,445,315,697,057,898,270,227,580,214,828,393,999,382,068,722,895,295,542,490,153,350,964,453,188,894,720,000,000,000,000,000

≒56.85%
0437132人目の素数さん
垢版 |
2019/01/22(火) 22:35:06.53ID:udk+E6BF
この問題分からないので教えていただけると幸いです。

The following data are given for an optimum rocket:
Average molecular mass 24 kg/kg-mol
Chamber pressure 2.533 MPa
External pressure 0.090 MPa
Chamber temperature 2900 K
Throat area 0.00050 m^2
Specific heat ratio 1.30
Determine (a)throat velocity; (b) specific volume at throat (c) propellant
flow and specific impulse; (d) thrust; (e) Mach number at throat.
0438132人目の素数さん
垢版 |
2019/01/22(火) 22:48:18.74ID:V93kFM0k
関数f(x,y)=log(1+x+y)を点(x,y)=(0,0)のまわりでテイラー展開せよ。
ただし計算するのは2次近似の範囲を考えるものとする。
また、(x,y)の範囲は点(0,0)に十分近い範囲を考えるものとする。
0439132人目の素数さん
垢版 |
2019/01/22(火) 23:55:53.03ID:V93kFM0k
もうええわ
0440132人目の素数さん
垢版 |
2019/01/23(水) 00:48:13.99ID:cnf0LU1Z
>>436
そのやり方も思いつきましたが、あえて避けてました
理由は、p[n]とq[n]が確定した値をもつと仮定してその値を求めることに少し違和感があったからです
際どい数値設定をすると、複数の解が存在してしまい互いに相手の選択を知らないと勝率が確定しないという状況があるのでは、という懸念もありました

改めて考えてみたところ、今回の問題ではそのような事は起こらず、p[n],q[n]の解は常にちょうど1つ存在します
また、不等式評価の方法(>>419)も総当たりの方法(>>436)もどちらも正当化できます
参照する解の数N(今回であればp[n-1]~p[n-9]及びq[n-1]~q[n-9])と、p[n],q[n]の漸化式においてp[n],q[n]に依存する項の係数達C(今回であれば全て1/100)をパラメータとみなすとき、NとC次第でどちらのやり方がより効率よく計算できるかは変わってきます
ざっくり書くと、
Cが小さく、Nが大きいほど不等式評価の方が効率が良くなる
ことが分かります
0442132人目の素数さん
垢版 |
2019/01/23(水) 02:25:36.92ID:0KQkAS3a
金融工学は錬金術の一つ。

灰吹き法
 金・銀を鉛(or 亜鉛、水銀など)と共に熔融したのち骨灰ルツボ中で加熱すると鉛や不純物はルツボと反応して無くなり、金・銀の玉が残る。
0443132人目の素数さん
垢版 |
2019/01/23(水) 02:28:43.83ID:0KQkAS3a
>>423 >>434

p=0 のとき
 a[k] = 1   (k≧1)

p=-1 のとき
 a[k] = a ≦ -1 (k:偶数)
 a[k] = 1-a ≧ 2 (k:奇数)

|p|≧2 のとき
 a[k] = (p^k)a + (p^k - 1)/(p-1),
 a≦-1 より a[k] ≠ 0

∴ p=1 のみ。
 a[k] = a + k,
 k = -a のとき a[k] = 0,

>>429 >>430

∫_C (z+1)dz = [ (1/2)z^2 + z ](z:0→1+i) = 1+2i
途中の経路Cには依らない。
ポテンシャルが存在する。
0444132人目の素数さん
垢版 |
2019/01/23(水) 03:26:01.22ID:tmTJz0zL
>>440
「不等式評価の方法(>>419)」って具体的にはどうするの?
範囲を絞るために上限と下限を求めようと思ったら、結局は総当たりになるんじゃない?
0445132人目の素数さん
垢版 |
2019/01/23(水) 07:04:57.43ID:cnf0LU1Z
>>444
例として
p=1/3+1/2•max(1/3,q)
q=1/6+2/7•min(2/3,p)
を考えると
0≦p≦1,0≦q≦1
→1/2≦p≦5/6,0≦q≦1
→1/2≦p≦5/6,13/42≦q≦5/14
→1/2≦p≦43/84,13/42≦q≦5/14
→1/2≦p≦43/84,13/42≦q≦46/147

ここまで計算するとp<2/3,q<1/3が分かるのでp,qに関する連立方程式が決まり解けます
今回の問題であればmaxやminの項数が増えますが、原理は変わりません
同じ計算の繰り返しなので、パソコンにやらせる場合は単純なコードで楽に計算できると思います
0446132人目の素数さん
垢版 |
2019/01/23(水) 07:38:17.95ID:tmTJz0zL
>>445
なるほど
その方法で単純な計算式に変えていくんですね。
計算機に詳しいひとならすぐできるかなと思います。
0447132人目の素数さん
垢版 |
2019/01/23(水) 11:59:48.00ID:f/bMhxOp
位相空間の族{X_λ,O_λ}_λ∈Λからなる直積空間をXとする.コンパクトでない空間(X_λ,O_λ)が無限個存在するとXのコンパクト集合は内点を持たないことを示せ.
これは対偶を使うのが筋ですか?
0452132人目の素数さん
垢版 |
2019/01/23(水) 19:16:20.61ID:QCyKDCPv
以下の場合においてD上の広義積分∫∫f(x,y)dxdyを求めて下さい
D={(x,y)∈R^2 : 0≦x,y}, f(x,y)=1/(1+(x+y)^a) (2<a)
D={(x,y)∈R^2 : 0≦x, a≦y≦b}, f(x,y)=e^(-xy) (0<a<b)
0453132人目の素数さん
垢版 |
2019/01/23(水) 19:56:49.75ID:9d1QtcK2
次の積分を求めよ.ただし積分路は,次に示す閉曲線とする.
∫c z^2/((z+2)(z*1)) dz
(1)原点を中心,半径√2の円周
(2)4点-1-i.-1+i.-3+i.-3-iを頂点とする正方形
(3)原点を中心,半径3の円周
よろしくお願いします.
0455132人目の素数さん
垢版 |
2019/01/24(木) 14:53:25.09ID:h9t1ye7h
グラフ理論のグラフって言葉を使い始めたのは誰ですか?
なんでわざわざ被るような言葉を選択したのかわからん
0456132人目の素数さん
垢版 |
2019/01/24(木) 16:57:48.31ID:TF8Pu5er
>>455
wikipediaに書いてたので気になるなら自分で調べてください

graphのどの意味と被ることを気にしているのか分かりませんが、至って自然な用語だと思います
0459132人目の素数さん
垢版 |
2019/01/24(木) 18:39:51.27ID:7LsMK2jS
∫{1−(X/a)^2}^(3/2)dX

はいくらになるでしょうか。
0463132人目の素数さん
垢版 |
2019/01/24(木) 20:12:44.24ID:UR20TsD6
>>459
∫ {1-(x/a)^2}^{3/2} dx
= a ∫ (cos t)^{3} d(sin t) = a ∫ (cos t)^{4} dt = (a/4) ∫ (1+cos2t)^{2} dt
= (a/4) ∫ ( 1+2cos2t + (1+cos4t)/2 ) dt
= (3a/8) asin t + (a/4) sin2t + (a/32) sin4t + C
= (3a/8) asin(x/a) + (a/4) sin(2 asin(x/a)) + (a/32) sin(4 asin(x/a)) + C
= …
sin(2 asin(x/a)) = Im{ ( √{1-(x/a)^2} + i(x/a) )^2 } = 2(x/a) √{1-(x/a)^2}
sin(4 asin(x/a)) = Im{ ( √{1-(x/a)^2} + i(x/a) )^4 } = 4(x/a) √{1-(x/a)^2}^{3} - 4(x/a)^3 √{1-(x/a)^2} = …
0464132人目の素数さん
垢版 |
2019/01/24(木) 20:14:53.63ID:4WqVldAj
無限数列{a_n}は等差数列である。
{a_n}の中からうまく2個の項を選び、それらを入れ替えると、元の数列は等比数列になるという。
このような{a_n}をすべて求めよ。
0466132人目の素数さん
垢版 |
2019/01/24(木) 21:00:45.36ID:rUAgr0ff
受験科目が少ないと見かけの偏差値が上がる
は正しいか?

科目の成績は独立した正規分布として5科目での偏差値と3科目での偏差値の関係をシミュレーションしてみたら
こんな感じになった。
http://i.imgur.com/hOhq2y8.png
0468132人目の素数さん
垢版 |
2019/01/24(木) 23:31:23.42ID:hO4hkx1y
https://i.imgur.com/7YnK3OU.jpg

この平行四辺形ABCDで、△APDと面積が等しい三角形はどれか?という問題なんだけど、答えだけ見ると△CPDと書いてあるのね。

これ、考えてもよくわからなくて。

解説載っていなくて、答えしか載っていないんだよ。
0469132人目の素数さん
垢版 |
2019/01/24(木) 23:51:36.02ID:ALPfhm18
>>468
Pは対角線BD上にあるの?
それならPDを底辺と見ると高さが同じになるから面積同じ
0470132人目の素数さん
垢版 |
2019/01/24(木) 23:52:31.07ID:+OxSWM4K
>>468
平行四辺形の性質として、対角線で切ったABDとCBDは面積が等しい。(2つの三角は合同)

これを例えばS(平方センチ)とする

BP:PD=APB:APD=BCP:PCD(高さが同じなら面積比は底辺の長さの比)なのはわかる?

BP:PDの長さの比を例えばa:bとすると、

APDの面積は、ABDの面積Sの、b/(a+b)倍。

PCDの面積は、CBDの面積Sの、b/(a+b)倍。

だから等しくなる。
0471132人目の素数さん
垢版 |
2019/01/25(金) 01:19:13.03ID:J73vuHKN
>>468
AD上に点Xを、CD上に点Yをとり、□ABCDと相似な平行四辺形□XPYDを作る。
ADからPへの高さをH1、CDからPへの高さをH2とおくと、
H1:H2=CD:AD
よってH1:H2=CD:AD=XP:YPだから、
△ADP:△CPD
=AD*H1:CD*H2
=AD*CD:CD*AD
=1:1
したがって△APD=△CPD
0472132人目の素数さん
垢版 |
2019/01/25(金) 05:26:47.77ID:ddzGN+6K
>>463
ありがとうございます。
助かりました。
0473132人目の素数さん
垢版 |
2019/01/25(金) 07:01:22.29ID:8MvFyZbI
バカなので教えて下さい...

トータル利益率の計算方法が正解かどうか教えて下さい。
   原価  売価  利益
商品A  200  258  22.48%
商品B  220  258  14.7%
商品B  200  258  22.48%

計   620  774  19.89%(3品のトータルの利益率を出すには商品 原価合計≒売価合計で良いのでしょうか?)

もし間違っていたら計算方法を教えて下さい。
よろしくお願いします。
0474132人目の素数さん
垢版 |
2019/01/25(金) 10:56:48.87ID:nVr9rURQ
>>469
>>470
>>471

みなさん、ありがとうございます。
Pは対角線BD上にあります。
0475132人目の素数さん
垢版 |
2019/01/25(金) 11:08:47.57ID:d79owZ7w
爆サイ.com > 甲信越版 > パチンコ全国 > 魚群3回連続でハズレるの遠隔?それともホルコン?

>>550

何を言っているのか理解できません。

マイナスからピッタリゼロまで出て止まる時って何の事?意味不明・・・・・・。

ストックなしみたいな感じとはどういう事。(昔の違法基盤ならストックで貯められたけどな)

確率とか関係ないとはカスのセリフだな。通常確率だけが確率と思ってる奴ほど馬鹿な奴はいない。
低確率や超低確率も高確率と超高確率で収束するから確率なんだよ。

どこの世界に均一に大当たりするパチンコ台で営業する店があるんだよ。確率とか騒いで馬鹿じゃねえの。



--------------------------------------------------------------------------------

[ 匿名さん ]
0476132人目の素数さん
垢版 |
2019/01/25(金) 11:13:08.31ID:fuJY0/lg
>>473
利益率というのは算数や学校数学の間は原価に対する利益の割合を言うらしく実務で扱う利益率とは違うらしい
利益率が利益÷売価であるのならそれで正しいと思う

≒は「ほとんど等しい」ことを意味する記号で割り算の記号÷とは別物だよ
0478132人目の素数さん
垢版 |
2019/01/25(金) 13:09:00.33ID:pFp2/sYN
>>476
ご回答頂きありがとうございました。
≒の使い方まで教えて頂き本当に有り難いです。
0480132人目の素数さん
垢版 |
2019/01/25(金) 21:24:32.99ID:ssbsGPcV
>>427
384=8!! 

53760=2(10!!)+12!!

8755200=50(12!!)+10(14!!)

1805690880=1199(14!!)+100(16!!)

471092428800=9640(16!!)+2000(18!!)

153043438141440=23724(18!!)+40000(20!!) 
0481132人目の素数さん
垢版 |
2019/01/25(金) 23:09:22.00ID:9JfAMiTD
大学数学を勉強したいのですが、まずは何から勉強しないと駄目なのでしょうか?
順番を教えてください。
0482sage
垢版 |
2019/01/25(金) 23:53:12.83ID:oK+RurMi
>>481
一例

線形代数→集合と位相→1変数の微積分→複素解析→代数学(群・環・体)→多様体
→多変数の微積(多様体上のストークスの定理)→測度論とルベーグ積分論→関数解析
→確率論→常微分方程式論→偏微分方程式論→微分幾何学→位相幾何学
→ガロア理論→類体論→代数幾何学→数論的代数幾何学
0483132人目の素数さん
垢版 |
2019/01/26(土) 00:32:19.53ID:AreR2OBr
しもつんです
フーリエ級数展開は数列になっていると思ったのですが、間違っていますか?
0485132人目の素数さん
垢版 |
2019/01/26(土) 02:30:02.47ID:Pjv0wqb5
>>482
回答ありがとうございます。

基本的に大学数学を勉強し始める場合、線型代数学から始めた方が良いのでしょうか?
0486132人目の素数さん
垢版 |
2019/01/26(土) 05:24:20.58ID:NwkhdSLN
NO.7217998 2019/01/25 21:39

うちの会社の給料がTポイントなんだが…

ふざけんな
[匿名さん]

#1 2019/01/25 23:03

笑いの発想がアスペ


[匿名さん]

#2 2019/01/25 23:11

私の会社なんて鯖ですよ。


[匿名さん]

#3 2019/01/26 00:17

いいなぁ


[匿名さん]

#4 2019/01/26 01:10

現物支給


[匿名さん]

#5 2019/01/26 02:23最新レス

お薬の時間ですよ。
0488132人目の素数さん
垢版 |
2019/01/26(土) 07:46:42.51ID:dvE1d/TE
あ、すみませんがage
0490132人目の素数さん
垢版 |
2019/01/26(土) 10:27:39.59ID:dvE1d/TE
ありがとうございます

大問2の(4)が分かりません…
そこまでは、
BC=6^(1/2)+2 ^(1/2)
三角形OBC=1/2
三角形EBO=[2+{2 ^(1/2)-6 ^(1/2)}a]/4
かな、と…
0491132人目の素数さん
垢版 |
2019/01/26(土) 10:52:37.77ID:dvE1d/TE
大問3は(3)から分かりません…
0493132人目の素数さん
垢版 |
2019/01/26(土) 11:23:47.60ID:2IBFbgv+
>>484
矩形波だとかノコギリ波などを表すサイン波の係数が
数列に見えたので聞いてみました

数列×sinになっているのではないかと思いまして…
0494132人目の素数さん
垢版 |
2019/01/26(土) 11:24:57.06ID:2IBFbgv+
>>492
形式的にセンターではないでしょうか
早く解けるかを見ているので面白さはないかと
0495132人目の素数さん
垢版 |
2019/01/26(土) 11:30:45.95ID:zo7FkCqI
明らかにセンターじゃない。
センターの形式もしらないゴミは話しかけるな。
0496132人目の素数さん
垢版 |
2019/01/26(土) 12:24:20.28ID:cD/k/NS2
>>493
数列とは、人間にとって理解しやすい関係性のあること、ではありません

自然数と実数の関数が数列です
一見してランダムに見えたとしても数列です
そういう意味ではフーリエ級数展開が数列になっているのは当たり前です
フーリエ級数展開はf(x)という関数から、sinとcosの計算anとbnを求める操作のことですね
0499132人目の素数さん
垢版 |
2019/01/26(土) 15:48:12.52ID:tjieLLWm
■速報■

数学の超難問「リーマン予想」を証明したと発表した、
英エディンバラ大名誉教授のマイケル・アティヤ氏が、
1月11日に亡くなった
論文は撤回され、「証明」は幻に終わった
0501132人目の素数さん
垢版 |
2019/01/26(土) 16:38:11.64ID:EdKZ3oot
このような感じで三角関数が組み合わさったグチャグチャの式を簡単にする方法で、確実に成功するものはありますか?

これにこの後sinθを掛けて積分させられるので、一番目のままの形だと確実に時間内に解けないで終了するのですが

これって必ずしも簡単に一番下の形への変形できなくないですか?

こういう風に簡潔な形にできる、必ず成功する方法ってあるんですか?


https://i.imgur.com/45deHRi.jpg
0502132人目の素数さん
垢版 |
2019/01/26(土) 16:39:03.93ID:EdKZ3oot
模範解答にはこういう変形が書いてあったのですが、なんか天下り的じゃない……?本当にこれ試験場でできるのかな?と思ってしまいまして
0505132人目の素数さん
垢版 |
2019/01/26(土) 17:10:54.56ID:PJeYIqOt
k^2={(r+10)-(h-10)}^2
=r^2-2r(h-20)+(h-20)^2

になるらしいんだが途中式がわからん

教えてエロい人
0507132人目の素数さん
垢版 |
2019/01/26(土) 17:33:37.52ID:H1SwuHDy
{(r+10)-(h-10)}^2
=(r-h+20)^2
h-20=sとでもおけば
(r-s)^2=r^2-2rs+s~2
=r^2-2r(h-20)+(h-20)^2
てゆうか、わざわざsなんて置き換えなくもできる
0508132人目の素数さん
垢版 |
2019/01/26(土) 17:51:50.47ID:PJeYIqOt
>>507
ありがとう
0509132人目の素数さん
垢版 |
2019/01/26(土) 18:02:39.57ID:Xiy/6Hx6
xyz空間の原点を中心とする半径1の球面Bについて、以下の問に答えよ。

(1)B上を点(a,b,c)が動くとき、積abcの取りうる値の範囲を示し、その最大値を取る点が何箇所あるか述べよ。答えのみで良い。

(2)(1)において、ab+bc+caの取りうる値の範囲についてはどうか。答えだけでなく求める過程も記述せよ。

東進の模試の問題ですが(1)から分かりません。
0511132人目の素数さん
垢版 |
2019/01/26(土) 18:35:35.02ID:ZMhfEdtq
>>501>>502

ふつーに教科書の傍用問題集に載ってるゴミ問題の式変形と同じ

それすらわかってないウスラバカが調子こいて東大の過去問やっても無意味
0512132人目の素数さん
垢版 |
2019/01/26(土) 18:40:23.01ID:EdKZ3oot
>>509
とりあえず(1)だけ解いてみました
間違っていたらすみません

xx+yy+zz=1の時、xyzの取りうる範囲を聞かれてるのに等しい

(1)
球面はxyyzzx平面に対して対称だから、xyzが全て0以上のときだけ考えれば、xyzが動く範囲を考えるのに足りる。

xを定数t(0≦t≦1)とおいてyy+zz=1-tt
z=√(1-tt-yy)と書けるから

xyzの値はyの関数として(0y≦√1-tt)
f(y)=t*y*√1-tt-yyと書ける
yで微分して、

f'(y)/t=√1-tt-yy - yy/√1-tt-yy

これは0≦yで明らかに単調減少で、y=√1/2*√(1-tt)で極値を取る

よってf(y)の最大値はt*(1-tt)/2

これをtの関数g(t)と見てtで微分すると、2g'(t)=1-3t^2

よってg(t)はt=√3/3で最大値を取る。

この時g(t)=√3/9でこれがxyzのとりうる最大。
最小値はxyz全て正という条件から0以上でこれは実際にある。t=0,1の時はg(t)=0だし

あとは対称性より最小値は最大値の-1倍になるから、abcのとりうる範囲は-√3/9≦abc≦√3/9
0513132人目の素数さん
垢版 |
2019/01/26(土) 18:45:27.87ID:EdKZ3oot
g(t)=√3/9を満たすtは0≦t≦1にただ一つしかないから、xyzが非負の時
abc=√3/9を満たす点は点P(1/√3,1/√3,1/√3)ただひとつ。
よってそれを満たす球面上の点はxy,yz,zxのいずれかについてPを何回か反転した点にしかなく、そのうちabcが正になるものは4箇所。よって4箇所のみ。
0515132人目の素数さん
垢版 |
2019/01/26(土) 18:48:06.96ID:ZMhfEdtq
>>509>>512
1=a^2+b^2+c^2≧3(abc)^(2/3)より-√3/9≦abc≦√3/9
相加相乗平均より1行でおわるウルトラバカ問題
0516132人目の素数さん
垢版 |
2019/01/26(土) 18:50:13.43ID:EdKZ3oot
推敲せずに送信したら最小値が〜とか日本語がおかしい…
ある点で座標の積がαになるなら、面xyに対して対称な点を取ればその点の座標の積は-αになるっていう説明が抜けてました
0519132人目の素数さん
垢版 |
2019/01/26(土) 19:23:51.85ID:EdKZ3oot
>>509
(2)も一応書いておきます😅

(a+b+c)^2=aa+bb+cc+2(ab+bc+ca)だから、a+b+cの動く範囲を考えれば足りる

なので球面と平面x+y+z=k(kは実数)が共有点をもつかを考えれば良い。

平面x+y+z-k=0の法線ベクトルは(1,1,1)だから、球面がそのような平面と接するのは(1/√3,1/√3,1/√3)と(-1/√3,-1/√3,-1/√3)で、ここがx+y+zが最大と最小になる点
その間の数値では平面は球面を輪切りにするので、x+y+zはその間の全ての数値をとりうる

だから-√3≦x+y+z≦√3

だからu=xy+yz+zxの動く範囲は-1/2≦u≦1 でしょうか
0520132人目の素数さん
垢版 |
2019/01/26(土) 22:05:18.50ID:Xiy/6Hx6
2次関数f(x)とg(x)があり、f(x)はx=1で最大値1をとり、g(x)はx=0で最大値1をとる。
また、関数h(x)=f(x)g(x)はx=-1で最小値をとるという。
f,gが満たすべき条件を述べよ。
0524Lagrange
垢版 |
2019/01/27(日) 23:07:40.61ID:NqmDnyZc
>>521
(-17n^4 +200n^3 -793n^2 +1258n -630)/6 ぢゃね?
0530132人目の素数さん
垢版 |
2019/01/28(月) 14:59:43.27ID:Xm31HNMj
数列a_n=((2+n)(5+n))/(4((5-n)!))を既約分数で表す
a_1=3/16
a_2=7/6
a_3=5/1
a_4=27/2
a_5=35/2

これの分子が>>521
0531132人目の素数さん
垢版 |
2019/01/28(月) 15:51:15.05ID:P8A4soJo
■ある係数αを探しています

1152=3(8!!)

26880=7(10!!)

230400=5(12!!)

17418240=27(14!!)

361267200=35(16!!)

2043740160=11(18!!)

1705594060800=17x27(20!!)

5313724416000=5x13(22!!)

二重階乗の左側にある数値を数列にした時の
関数を探しています

数列3, 7, 5, 27, 35, 11, 17x27, 5x13を表す関数はありますか?
0532132人目の素数さん
垢版 |
2019/01/28(月) 17:51:52.80ID:Qbpe4QLP
m,nを正の整数とするとき、
a^3+b^3=2^m*3^n
を満たす正の整数の組(a,b)をすべて決定せよ。
0533132人目の素数さん
垢版 |
2019/01/29(火) 00:28:43.95ID:ld57s0vO
nを正整数とする。
曲線y=x^nの接線で、ある整数mを用いてy=mx+mの形で表されるものは存在しないことを示せ。
0534132人目の素数さん
垢版 |
2019/01/29(火) 01:36:44.81ID:ld57s0vO
素数p,q,rについての以下の方程式に解が存在すると仮定する。
p^q+q^r+r^p=(pq)^r

(1)この仮定のもとで、rの最大値を求めよ。

(2)この仮定が正しいかどうか述べよ。考察の過程を記述すること。
0535132人目の素数さん
垢版 |
2019/01/29(火) 03:40:52.45ID:LI2OtV3O
>>532

 (a, b) = (2^k・3^L, 2^k・3^L)   m=3k+1, n=3L,
 (a, b) = (2^k・3^L, 2^{k+1}・3^L)  m=3k, n=3L+2,
 (a, b) = (2^{k+1}・3^L, 2^k・3^L)  m=3k, n=3L+2.
は題意を満たす。

>>533
 2は正整数である。
 放物線 y = x^2 の接線で、 y = -4x-4 で表されるものが存在する。
0536132人目の素数さん
垢版 |
2019/01/29(火) 06:31:28.20ID:LI2OtV3O
>>531
数列 b_n = (2+n)(5+n)/8 を既約分数で表わす。
b_1 = 9/4,
b_2 = 7/2,
b_3 = 5,
b_4 = 27/4,
b_5 = 35/4,
b_6 = 11,
b_7 = 27/2,
b_8 = 65/4,

これの分子は・・・・う〜む
0538132人目の素数さん
垢版 |
2019/01/29(火) 16:01:22.16ID:LIA/CvMV
ごめスレチかも知れないけど困ってるから聞きたい
重積分と微分方程式の2問
島怒(siny)^2cosy/y}dxdy D={(x,y)lx/3≦y ≦x,y≦π/2}

y’+xy=x^3
0541132人目の素数さん
垢版 |
2019/01/29(火) 18:20:06.45ID:Q511uMaC
>>539
下は自力でなんとか解けたありがとう
上はまず(siny)^2cosy/yの積分が分かんないんだけど
t=tanと置くとか試したけどダメだった
0542132人目の素数さん
垢版 |
2019/01/29(火) 21:29:54.24ID:10id4Ljd
>>531
c_n =((2+n)(5+n)/8)((2(3n^4-54n^3+361n^2-1062n+1172))/(17n^2-153n+346))

c_1 = 9,
c_2 = 7,
c_3 = 5,
c_4 = 27,
c_5 = 35,
c_6 = 11,
c_7 = 27,
c_8 = 65,
0544132人目の素数さん
垢版 |
2019/01/29(火) 23:42:47.68ID:LI2OtV3O
1/(1+xx)^2 = {1/(1+xx) + (1-xx)/(1+xx)^2} /2,

∫1/(1+xx)^2 dx = (1/2)arctan(x) + x/{2(1+xx)},
0546132人目の素数さん
垢版 |
2019/01/30(水) 00:23:14.54ID:Q/seRUAS
>>538
(上)
Dより 0≦y≦x 
Dは単純な凸領域だから積分の順序を替えてもよい。
∫[y,3y] dx = 2y,
∫[0,π/2] 2sin(y)^2・cos(y) dy = [ (2/3)sin(y)^3 ](y=0,π/2) = 2/3,

(下)
y ' +xy = e^(-xx/2) {e^(xx/2)・y} ' を入れる。
y = e^(-xx/2)∫e^(xx/2)・x^3 dx
 = e^(-xx/2) {e^(xx/2)・(xx-2) + c}
 = xx - 2 + c・exp(-xx/2),
0547132人目の素数さん
垢版 |
2019/01/30(水) 01:13:09.30ID:Q/seRUAS
>>531
d_n = {-9/(n-1) +49/(n-2) -105/(n-3) +945/(n-4) -1225/(n-5) +231/(n-6) -3213/(n-7) +65/(n-8)}
* (n-1)(n-2)(n-3)(n-4)(n-5)(n-6)(n-7)(n-8)
= {-233n^7 +6734n^6 -79190n^5 +488810n^4 -1699877n^3 +3304256n^2 -3286260n+1269000)/360,
0548132人目の素数さん
垢版 |
2019/01/30(水) 02:00:10.62ID:K780wfvs
>>545
>>546
ありがとう
めちゃ勉強になります
0552132人目の素数さん
垢版 |
2019/01/30(水) 10:07:01.48ID:EmIp4qLU
正方形を、その対角線で回転させたらどのような立体になるかって、頭いい人ならなんとなくイメージできるものでしょうか?
0553132人目の素数さん
垢版 |
2019/01/30(水) 10:10:07.30ID:bRecRJu8
>>552

コーンを底面で張り合わせた立体になると誰でもわかると思います。
0555132人目の素数さん
垢版 |
2019/01/30(水) 12:08:28.11ID:bRecRJu8
>>552

正方形で分かりにくければ、直角二等辺三角形を斜辺を軸にして回転するのをイメージすればいいと思います。
0557132人目の素数さん
垢版 |
2019/01/30(水) 16:35:35.18ID:Pw4tMT9O
一般に長方形を対角線を軸に回転させた場合、できる立体は円錐の組み合わせにはなりませんよね
どんな形になりますか
0558132人目の素数さん
垢版 |
2019/01/30(水) 16:50:20.86ID:yYGa9FA5
長方形を対角線で折り返してできる
. /\/\
/ \
の回転体なので円錐+円錐の一部+円錐の一部+円錐。
0559132人目の素数さん
垢版 |
2019/01/30(水) 17:09:08.97ID:F82JL34M
円錐にならないのは立方体の対角線の回転体のように
ねじれの関係になる二直線の場合で双曲面ができる
0560132人目の素数さん
垢版 |
2019/01/30(水) 18:45:03.72ID:5DWyefwR
>>520
(-17n^4 +200n^3 -793n^2 +1258n -630)/6 ぢゃね?
0561132人目の素数さん
垢版 |
2019/01/30(水) 18:46:13.71ID:VaszuuCA
>>521
1=a^2+b^2+c^2≧3(abc)^(2/3)より-√3/9≦abc≦√3/9
0562132人目の素数さん
垢版 |
2019/01/30(水) 18:47:10.06ID:VI9EBsiF
>>534
(上)
Dより 0≦y≦x 
Dは単純な凸領域だから積分の順序を替えてもよい。
∫[y,3y] dx = 2y,
∫[0,π/2] 2sin(y)^2・cos(y) dy = [ (2/3)sin(y)^3 ](y=0,π/2) = 2/3,

(下)
y ' +xy = e^(-xx/2) {e^(xx/2)・y} ' を入れる。
y = e^(-xx/2)∫e^(xx/2)・x^3 dx
 = e^(-xx/2) {e^(xx/2)・(xx-2) + c}
 = xx - 2 + c・exp(-xx/2),
0563132人目の素数さん
垢版 |
2019/01/30(水) 18:49:56.77ID:VI9EBsiF
以下の性質を満たす集合 E は存在するか?

(1)
E ⊂ R × {0} ⊂ R × R
E は R × R で閉集合、 R × {0} で閉集合ではない。

(2)
E ⊂ R × {0} ⊂ R × R
E は R × R で閉集合ではない、 R × {0} で閉集合。
0564132人目の素数さん
垢版 |
2019/01/30(水) 18:58:25.25ID:VI9EBsiF
3 5 11 21 43 85 171 341 683 1365 2731 5461 10923 21845 43691

を一つの式で表すとどうなりますか?

末尾で1 1 3 5を繰り返すようです
0565132人目の素数さん
垢版 |
2019/01/30(水) 19:01:47.04ID:1+IaFFvp
>>423
384=8!! 

53760=2(10!!)+12!!

8755200=50(12!!)+10(14!!)

1805690880=1199(14!!)+100(16!!)

471092428800=9640(16!!)+2000(18!!)

153043438141440=23724(18!!)+40000(20!!) 
0566132人目の素数さん
垢版 |
2019/01/30(水) 19:02:36.32ID:6Sa5WD/n
ヒカキンの年収が10億超え!?明石家さんま・坂上忍も驚愕の総資産とは??
https://logtube.jp/variety/28439
【衝撃】ヒカキンの年収・月収を暴露!広告収入が15億円超え!?
https://nicotubers.com/yutuber/hikakin-nensyu-gessyu/
HIKAKIN(ヒカキン)の年収が14億円!?トップYouTuberになるまでの道のりは?
https://youtuberhyouron.com/hikakinnensyu/
ヒカキンの月収は1億円!読唇術でダウンタウンなうの坂上忍を検証!
https://mitarashi-highland.com/blog/fun/hikakin
なぜか観てしまう!!サバイバル系youtuberまとめ
http://tokyohitori.hatenablog.com/entry/2016/10/01/102830
あのPewDiePieがついに、初心YouTuber向けに「視聴回数」「チャンネル登録者数」を増やすコツを公開!
http://naototube.com/2017/08/14/for-new-youtubers/
27歳で年収8億円 女性ユーチューバー「リリー・シン」の生き方
https://headlines.yahoo.co.jp/article?a=20170802-00017174-forbes-bus_all
1年で何十億円も稼ぐ高収入ユーチューバー世界ランキングトップ10
https://gigazine.net/news/20151016-highest-paid-youtuber-2015/
おもちゃのレビューで年間12億円! 今、話題のYouTuberは6歳の男の子
https://www.businessinsider.jp/post-108355
彼女はいかにして750万人のファンがいるYouTubeスターとなったのか?
https://www.businessinsider.jp/post-242
1億円稼ぐ9歳のYouTuberがすごすぎる……アメリカで話題のEvanTubeHD
https://weekly.ascii.jp/elem/000/000/305/305548/
世界で最も稼ぐユーチューバー、2連覇の首位は年収17億円
https://forbesjapan.com/articles/detail/14474
ヒカルの収入が日収80万、月収2400万、年収3億と判明www
https://matomenewsxx.com/hikaru-income-8181.html
はじめしゃちょーの年収は6億?2017年は30億突破か?
https://2xmlabs.com/archives/1873
0567132人目の素数さん
垢版 |
2019/01/30(水) 19:10:41.45ID:HjVxEdXI
数検1級の参考書って何がいいですか?
微積と線形代数は大学数学ちょっとかじっただけで他は全くしてません
0568132人目の素数さん
垢版 |
2019/01/30(水) 19:13:54.53ID:SgRlxMiu
105 132人目の素数さん 2019/01/04(金) 18:58:57.05 ID:s68Y7dWN
リチャード・テイラーっていうイギリスの数学者はどのくらいのレベルの数学者ですか?
現役ではそこそこ上位の方に入るぐらいの学者ですか?

106 132人目の素数さん 2019/01/04(金) 23:19:10.56 ID:UTaC5hnL
>>105

底辺のものが語るべき話題にあらず
0569132人目の素数さん
垢版 |
2019/01/30(水) 23:21:25.56ID:UlrRz/dc
y’=5y/(x-2y)+2

微分方程式の解き方参考書とかないかな
0572132人目の素数さん
垢版 |
2019/01/31(木) 00:30:49.39ID:Fg4bF5ky
>>559
 一葉双曲面ですね。


>>569
同次型なので y = x・z とおくと
dy/dx = z + x(dz/dx)

x(dz/dx) = 2(1+zz)/(1-2z)
(1-2z)/(1+zz) dz = (2/x)dx   ・・・・ 変数分離型
arctan(z) -log(1+zz) = 2log(x) + c,
arctan(y/x) = log(xx+yy) + c = 2log(r) + c,

・別解
x = r・cosθ
y = r・sinθ
とおくと
dy/dx = (r・cosθ・dθ + sinθ・dr)/(cosθ・dr - r・sinθ・dθ)
 = {x・dθ + (y/r)・dr} / {(x/r)・dr - y・dθ}
 = {2x + (2/r)(dr/dθ)y} / {(2/r)(dr/dθ)・x -y},
これと
  (2x+y)/(x-2y)
を見比べて
 (2/r)(dr/dθ) = 1,
 2log(r) = θ - c,
0575132人目の素数さん
垢版 |
2019/01/31(木) 00:42:53.98ID:TPgX/goz
>>572
別解までありがとう、自分でも手動かして解いてみた
y=x・zとなんで置こうと思ったか教えて頂きたい
0577552
垢版 |
2019/01/31(木) 09:53:58.40ID:C2Un6lAb
ありがとうございます
正方形の場合は簡単でしたね
>>556
それも頭でイメージできますか?
あとその立体の体積を求めようと思ったらやはり積分が必要になりますかね
0578132人目の素数さん
垢版 |
2019/01/31(木) 10:01:10.31ID:OknIWWuv
>>577
イメージ出来る
自分はゆっくり回転させて残像をイメージする
積分使わなくても円錐の体積の足し算引き算で計算出来るんじゃないか?
0579132人目の素数さん
垢版 |
2019/01/31(木) 12:13:57.92ID:Fg4bF5ky
>>564
a_{n+1} = 2a_n + (-1)^n,
より
a_n = (2^{n+2} - (-1)^{n+2}) / 3,

生成関数
g(x) = 4/[3(1-2x)」 - 1/[3(1+x)].
0580132人目の素数さん
垢版 |
2019/01/31(木) 12:18:02.00ID:xZUnkZXT
自然数の逆数和≒log(n)
素数の逆数和 ≒log(log(n))
??の逆数和 ≒log'(log(log(n)))

??の部分は何になるのでしょうか?
0586132人目の素数さん
垢版 |
2019/01/31(木) 15:24:49.13ID:xZUnkZXT
あっ失礼。この定理はlogが足りなかった。定数倍違うだけだった
>>583 
へー メルセンヌ数の2^n -1の素因数の和ですか。
この和はたとえメルセンヌ素数が有限個でも関係ないですね
0589132人目の素数さん
垢版 |
2019/01/31(木) 23:21:36.56ID:kMNaa1mT
a,bを実数とする。
次の条件を満たす実数xをaとbで構成せよ(aとbで表せ)
・a<x<b
・xは無理数
0590132人目の素数さん
垢版 |
2019/02/01(金) 01:33:51.20ID:qU8e3dCF
a,bが異符号ならa=b/2とかb=a/2とかにして同符号にしておく
a,b>0なら
n=[log_2(b-a)]-2, c=2^n, x=c√(-[-a/c] [b/c])
a,b<0も同様
0594132人目の素数さん
垢版 |
2019/02/01(金) 03:27:31.23ID:q6zhJLvW
>>590
ルートやlogのような初等関数だけで常に無理数を作れるようにはできませんか?
確かにガウス記号使えばどんなa,bに対しても簡単に無理数を作れますが
0595132人目の素数さん
垢版 |
2019/02/01(金) 04:39:01.84ID:nDpUgAzk
>>592

y = x^{3/2}
y ' = (3/2)√x,

s = (3/2)∫√(x +4/9) dx = (x +4/9)^{3/2} = (1/27)(4+9x)^{3/2}
x=0 から x=1 まで
s = (1/27)[13^{3/2} - 8] = 1.439709873
0597132人目の素数さん
垢版 |
2019/02/01(金) 06:46:24.81ID:nDpUgAzk
>>589
2/(b-a) より大きい自然数nをとると b-a > 2/n,

a < m/(n√2) < b を満たす整数mが2つ以上あり、その一方は0でない。
0598132人目の素数さん
垢版 |
2019/02/01(金) 11:59:23.48ID:Y36rLnVz
線型代数の応用です。
自分にはさっぱりわからなくて。
どなたかわかる方いらっしゃいましたらお願いします。
https://i.imgur.com/qI1Zzaw.jpg

または参考にできるサイトなどの紹介でも構いません。
0599132人目の素数さん
垢版 |
2019/02/01(金) 18:57:06.92ID:/HfsC4j/
ヒカキンの年収が10億超え!?明石家さんま・坂上忍も驚愕の総資産とは??
https://logtube.jp/variety/28439
【衝撃】ヒカキンの年収・月収を暴露!広告収入が15億円超え!?
https://nicotubers.com/yutuber/hikakin-nensyu-gessyu/
HIKAKIN(ヒカキン)の年収が14億円!?トップYouTuberになるまでの道のりは?
https://youtuberhyouron.com/hikakinnensyu/
ヒカキンの月収は1億円!読唇術でダウンタウンなうの坂上忍を検証!
https://mitarashi-highland.com/blog/fun/hikakin
なぜか観てしまう!!サバイバル系youtuberまとめ
http://tokyohitori.hatenablog.com/entry/2016/10/01/102830
あのPewDiePieがついに、初心YouTuber向けに「視聴回数」「チャンネル登録者数」を増やすコツを公開!
http://naototube.com/2017/08/14/for-new-youtubers/
27歳で年収8億円 女性ユーチューバー「リリー・シン」の生き方
https://headlines.yahoo.co.jp/article?a=20170802-00017174-forbes-bus_all
1年で何十億円も稼ぐ高収入ユーチューバー世界ランキングトップ10
https://gigazine.net/news/20151016-highest-paid-youtuber-2015/
おもちゃのレビューで年間12億円! 今、話題のYouTuberは6歳の男の子
https://www.businessinsider.jp/post-108355
彼女はいかにして750万人のファンがいるYouTubeスターとなったのか?
https://www.businessinsider.jp/post-242
1億円稼ぐ9歳のYouTuberがすごすぎる……アメリカで話題のEvanTubeHD
https://weekly.ascii.jp/elem/000/000/305/305548/
世界で最も稼ぐユーチューバー、2連覇の首位は年収17億円
https://forbesjapan.com/articles/detail/14474
ヒカルの収入が日収80万、月収2400万、年収3億と判明www
https://matomenewsxx.com/hikaru-income-8181.html
はじめしゃちょーの年収は6億?2017年は30億突破か?
https://2xmlabs.com/archives/1873
0600132人目の素数さん
垢版 |
2019/02/01(金) 20:08:23.68ID:kkJzd9oh
複素数平面の質問です

z/(z+1)の偏角がπ/6となるような複素数平面上の点zの集合を求めてください。

よろしくおねがいします。
0602132人目の素数さん
垢版 |
2019/02/01(金) 20:53:56.23ID:kXZ7vBVy
過程も軽くでいいんでおねがいします
0603132人目の素数さん
垢版 |
2019/02/01(金) 21:07:47.41ID:joxOwWuT
z/(z+1)=(0-z)/(-1-z)だから0,-1,zのなすzの角度がπ/6
一方0,-1, √3iのなす√3iの角度もπ/6
ゆえにzは0,-1,√3iを通る円
0604132人目の素数さん
垢版 |
2019/02/01(金) 21:16:23.76ID:k6WJPdDp
ただし虚部 i の係数は正とする

図を描くと下を向いたランドルト環になるはず
0605132人目の素数さん
垢版 |
2019/02/01(金) 21:28:05.68ID:kXZ7vBVy
あー円周角の定理か…

ありがとうございます!
0608132人目の素数さん
垢版 |
2019/02/01(金) 22:56:31.43ID:5SaBmCFF
誰だか分からない人間にとやかく言われても迷惑なだけ。
「おりろ」も聞こえてくるが、何を意味しているのか分からない。
迷惑な、意味不明発言を聞かせるのを止めろ。
0609132人目の素数さん
垢版 |
2019/02/01(金) 22:59:48.46ID:5SaBmCFF
「調子に乗るのもいい加減にしろ」と聞こえてくるが
具体的に私の何が調子に乗っていて、それが誰のどんな迷惑なのかをはっきり言え
それができないのだったら、意味不明な誹謗を繰り返すのを止めろ
0610132人目の素数さん
垢版 |
2019/02/02(土) 09:16:08.52ID:oMGqsdrI
>>600

z/(z+1) の偏角が a のとき、e^(-ia)・z/(z+1) は実数だから、
0 = e^(-ia)・z/(z+1) - e^(ia)・z~/(z~+1) = 0,
2i(z+1)(z~+1)/sin(a) を掛けて
0 = 2{i/tan(a) +1}z(z~+1) - 2{i/tan(a) -1}z~(z+1)
 = (2z+1)(2z~+1) + 2{i/tan(a)}(z-z~) -1
 = {2z +1 -i/tan(a)}{2z~ +1 +i/tan(a)} -1/tan(a)^2 -1
 = |2z +1 -i/tan(a)|^2 - 1/sin(a)^2,

∴ 中心 (-1 +i/tan(a))/2, 半径 1/2sin(a) の円周。(z≠-1)
0611132人目の素数さん
垢版 |
2019/02/02(土) 10:10:55.35ID:g2kqySN6
>>606
医学部卒だが必要な場面は臨床ではない。
統計は必要だがソフトが扱えれば足りる。
0615132人目の素数さん
垢版 |
2019/02/02(土) 13:52:38.64ID:WAPXW2T6
中学生レベルですみません。

赤玉2個青玉3個白玉5個の10個中から4つを同時に取り出すとき、取り出した4つの中に青赤がそれぞれ一つ以上含まれる確率は?

って問題で、自分は
全事象を10C4で210通り、
青玉赤玉が一個以上あればいいから、青赤を一つずつ取り出しておいて、残りの二つを選んで8C2で28通り
にして2/15と考えたんですけど、なにが間違っているのか教えてもらってもいいですか?
0616132人目の素数さん
垢版 |
2019/02/02(土) 14:58:44.06ID:EvfEWSFA
>>615
> 全事象を10C4で210通り
これって赤1、赤2、青1〜青3、白1〜白5と全部区別してその中から4つ取り出すって意味でしょ?
確率なのでそう考えるのはよいと思う
しかし、
> 青玉赤玉が一個以上あればいいから、青赤を一つずつ取り出しておいて、残りの二つを選んで8C2で28通り
これは例えば「赤1と青1を取り出し、残りの8個から2個選ぶ」というような考え方になってしまっていないか?
赤1や青1を取り出さない場合を考慮出来ていない
赤1個、青1個、白2個である場合だけでも2C1*3C1*5C2で60通りある
0617132人目の素数さん
垢版 |
2019/02/02(土) 15:16:10.02ID:QF340/47
1 14 190 2799 45640 823724

を一つの式にしてくれ〜(・ω・)ノ
0618132人目の素数さん
垢版 |
2019/02/02(土) 15:17:10.69ID:ukeNAhOc
>「赤1と青1を取り出し、残りの8個から2個選ぶ」というような考え方
この場合、たとえば、赤2,青1,白1,白5の4つを取り出したパターンはその28通りには含まれない
つまり漏れがあるってこと
0619132人目の素数さん
垢版 |
2019/02/02(土) 15:47:14.21ID:u0aYLgOM
>>615
確率でコンビネーションを正しく使ってないから

そういう解き方じゃなくてもっとオーソドックスにやったほうがいい

確率は場合の数と違って同じ色の玉でも区別する
(例えば赤1〜2青1〜3.白1〜5)
つまり全事象は10P4

10C4は1〜10と書かれた玉を4つ取り出した時の組み合わせの全通りであって、場合の数では全ての組み合わせだけど、確率に於いては全事象として扱うことはできるけど、細かくいうと全通りではないではない
1個ずつ4回引くのと4つ一気に引くのは本質的に同じ
(たとえば箱に手を入れて4つ一気に選ぶ時も箱の中で一個ずつ手に取ってる)
これがよくわからない場合、簡単にいうと確率では下手にCは使わないほうがいい

長いから回答は別に書く、ストレートに求めるのと余事象で出すの、検算にもなるし
答えは11/21
0620132人目の素数さん
垢版 |
2019/02/02(土) 15:50:45.33ID:u0aYLgOM
>>619
すべての玉を区別するなら全事象は10P4だから5040通り
次に 求める事象を 赤と青がすくなくとも一個ずつあるのを素直にその事象でとくか、全事象から赤と青が両方ない組み合わせを全部引くかの二択になる

前者
赤と青の組み合わせにはしたのようなものがある(全てではない)
1赤 青 白 白
2青 赤 赤 青
3白 白 青 赤
4白 白 白 白
(さらにこれには上で区別したように番号もある)
上みたいに赤と青がある組み合わせを全部出す必要がある
赤青を含む結果は
赤青白白 赤赤青白 赤青青青 赤青青白 赤赤青青
この五種類

上の取り出した順番1〜4で示したようにこの五種類は取り出した順番によって組み合わせも別々に書く必要があり、さらに存在する色の玉の個数によってわける必要もある
赤青白白
赤2種 青3種 白5種あるため
赤2種のうち1つ×青3種のうち1つ×白5種のうち2つ×取り出す順番の組み合わせ
2×3×5C2×4!
1440 というふうに求める
赤赤青白
赤は2種だから1通り×3×5×4!
360
赤青青青
赤2種のうち1つ×青は全部だから1通り
2×4!
48
赤青青白
2×3C2×5×4!
720
赤赤青青
1×3×4!
72
合計2640通り
答え2640/5040
0621132人目の素数さん
垢版 |
2019/02/02(土) 15:51:12.50ID:u0aYLgOM
>>620
後者
後者の合計は上の事象から1引いた2400になるはず
赤と青を両方引かないパターン
赤赤白白 赤白白白 青白白白 青青白白 青青青白
白白白白
赤赤白白
5C2×4! 240
赤白白白
2×5C3×4! 480
青白白白
3×5C3×4! 720
青青白白
3×10×24 720
青青青白
5×24 120
白白白白
5×24 120
合計2400
余事象なので
1-(2400/5040)
2640/5040
0622132人目の素数さん
垢版 |
2019/02/02(土) 15:58:54.00ID:WAPXW2T6
>>616 >>618
なるほど、
たしかに改めて考え直すと赤1青1を取り出さない場合が漏れてしまいますね。

この考え方で場合分けして計算したら答えと一致しました。ご指摘ありがとうございました
0623132人目の素数さん
垢版 |
2019/02/02(土) 16:01:08.79ID:X0sFMfIX
>>615
よくありそうな質問なら、
青赤1個ずつ選んでから残り8個から2個選んで*8C2ではなぜ間違いか?
だろうけど、3C1*2C1を掛けてないからそれでもないし質問がアホすぎ
0624132人目の素数さん
垢版 |
2019/02/02(土) 16:07:08.67ID:dbpYVLjG
CでもPでも論理さえ合っていれば問題ない

10個の玉から4個を取り出す組合せが 10C4=210通り
赤以外の8個から4個を取り出す組合せが 8C4=70通り
青以外の7個から4個を取り出す組合せが 7C4=35通り
赤青以外の5個から4個を取り出す組合せが 5C4=5通り

(赤有and青有)
=(全事象)-(赤無and青有)-(赤有and青無)-(赤無and青無)
=(全事象)-{(赤無and青有)+(赤無and青無)}+(赤無and青無)-{(赤有and青無)+(赤無and青無)}+(赤無and青無)-(赤無and青無)
=(全事象)-(赤無)-(青無)+(赤無and青無)
=10C4-8C4-7C4+5C4=110
0625132人目の素数さん
垢版 |
2019/02/02(土) 16:13:07.09ID:AllqBdzw
>>621

みればわかるけど全部に4!をかけてるけど、10C4はそれを省く形になる
そうなると求める事象も4!割ってる前提で考えないといけない
それを忘れてるのと、そもそもの確率の求め方に抜けがある

余計な一言加えるなら慣れるまではちゃんと全通り考えてやったほうがいい、あと確率は検算も間違ってる可能性があるから間違った解説が無いように答えを付け加えといたほうがいい
0626132人目の素数さん
垢版 |
2019/02/02(土) 17:51:43.85ID:pufI1A9m
aを正数とする。
実数の列x_1,x_2,...は任意のiに対し0<x_i<aをみたし、またk<aなるどのような実数kに対してもk<x_jなるjを上手く選ぶことができる。
このとき、以下の命題が偽であることを証明せよ。
命題「lim[n→∞] x_i = a」
0629132人目の素数さん
垢版 |
2019/02/02(土) 20:04:05.31ID:QF340/47
1 14 190 2799 45640 823724 16372071


(13+n)!/(12!)+2n(2n!)^2+n^2+1

を補正してくれ〜(・ω・)ノ
0631132人目の素数さん
垢版 |
2019/02/03(日) 02:13:53.72ID:HJPGw57h
pn+1=5/14pn+(1ーpn)*2/2もとい
pn+1=pn^2ー2/14*pn+1/4の漸化式の解き方が分かりません。
上の式を特性方程式に当てはめるとpnと一緒にpn*2が残って詰むし、下の形はネットで色々検索したけど力不足で分かりませんでした。明日の昼までに解き方だけでもいいので誰か教えて頂けませんか?
0632132人目の素数さん
垢版 |
2019/02/03(日) 02:22:27.34ID:MLglhrmw
>>631
掲示板での式の書き方を調べて書き直してください
それか問題を画像で上げてください
0634132人目の素数さん
垢版 |
2019/02/03(日) 09:18:01.66ID:V7EfVs39
>>626
あとは
x_i = a ( sin i )^2
でも行けそうな気がするけど、証明はできないw

ちなみに角度はラジアンね
0636132人目の素数さん
垢版 |
2019/02/03(日) 10:42:43.54ID:AejEpGuh
>>635
たとえばだけど、
 0<k<a ⇒ a/{2(a-k)} より大きい偶数 j をとれる。(アルキメデスの原理)
 k < x_j < a
0637132人目の素数さん
垢版 |
2019/02/03(日) 11:32:56.84ID:AejEpGuh
>>598
問1だけ。

P_i における↑のx座標 x_i が  x_3 < x_2 < x_1  を満たせばよい。
5本の筋から3本選ぶから C[5,3] = 10 とおり。

実例) 四ッ橋筋、心斎橋筋、松屋町筋、堺筋、谷町筋
0638132人目の素数さん
垢版 |
2019/02/03(日) 12:10:44.73ID:AejEpGuh
>>628
(上)
I.
 1. y = (x-1)^2 とおく。
 左辺 = y(y+1) + y -8 = (y+4)(y-2),
 y+4 = (x-1)^2 + 4 > 0 ゆえ 
 (x-1)^2 -2 = y-2 = 0,
 x = 1±√2.

 2.
 y = |2x-1| + x
  = 1-x  (x≦1/2)
  = 3x-1  (x≧1/2)
 これは (1/2, 1/2) を下端とし、左側は傾き-1、右側は傾き3である。
 もしも a≧14 とすれば、{x≧5 または x≦-13} となる。
 ∴ 存在しない。

II.
 1. "N" 2枚、"A" 2枚を区別すれば 6! とおり。
   そのうち「ながの」は 4とおり。4/6! = 1/180
 2. "N" "G" "N" が奇数番目、"A" "A" "O" が偶数番目にくる。
   (3!)(3!)/6! = 1/20
 3. "G" と "O" の位置が各3とおり だから 9通り
0639132人目の素数さん
垢版 |
2019/02/03(日) 12:20:30.62ID:Z7WgdjHw
>>628
変な問題だよな
IIIだけマトモそう
1. π/3
2. √3
3. 5
0640132人目の素数さん
垢版 |
2019/02/03(日) 12:20:41.40ID:AejEpGuh
>>628
(下)
III
 1. ∠CAB = (1/2)∠COB = π(n+1)/{n+(n+1)+(n+2)} = π/3,
 2. 2sin(∠COB/2) = 2sin(π/3) = √3 (cm)
 3. ∠BCA = (1/2)∠BOA = πn/{n+(n+1)+(n+2)} = πn/{3(n+1)},
   50°= (5/18)π だから、n/(n+1) = 5/6, n=5.
0641132人目の素数さん
垢版 |
2019/02/03(日) 15:32:05.41ID:wLIpAp6k
f をある区間で連続かつ1対1であるとする。

f はその区間で増加または減少関数であることを示せ。
0643132人目の素数さん
垢版 |
2019/02/03(日) 16:56:45.76ID:I+9lMF3X
>>641
背理法で証明する。
I=[a, b], a≠bとする。
あるx_1, x_2∈Iが存在し, x_1 < x_2かつf(x_1) ≧ f(x_2)と仮定する。
f(x_1) = f(x_2)のときは明らかにfの単射性に反する。
f(x_1) > f(x_2)のときは、
fはIで連続なので中間値の定理よりf(c) = f(x_2) となるcがaとx_1の間に存在する。
これはfの単射性に反する。
0644132人目の素数さん
垢版 |
2019/02/03(日) 17:31:10.26ID:eUd+pqcC
>>642
三角形APQの面積は、
PがAB上にあるときはAPを底辺にして考える
PがBC上にあるときはAQを底辺にして考える
とすると早いかな
0645132人目の素数さん
垢版 |
2019/02/03(日) 17:40:03.61ID:QijVyK5+
>>644
4≦x≦5がわからないです……
0646132人目の素数さん
垢版 |
2019/02/03(日) 17:49:46.22ID:xBNc0f0v
>>645
後半は削ったほうが楽じゃないかな

QからBCに垂線QHを引いたら、△CQH∽△CABで、CQがわかってるから
QH、CHの長さもわかって、△CQHの面積がわかる。(相似比使ってもいいけど)
△APQ=△APC-△CPQ

PC=14-2xで
△APC=PC×AB/2
だから、△APCもわかる。

なので、△APQも計算できるよーでおしまい。
0647132人目の素数さん
垢版 |
2019/02/03(日) 17:57:09.36ID:xBNc0f0v
>>646
自分のメモが汚すぎて間違えた…

>> QからBCに垂線QHを引いたら、△CQH∽△CABで、CQがわかってるから
>> QH、CHの長さもわかって、△CQHの面積がわかる。(相似比使ってもいいけど)

QからBCに垂線QHを引いたら、△CQH∽△CABで、CQがわかってるから
QHの長さもわかって、
△CPQ=QH×CP/2
だから、△CPQの面積がわかる。(辺の比で△CPQ=△ABC×(CQ/10)×(CP/6)でもいいか)

===
てことです。
0648132人目の素数さん
垢版 |
2019/02/03(日) 18:03:01.82ID:doFFc48n
>>642
問題が間違ってないか?

0<x≦4
y=(6/5)* x^2

4≦x≦5
y=(8/5)*|x^2-4x-3|

5≦x≦7
y=8(7-t)
0649132人目の素数さん
垢版 |
2019/02/03(日) 18:18:02.26ID:doFFc48n
すまん、間違えた

0<x≦4
y=(6/5)* x^2

4≦x≦5
y=(8/5)*x(7-x)

5≦x≦7
y=8(7-x)
0651132人目の素数さん
垢版 |
2019/02/03(日) 19:33:47.20ID:m6pOABtb
頂点Cから辺ABへの距離は 8×6÷10=24/5
x=4 のとき P=C なので、AQを底辺としたときの△APQの高さは 24/5、
x=5 のときはその (4/6) 倍で 16/5
これらから、AQを底辺としたときの△APQの高さ=56/5-(8/5)x
一方、AQ=2x
よって △APQ = (1/2)AQ×高さ = (以下略)
0652132人目の素数さん
垢版 |
2019/02/03(日) 20:04:35.22ID:wLIpAp6k
>>643

間違っていますね。
0653132人目の素数さん
垢版 |
2019/02/03(日) 20:06:04.18ID:wLIpAp6k
f が単調増加だと仮定すると、 f は単調増加であるということを言っているだけですよね。
0654132人目の素数さん
垢版 |
2019/02/03(日) 20:37:33.77ID:oamzFL6l
問題解いて欲しいが、画像の貼り方が分からん
0655132人目の素数さん
垢版 |
2019/02/03(日) 20:39:03.49ID:Mli4VbJ1
>>652
問題の区間Iの設定、定義が曖昧。
実数区間なのか順序は?など。
実数の閉区間と仮定するが、
背理法で証明する。
I=[a, b], a≠bとする。fは単射だからf(a)=f(b)は明らかに成り立たないので、
f(a)<f(b)とし、単調増であることを示す。単調減の場合の証明も同様。

fは閉区間Iで連続なのでIで最小値mをとる。そのときm=f(α)とする。
同様に、fはIで最大値M=f(β)をとる。
J=[α, β]⊂Iでfが単調増であることを示す。
あるx_1, x_2∈Jが存在し, x_1 < x_2かつf(x_1) ≧ f(x_2)と仮定する。
f(x_1) = f(x_2)のときは明らかにfの単射性に反する。
f(x_1) > f(x_2)のときは、
fはJで連続なので中間値の定理よりf(c) = f(x_2) となるcがαとx_1の間に存在する。
これはfの単射性に反する。つまり、
任意のx_1, x_2∈Jに対し, x_1 < x_2ならば、f(x_1) < f(x_2)
a<αかつf(a)≧f(α)と仮定すると、
これまでと同様の議論でf(a)=f(α)のときは明らかに矛盾、また、
不等号のときは十分小さいαの近傍で
中間値の定理より
s,t∈I, s<α<tでf(s)=f(t)となるs,tが存在し、単射性に反する。
よって、a<αならば、f(a)<f(α)
しかし、これはf(α)の最小性に矛盾。
よってa=αでなければならない。
b=βの証明も同様。
0656132人目の素数さん
垢版 |
2019/02/03(日) 20:56:03.96ID:qNjH1yw4
数学の問題を説いたのですがあってるのかわからないので確認お願いします
大学一年ですのでそこまで難しくないはずです...
https://imgur.com/NVlFnub
https://imgur.com/PdiiDn7
0657132人目の素数さん
垢版 |
2019/02/03(日) 21:03:55.20ID:wLIpAp6k
>>655

区間は、以下のどれかです。

(a, b)
(a, b]
[a, b)
[a, b]
(-∞, b)
(-∞, b]
(a, +∞)
[a, +∞)
(-∞, +∞)

但し、 a ≦ b、 a, b ∈ R とする。
0658132人目の素数さん
垢版 |
2019/02/03(日) 21:06:04.55ID:wLIpAp6k
なお、すべての場合に通用するエレガントな解法が教科書には書いてありましたが、素朴な方法が知りたかったので質問しました。
0659132人目の素数さん
垢版 |
2019/02/03(日) 21:29:02.04ID:nxEzIXIW
数学初心者で、彌永先生の集合位相を読んでます。
非順序対の存在は証明できたのですが、「問題 1.5 のつぎの (@), (A) が成り立つことを示せ.」がよくわかりません。どなたかヒントください。
https://i.imgur.com/7cup5Rf.jpg
0661132人目の素数さん
垢版 |
2019/02/03(日) 22:16:05.99ID:4+CPWVnX
>>660
4(√2 -1), 8√2
であってる?
0662132人目の素数さん
垢版 |
2019/02/03(日) 22:20:25.33ID:4+CPWVnX
>>661
面積間違えた
0663132人目の素数さん
垢版 |
2019/02/03(日) 22:29:01.14ID:4+CPWVnX
>>660
OE=4(√2 -1),
△BEF=8(2 -√2)
0664132人目の素数さん
垢版 |
2019/02/03(日) 23:22:11.18ID:YUVIoBEx
sum[k=0...min(p,q)] C{p,k} C{q,k} = (p+q)!/ (p! q! )
ちょいと意味合いを考えれば この等式が成立する事は明らかなのですが
式変形での示し方がさっぱり分からないので教えてください。
0666イナ ◆/7jUdUKiSM
垢版 |
2019/02/04(月) 01:05:15.21ID:wo4Q+gL7
>>660
OE=4√2-4
(DE=8-4√2)
たぶんOEだと思うけど、
手書きでDEにも見えるんで一応。
△EFB=8√2・(8-4√2)^2/(4√2)^2・4√2/(8-4√2)
=8√2・(8-4√2)/4√2
=16-8√2
あってるかな?
0667132人目の素数さん
垢版 |
2019/02/04(月) 01:46:18.46ID:/0JrilkG
可微分多様体における埋め込みを、「単射かつ固有(コンパクト部分集合の逆像がコンパクト)なはめ込み」と定義するのは一般的ですか?
0669132人目の素数さん
垢版 |
2019/02/04(月) 02:31:42.06ID:v5Ygf6l9
X_0,X_1を単連結かつ局所弧状連結としてp_0:X_0→X,p_1:X_1→Xをn重被覆とするとき、p_1,p_2が同値であることを示せ

S1,S1×S1,単連結かつ局所弧状連結な位相空間、それぞれを底空間とする被覆空間を全て求めよ

お願いします
0670132人目の素数さん
垢版 |
2019/02/04(月) 02:55:28.39ID:JprV8WCK
>>667
一般的ではないと思います
開部分多様体の包含写像は埋め込みと考えるのが普通ですが、properとは限りません

>>669
被覆空間の理論、特に基本群との関わりや普遍被覆空間について勉強してください
被覆理論の中ではとても基本的な問題です
0672132人目の素数さん
垢版 |
2019/02/04(月) 09:15:42.93ID:862M2DRs
>>641
 g(x,y) = (y-x){f(y)-f(x)}
とおくと、g は連続である。
fは1対1だから
 x≠y ⇒ g(x,y) ≠ 0,

背理法で証明する。
領域 D = {(x,y) | x<y } で考える。
或る A,B∈D に対して g(A)g(B) <0 だったと仮定する。
領域Dは連結だから AとBを結ぶJordan弧Γが存在する。
gは連続だから、中間値の定理によりΓ上の或る点C で g(C)=0 となる。(矛盾)
∴ g はDで定符号。
∴ f は増加関数または減少関数。
0674132人目の素数さん
垢版 |
2019/02/04(月) 12:28:06.59ID:UQQ+IDNz
3x3のマス目で三目並べをすると両者最善をつくすと引き分けになりますが
両者がランダムに打つと先手が勝つ確率と引き分けの確率はどうなるか
0675イナ ◆/7jUdUKiSM
垢版 |
2019/02/04(月) 12:38:02.59ID:wo4Q+gL7
>>666
>>674そんなことないだろ。先手なら勝つ。
□┃□┃●
━╋━╋━
□┃□┃□
━╋━╋━
□┃□┃□
0677イナ ◆/7jUdUKiSM
垢版 |
2019/02/04(月) 13:10:01.73ID:wo4Q+gL7
>>675
>>676見ろ、俺の勝ちだ。
□┃□┃●
━╋━╋━
□┃×┃□
━╋━╋━
●┃□┃□
0679イナ ◆/7jUdUKiSM
垢版 |
2019/02/04(月) 13:20:58.00ID:wo4Q+gL7
>>677
>>678やりおるな。
□┃□┃●
━╋━╋━
●┃×┃×
━╋━╋━
●┃□┃□
0683132人目の素数さん
垢版 |
2019/02/04(月) 14:27:22.30ID:UZzDpsmh
>>674
先手後手ともに空いているマスを無作為に選んで打つ場合の確率

先手勝ち 約57.3% 17327/30240
後手勝ち 約30.0% 9073/30240
引き分け 約12.7%   8/63
0684イナ ◆/7jUdUKiSM
垢版 |
2019/02/04(月) 14:36:01.31ID:wo4Q+gL7
>>681
>>682やるさ。やらいでか。せやて勝つかもしれんじゃないか。
×┃●┃●
━╋━╋━
●┃×┃×
━╋━╋━
●┃×┃●
0686132人目の素数さん
垢版 |
2019/02/04(月) 15:17:24.46ID:lKG5DED2
係数と定数項が0でない座標空間内の平面式を作り、それを表す方程式が重回帰式になるようにその平面上にない点を含む4点を決めよ。

解析学の問題です。どなたかお願いします…
0688132人目の素数さん
垢版 |
2019/02/04(月) 16:25:51.82ID:UZzDpsmh
>>685
導出方法
盤面の9マスを5つの●と4つの×で埋める組み合わせは、対称形も含めて 9C5=9C4=126通り。そのうち、
1)●×共ビンゴなし:16通り
2)●のみビンゴあり:62通り
3)×のみビンゴあり:12通り
4)●×共ビンゴあり:36通り
(1)〜(3)について結果は明らか。
(4)は、先にビンゴを取る確率は手順で決まるが、各々数え上げると ●:×=13:27 の比率となる。

以上より、
先手勝ち = 62/126+(36/126)*(13/40) = 737/1260
後手勝ち = 12/126+(36/126)*(27/40) = 121/420
引き分け = 16/126 = 8/63
0689132人目の素数さん
垢版 |
2019/02/04(月) 21:23:55.19ID:+f81JiDc
(2xx+1)/xx+x+1
島度ysin(π(xx+yy))dxdy D= xx+yy≦1 0≦y≦x

お願いします!
0690132人目の素数さん
垢版 |
2019/02/04(月) 21:54:56.63ID:yj36nFFJ
xy座標に対して新しくXY座標を以下の手順で作る。

・XY座標の原点(0,0)は、xy平面の座標では(a,b)である。a,bは負でない実数である。
・X軸の正の向きは、x軸の正の向きから反時計回りにθだけ回転している(0<θ<π/2)。
・Y軸の正の向きはX軸の正の向きから反時計回りにπ/2だけ回転している。

このとき、xy平面でもXY平面でも座標が同じ点が存在することを示せ。
またそのような点の個数は何個か。
0691132人目の素数さん
垢版 |
2019/02/04(月) 23:15:12.79ID:ckgBClyT
>>637
ありがとうございます
3.4わかるかたいますか?
途中まででも結構です
0692132人目の素数さん
垢版 |
2019/02/05(火) 10:59:13.23ID:xI3EwwZt
>>689
(上)
 (2xx+1)/(xx+x+1) = 2 - (2x+1)/(xx+x+1),
∫(2xx+1)/(xx+x+1) dx = 2x - log(xx+x+1) +c,

(下)
 X = πxx, Y = πyy とおくと
 dX = 2πx dx, dY = 2πy dy
∬ xy sin(π(xx+yy)) dx dy
 = (1/2π)^2 ∬ sin(X+Y)) dX dY
 = (1/2π)^2 ∫{ -cos(X+Y)} dY
 = -(1/2π)^2 sin(X+Y)
 = -(1/2π)^2 sin(π(xx+yy)),
0693132人目の素数さん
垢版 |
2019/02/05(火) 11:36:04.34ID:xI3EwwZt
>>689 (下) 続き

D~ = {(X,Y)| X+Y≦π, 0≦Y≦X } = {(X,Y)| 0≦X≦π, 0≦Y≦min(X,π-X)}
より
∬_D xy sin(π(xx+yy)) dy dx
 = (1/2π)^2 (1/2)∬_D~ sin(X+Y) dY dX
 = (1/2π)^2 ∫[0,π] ∫[0,min(X,π-X)] sin(X+Y) dY dX
 = (1/2π)^2 ∫[0,π/2] ∫[0,X] sin(X+Y) dY dX + (1/2π)^2 ∫[π/2,π] ∫[0,π-X] sin(X+Y) dY dX
 = (1/2π)^2 ∫[0,π/2] {cos(X) - cos(2X)}dX + (1/2π)^2 ∫[π/2,π] {cos(X) - cos(π)}dX
 = (1/2π)^2 [ sin(X) -(1/2)sin(2X) ](X=0,π/2) + (1/2π)^2 [ sin(X) +X ](X=π/2,π)
 = (1/2π)^2 + (1/2π)^2 (π/2 -1)
 = (1/2π)^2 (π/2)
 = 1/(8π),
0694132人目の素数さん
垢版 |
2019/02/05(火) 13:42:11.38ID:da9jPBxY
>>690
行列を使う。座標変換の手順通り地道に計算するだけ。
θの範囲の条件から>0となり
係数行列が逆行列をもち(x,y)が定まることがわかる。

ちなみに不動点の座標(x,y)は
f(θ)=sinθ/(1-cosθ)とすると
x=(a-b*f(θ))/2
y=(a*f(θ)+b)/2
0696132人目の素数さん
垢版 |
2019/02/05(火) 16:47:31.55ID:S8MgQgy4
複素平面と行列のどちらかを選ぶときに行列を選ぶのが20世紀少年。
俺もおっさんなので特に理由がなければ行列を使う。
0697132人目の素数さん
垢版 |
2019/02/05(火) 17:52:56.25ID:9pU6PnXd
東進数学コンクールの問題に挑戦しようとしたのですが、無理でした。
締め切りを過ぎたので、今回はあきらめようと思うのですが、答が気になってしかたないのです。
どなたか、ヒントでもいいので、お願いいたします。
https://www.toshin.com/sp/concours/
0698132人目の素数さん
垢版 |
2019/02/05(火) 18:00:15.15ID:mC/meCKO
>>695
複素数で回転させたとしても、
実部、虚部を係数比較して連立方程式が出てくるだろ?
それが解をもつ条件を考えて解くとき、
どっちにしても線型代数の知識を使うんじゃね?
0699132人目の素数さん
垢版 |
2019/02/05(火) 18:14:24.69ID:Lrv9KmWr
>>690を複素数で解くなら
解を持つことを示せばいいだけだから

zの1次方程式を作る
θの条件より、zの1次の項≠0
∴zは解を持つ

でおk
同時に解が1つのみであることも示せる
0700132人目の素数さん
垢版 |
2019/02/05(火) 18:19:44.41ID:eQPcBc1g
四面体OABCがあり、3辺OA,OB,OCはともに長さ3で互いに垂直である。
1.頂点Oから底面ABCへ下ろした垂線OHの長さを求めよ

どうかお願いします
0702132人目の素数さん
垢版 |
2019/02/05(火) 18:34:01.80ID:eQPcBc1g
>>701
すみません、解き方も教えていただけないでしょうか。
0703132人目の素数さん
垢版 |
2019/02/05(火) 18:49:27.17ID:xXDh4jM7
aを正の実数とする。2円C1とC2があり、C1の方程式は
C1: x^2+(y-a)^2=a^2
である。C2は半径rの円で、C1とx>0の領域で外接し、またx軸と接しており、C1とC2は互いに外部にある。

(1)rとaの満たす関係式を求めよ。

(2)C1、C2、のいずれとも外接し、x軸と接する円をC3とおく。円Ciの中心をPiとおき、△P1P2P3の面積をS(r)とおく。
T(r)=(√3/4)*(r+a)^2とおくとき、r>0におけるS(r)/T(r)の増減を調べよ。
0705132人目の素数さん
垢版 |
2019/02/05(火) 19:38:40.62ID:rBQy+YKy
ボードゲームの原価率はどうあるべきか?
http://yaplog.jp/drosselmeyers/archive/115
自作カードゲームの印刷価格を調べてみた
https://kdsn.xyz/original_card_print_price/
知育に最適、自作教材!かんたんオリジナルカードゲームで数字の勉強!
http://yuu73.xsrv.jp/jisaku-kyouzai
ゼロから始めるボードゲーム制作
https://trap.jp/post/450/
ボードゲームはどう作るのか、自分なりに考えた
http://roy.hatenablog.com/entry/2014/07/09/124824
カフェも急増 ボードゲームにアラサーがハマる理由
http://style.nikkei.com/article/DGXMZO10921930R21C16A2000000?channel=DF260120166491
ボードゲームの展示イベント「ゲームマーケット」の成長記録からこれからの
市場に必要なことを妄想してみた。6年間の来場者数推移(2016年4月時点調べ)
https://bodoge.hoobby.net/columns/00001
ボードゲーム市場がクラウドファンディングの出現で急成長を遂げ市場規模を拡大中
http://gigazine.net/news/20150820-board-game-crowdfunding/
0706イナ ◆/7jUdUKiSM
垢版 |
2019/02/05(火) 20:12:32.52ID:3ifx+gi9
>>700
OA、OB、OCはたがいに直交するから、
AB=AC=BC=3√2
四面体OABC=△ABC×OH×(1/3)
=(1/2)×3^2×(1/3)×3
=9/2
△ABCは一辺3√2の正三角形なんで、底辺をBC=3√2と見ると、高さはAB=3√2に対する(√3)/2だから、
△ABC=3√2×(3√2×√3/2)×(1/2)
=(9√3)/2
∴(9√3)/2×OH×1/3=9/2
OH=(9/2)×2/(9√3)×3
=3/√3
=√3
0708132人目の素数さん
垢版 |
2019/02/05(火) 22:06:18.16ID:EnTe07Hf
A:ネーター環
B:Aの剰余環
としたとき、BはA加群としてネーターですが、
B加群としてもネーターになるのはなぜでしょうか?
0709132人目の素数さん
垢版 |
2019/02/05(火) 22:42:36.54ID:iOo7FxK6
>>697
b1〜bnが全て1の時題意は満たさないので1以外の数を含むとしてよい。
1以外の数が一つでもある場合1はあってもなくても変わらない(好きに指数を決められるから)ので全て取り除く。
よってb1〜bnは全て1でないとしてよい。

またb1〜bnは全て互いに素だから分解した時同じ素因数を一切含まない。
だからb1〜bnに合成数があって、異なる解(x,y)のセットを2組持つ場合、
bに含まれる合成数を全て素因数分解して、素数のみからなる新しいbのセットを作れば、それもまた異なる解(x,y)のセットを2組以上持つ。

だからまずb1〜bnは全て異なる素数として考える。
合成数を含む組については、あとでbのうちいくつかやそのべきを適当にかけ合わせて、xyの組が2つ以上構成できるものがあるか探せば良い。

こういうわけで結局問は、正整数aに対して、a^n-1とa^m-1が過不足なく互いに全く同じ素因数のセットを持つような、異なるn、mを持つものを探せということになる。
0710132人目の素数さん
垢版 |
2019/02/05(火) 22:43:08.75ID:iOo7FxK6
>>697
ここでa^n-1とa^m-1が同じ素因数セットcで割り切れるとする(素因数はこれのみでなくてもよい)。

n>mとすると、a^n-1=(a^m-1)*a^(n-m)+a^(n-m)-1なので、べきがn-mのaの累乗-1も同じセットcで割り切れると分かる。
これを繰り返してn-mkが正である限り好きなだけ小さい、セットcで割り切れるべきを得られる。
これは互除法と同じ。

つまりnとmが互いに素な場合、a^n-1とa^m-1の2数が共通の約数を持つなら、その約数はa-1の約数でないといけない。

つまり、a-1と全く同じ素因数を過不足なく持つaのべき-1を探せば、mnが互いに素なこういう解を探すのと同じことになる。

解となるnとmが互いに素じゃない場合は、その最大公約数をdとして、a^dを新しいa'に設定すればこれでべきが互いに素になるから同じ。

だからa-1のもつ素因数のみを、欠けずに全てもつa^n-1が存在するような自然数a、nの組はあるか?という問になる。
0711132人目の素数さん
垢版 |
2019/02/05(火) 22:46:50.31ID:iOo7FxK6
>>697
あるaに対してこのようなnで最小のものをNとする。
a-1を割り切る全ての素数のうち任意のものをbとする。bでない素数をqとする。
位数の性質から、Nの約数d全てについて、a^d-1を割り切るqは存在しない。
また全てのa^d-1はbで割り切れるので、これはa^d-1も問の解であることを意味する。

つまり、Nは素数でないといけない。
あとはこのNが仮にbに含まれるなら、
a^(N-1)≡1(modN)だから、Nが最小なものである以上N=2が確定して

この場合解はa=3,b=2しかない

Nがbに含まれることの証明がわからん
ギブアップ
0714132人目の素数さん
垢版 |
2019/02/06(水) 00:26:12.04ID:BEeFkno9
ジグモンディの定理って数オリで使えんの?
証明書くの無理じゃね?
こういう時に「写経」が役立つのか笑
0715132人目の素数さん
垢版 |
2019/02/06(水) 00:34:32.94ID:26Xz3TPl
>>713
記号は先の通りとして、f:A→Bを自然な準同型としたとき、B加群Mの部分加群と、fでの制限としてのA加群Mの部分加群が同じになるのでしょうか?
0716132人目の素数さん
垢版 |
2019/02/06(水) 00:35:45.83ID:+dJ9vhGm
>>709-711
>>710の最後の一文の前までは同意しますが、そこからは少し違うのではないかと思います
問題文は「対応するどの成分も一致しないようなものが2つ」なので、a^n-1はa-1と同じ素因数を含むだけではなく、さらに各素因数の冪が大きくないといけません
(a^n-1)/(a-1)がちょうどa-1と同じ素因数を含む、と言い換えることもできます
(この条件を無視すると
7^2-1=48=2^4*3^1
7-1=6=2^1*3^1
等、無数に解がでてきます)

Zsigmondyの定理というより一般的な結果を示した定理があるのですが、この問題はその一部を示す問題といえます
(定理についてはhttps://mathtrain.jp/7theoremの6番など)

但し定理の証明には普通は大学数学を用いるので、以下元の問題の高校生向けの証明を書いておきます

つづく
0717132人目の素数さん
垢版 |
2019/02/06(水) 00:36:37.80ID:+dJ9vhGm
つづき

簡単のため
a-1=p^k
a^n-1=p^l
とする(pは素数、k<l)
(素因数が複数の場合もほぼ同様にしてできる)

(a^n-1)/(a-1)=1+a+……+a^(n-1)=p^(l-k)
いま
a≡1 (mod p)
なので、上式をmod pで考えることにより
n≡0 (mod p)
を得る
(素因数が複数の場合は各素因数でmodをとることにより、nがa-1の任意の素因数で割り切れることが分かる)

a^p-1はa-1の倍数であり、またa^n-1の約数となるので、a-1と丁度同じ素因数を含む
よって
a^p-1=p^m
と書ける
(素因数が複数の場合は
a^(pq)-1=p^m*q^m' (m≧1,m'≧1)
などと書ける)

a=1+p^kを代入して
(1+p^k)^p -1=p^m
左辺を二項定理を用いて展開すると
p^(k+1)+(p(p-1)/2)*p^2k+……+p^kp
p>2のとき、各項のpの指数は
第1項:k+1
第2項以降:2k+1以上
となるので、
p^(k+1)*(1+p*(0でない整数))
と書ける
これは式の値がp^mであることに矛盾
(素因数が複数の場合は同様にして矛盾が導ける)

p=2のとき、左辺は
2^(k+1)+2^2k
これが2^mと書けるのはk+1=2kすなわちk=1のときのみ

以上より答えは
(a,b)=(3,2)
0718132人目の素数さん
垢版 |
2019/02/06(水) 00:39:05.17ID:bArcMKpl
>>714
さぁ?
懸賞問題なんだから何つ使ってもいいんじゃないの?
きちんと Reference つければ。
数オリのルールなんかどうでもいい。
0720132人目の素数さん
垢版 |
2019/02/06(水) 01:22:08.31ID:26Xz3TPl
>>719
確かに、Z→Rとして加群Rを考えればQは部分Z加群だけど部分R加群にはなりませんね
ありがとうございます
0721132人目の素数さん
垢版 |
2019/02/06(水) 01:58:06.54ID:/r47/vPf
東進の数学コンクールの質問をした者です。
皆様、ありがとうございます。
さすがに専門家はちがいますね。
自分も、皆様のようになれるように頑張ります。ありがとうございました。
0722132人目の素数さん
垢版 |
2019/02/06(水) 06:00:24.26ID:qKduZ/Zy
aを正の実定数とする。
定円C1と、正の範囲を動く実数b,rの値により変化する円C2があり、それぞれの方程式は

C1: x^2+(y-a)^2=a^2
C2: (x-b)^2+(y-r)^2=r^2

である。
C1とC2が外接しているとき、以下の問に答えよ。

(1)bをrの関数とみなし、rとaで表せ。

(2)円C3は方程式
C3: (x-c)^2+(y-d)^2=d^2
の形で表すことができ、0<c<bかつd>0を満たす。
またC3はC1と外接し、C2とも外接する。c,dをそれぞれrとaで表せ。

引き続き、C3は(2)の位置関係にある円とする。

(3)円C_i(i=1,2,3)の中心をP_iとおく。
△P_1P_2P_3の面積Sをrの関数とみてS=f(r)とおく。このとき以下の設問に答えよ。

(i)3つの線分P_1P_2、P_2P_3、P_3P_1の長さを比較し、大きくない順に並べよ。

(ii)(i)の順に並べた線分長をL≦M≦Nの形式で表したとき、Mをrとaで表せ。

(iii)rの関数T(r)をT(r)=(√3/4)*M^2と定める。g(r)=f(r)/T(r)とおくとき、g(r)/T(r)の増減を調べよ。
0726132人目の素数さん
垢版 |
2019/02/06(水) 13:32:22.27ID:uDKixBno
質問場所が異なれば誘導お願いします

ある数の平方根を求めて、さらにその数の平方根を求めて・・を繰り返すと
最終的にどうなるの?と子供に聞かれました。
a>0の時は1に近づくというのは理解して教えられたのですが
a<0の時にどうなるのかが分かりません。
どなたか中学数学レベルの人間にも分かるように教えていただけないでしょうか。
0729132人目の素数さん
垢版 |
2019/02/06(水) 13:43:10.68ID:uDKixBno
>>727
√a = aの1/2乗で、n回ルートにする(すみません変な言い方ですが)と
aの1/2n乗になる
でnがどんどん大きくなっていくと1/2nは0に近づく
aの0乗は1になるから、最終的にaの1/2n乗は1に近づく

こんな感じです。
そこでじゃあaがマイナスだったら?マイナスの平方根だとiってやつが付くんでしょ?
(虚数の概念は良く分かってないようです)と言われ困ってしまった次第です。
0732132人目の素数さん
垢版 |
2019/02/06(水) 14:04:18.80ID:HvvZB+CF
諦めて複素数と複素平面まで教える
で、2変数関数の収束について説明する

複素数を教えて√iが一体どんな数になるか調べさせてから
本当に説明の続きを知りたいのか確認する
0733132人目の素数さん
垢版 |
2019/02/06(水) 14:06:25.17ID:X+1KuCl/
>>729
iの平方根は±(1+i)/√2だし、証明はド・モアブルの定理を条件付で
できるけど、中学生に説明するにはなーって感じ
0734132人目の素数さん
垢版 |
2019/02/06(水) 14:08:19.99ID:BEeFkno9
>>726
負の数-aの√をn回取ったら、根は
|a|の2^n乗根 * (cos(mπ/2^n)+isin(mπ/2^n))  (mは奇数整数)になるんじゃないかな

お子さんが虚数について理解しているなら
ド・モアブルの公式の図形的意味(偏角の変化)について教えてあげたら分かると思う
https://i.imgur.com/IcD8s93.png
0735132人目の素数さん
垢版 |
2019/02/06(水) 19:02:05.77ID:uDKixBno
726です
ありがとうございます、まずは虚数・複素数と複素平面ですね!
恥ずかしながら親の私が複素平面をすっかり忘れてますのでそこからですが
何とか頑張って説明してみます。
0740132人目の素数さん
垢版 |
2019/02/07(木) 01:23:24.64ID:t/cmESHF
y=x+2/π*sin{π(y+x)/2}をy=(xのみの式)で表したいんですがどうすればいいのでしょうか
0743イナ ◆/7jUdUKiSM
垢版 |
2019/02/07(木) 05:44:14.73ID:0isiFr3R
/_/_/人人_/_/_/_
/_/_(_^_)/_/_/_
/_/_(__)/_/_/_
/_/_((^。^)/_/_/_
/_/_(_っ-┓_/_/_
/_/_◎゙┻υ◎゙/_/_/_/_/_/_/_/キコキコ……/_/_/_/_/_/_/_/_/_/_/_/_/_/_/_>>736
r=0
0744132人目の素数さん
垢版 |
2019/02/07(木) 06:29:24.24ID:HhHhpgu4
>>736
マルチポストいくない

式を変形すると
5^(2r)-2^(2r)=3^(2r)
指数部分が1のとき、元の式は
5-2=3 となり成り立つ
よって 2r=1, r=1/2

式の意味は経済の専門家にお聞きください
0747132人目の素数さん
垢版 |
2019/02/07(木) 12:52:42.94ID:x1nsMPK+
52枚のトランプを4枚ずつ13の山に適当に分ける。
それぞれの山から1枚ずつカードをうまく選ぶと
1から13までの13枚を必ず揃えることができるか?
0749132人目の素数さん
垢版 |
2019/02/07(木) 18:41:10.10ID:lIZxW39h
aを正の実定数とする。
定円C1と、正の範囲を動く実数b,rの値により変化する円C2があり、それぞれの方程式は

C1: x^2+(y-a)^2=a^2
C2: (x-b)^2+(y-r)^2=r^2

である。
C1とC2が外接しているとき、以下の問に答えよ。

(1)bをrの関数とみなし、rとaで表せ。

(2)円C3は方程式
C3: (x-c)^2+(y-d)^2=d^2
の形で表すことができ、0<c<bかつd>0を満たす。
またC3はC1と外接し、C2とも外接する。c,dをそれぞれrとaで表せ。

引き続き、C3は(2)の位置関係にある円とする。

(3)円C_i(i=1,2,3)の中心をP_iとおく。
△P_1P_2P_3の面積Sをrの関数とみてS=f(r)とおく。このとき以下の設問に答えよ。

(i)3つの線分P_1P_2、P_2P_3、P_3P_1の長さを比較し、大きくない順に並べよ。

(ii)(i)の順に並べた線分長をL≦M≦Nの形式で表したとき、Mをrとaで表せ。

(iii)rの関数T(r)をT(r)=(√3/4)*M^2と定める。g(r)=f(r)/T(r)とおくとき、g(r)の増減を調べよ。
0750132人目の素数さん
垢版 |
2019/02/07(木) 20:48:00.51ID:lIZxW39h
kを正整数とし、a={sin(π/k)}^2、b={cos(π/k)}^2とおく。

ある自然数iが存在して、n>iでは常に{(1/a^n)+(1/b^n)}(a+b)^nが整数となるようなkを全て決定せよ。
0751イナ ◆/7jUdUKiSM
垢版 |
2019/02/07(木) 22:29:43.21ID:0isiFr3R
>>747
そんなことできるわけないだろ。できるとしても、かなり偶然だよ。
0752132人目の素数さん
垢版 |
2019/02/07(木) 22:30:33.30ID:x1nsMPK+
>>748
>>747
ありがとうございます。二部グラフの完全マッチングの問題になるんですね
http://dopal.cs.uec.ac.jp/okamotoy/lect/2016/gn/lect07.pdf

13個の山に分けるときに3個と5個の山も何個か認めたりしたら13枚を揃えることが不可能になるのだろうかって
のが気になった。山の個数の条件を均等からどこまで緩めても必ず13枚揃うのだろうかなって
0754132人目の素数さん
垢版 |
2019/02/08(金) 03:46:37.13ID:rI5D0a2N
平面上に2点A,Bがあり、また同じ平面にある定円周C上に点Pがある。∠APB=60°で、△APBの垂心はC上にある。
P,A,Bの位置関係を述べよ。
0755132人目の素数さん
垢版 |
2019/02/08(金) 06:39:35.94ID:XBKD3Ybf
ほんのちょっと気になったどうでも良い質問ですが、ジェンガって理論上何段まで上に伸ばすことが出来ますか?w
0756132人目の素数さん
垢版 |
2019/02/08(金) 06:49:33.16ID:XBKD3Ybf
あ、自己解決しましたね
3個で1段をなしているとして、n段あるとするなら、ブロックの総個数=3n
で、理論上の最高段数は1段を1個で成しているときなので、3n段ですね
0757132人目の素数さん
垢版 |
2019/02/08(金) 07:42:27.72ID:3FxrZGOz
>>755-756
ジェンガ製造元の公式ルールでは
・ブロックの数は54個
・3個揃った段の下からしか取れない
・取ったブロックは最上段が埋まるように置く
という縛りがある

これに従うと、上から
2, 3, 1, 1, 1, ..., 1, 1
の51段が最高で、これ以上は取れない
0761132人目の素数さん
垢版 |
2019/02/08(金) 17:14:06.60ID:bYqYfmfZ
>>747
>>752
任意の2個の山の和は5以上
任意の3個の山の和は9以上
任意の4個の山の和は13以上
・・・であればいいので全部4以外には以下の9つ
の場合に必ず1から13までそろえることができる
(1,4,4,4,4,4,4,4,4,4,4,4,7)
(1,4,4,4,4,4,4,4,4,4,4,5,6)
(1,4,4,4,4,4,4,4,4,4,5,5,5)

(2,3,4,4,4,4,4,4,4,4,4,4,7)
(2,3,4,4,4,4,4,4,4,4,4,5,6)
(2,3,4,4,4,4,4,4,4,4,5,5,5)

(3,3,3,4,4,4,4,4,4,4,4,4,7)
(3,3,3,4,4,4,4,4,4,4,4,5,6)
(3,3,3,4,4,4,4,4,4,4,5,5,5)
0762132人目の素数さん
垢版 |
2019/02/08(金) 21:22:12.85ID:IDswJIPb
384=8!! 

53760=2(10!!)+12!!

8755200=8(12!!)+13(14!!)

1805690880=15(14!!)+12(16!!)+9(18!!)

471092428800=10(16!!)+15(18!!)+16(20!!)+5(22!!)
0763132人目の素数さん
垢版 |
2019/02/09(土) 11:41:49.82ID:jjMLHQvb
>>750
 X_n = 1/a^n + 1/b^n
とおくと
 X_{-1} = a+b = 1,
 X_0 = 2,
 X_1 = 1/ab,
 X_{n+1} = X_1・X_n - (1/ab)X_{n-1} = (1/ab)(X_n - X_{n-1}),
∴ 1/ab が整数なら十分。

1/ab = 1/[sin(π/k)cos(π/k)]^2 = [2/sin(2π/k)]^2 = 8/[1-cos(4π/k)],

k=4 のとき 1/ab = 4,
k=8 のとき 1/ab = 8,
k=12 のとき 1/ab = 16,
0764132人目の素数さん
垢版 |
2019/02/09(土) 19:39:19.27ID:0dlClZ38
https:// twitter.com/ tyu7ron
キチガイゴキブリ奇形底辺漫画家裕和は頸動脈切って自殺しろヒトモドキニホンザル南京虐殺奴隷実習生殺害民族はこの世から絶滅しろ
0765132人目の素数さん
垢版 |
2019/02/09(土) 21:05:51.17ID:rsEYW4zi
xy平面上で点P(x,y)のxとyはともに整数であり、かつx^2+y^2≦25を満たす。このような点Pは、シス個ある。また、原点をOとし、点Q(5,0)のとき、OPベクトル・OQベクトル≧5を満たす点Pはセソ個ある。
この問題のシスとセソに入る数字を教えてください。大学入試の問題です。
0767132人目の素数さん
垢版 |
2019/02/09(土) 21:40:06.69ID:rsEYW4zi
>>766
ありがとうございます!
81と35になりました。
0768132人目の素数さん
垢版 |
2019/02/09(土) 22:09:41.88ID:xWYHJ5Wt
384=8!! 

53760=2(10!!)+12!!

8755200=8(12!!)+13(14!!)

1805690880=15(14!!)+12(16!!)+9(18!!)

471092428800=10(16!!)+15(18!!)+16(20!!)+5(22!!)

153043438141440=4(18!!)+2(20!!)+3(26!!)

規則性を見つけてくれ〜(・ω・)ノ
0769132人目の素数さん
垢版 |
2019/02/10(日) 00:03:35.13ID:TaC8+83Q
>>740
f(x) = 6(x - sin(x)) は連続かつ単調増加なので、逆関数gが存在する。それを使うと・・・・
題意より
 f(π(y+x)/2) = 6πx,
 π(y+x)/2 = g(6πx),
 y = (2/π)g(6πx) - x,

* マクローリン級数は
 f(x) = x^3 - (1/20)x^5 + (1/840)x^7 - (1/60480)x^9 + (1/6652800)x^11 - ・・・・
 g(x) = x^{1/3} + (1/60)x + (1/1400)x^{5/3} + (1/25200)x^{7/3} + (43/17248000)x^3 + ・・・・
0770132人目の素数さん
垢版 |
2019/02/10(日) 08:50:52.99ID:Qxxyq1iE
縦の長さがa、横の長さがbの長方形Aを2枚重ね合わせ、その共通部分の面積をSとする。

(1)Sは区間(0,ab]の全ての値を取ることを示せ。

(2)0<t≦abなるtをとる。S=tとなるときの、2枚の長方形の位置関係を述べよ。
0771132人目の素数さん
垢版 |
2019/02/10(日) 13:54:21.02ID:UCOIZffv
曲線y=x^2と中心のx座標が正で半径3の円Cが点Aで接していて、かつ円Cがx軸と接している時、点Aの座標及び円Cの中心の座標を求めよ。
0777132人目の素数さん
垢版 |
2019/02/10(日) 18:33:55.31ID:q9DB31nQ
数物系で発散するには才能が相当控え目なヒスババアが居着いちゃってるとどうもねぇ
0778132人目の素数さん
垢版 |
2019/02/10(日) 18:38:59.21ID:OAdffEwe
>>776
無限大ってプラスマイナス無限大のことじゃないんですか?
色々と自分が勘違いしてると思うんですが、どこが間違っているかも分からなくて、でも 無限大 の言葉がずっと
0779132人目の素数さん
垢版 |
2019/02/10(日) 19:03:04.89ID:6Wx7Nodd
>>778
n÷0=∞と定義すると
∞×0=nになる
nは好きな数でいいわけだから
∞*0=1かつ∞*0=2だから1=2??

つまりこういう定義をすると、非常な不都合が起きる
「あるいは∞に限っては割り算の結果を掛けたらもとに戻るというような掛け算の逆演算は成立しない」とか「∞は実数ではない」とかわざわざ余計なことを言わざるを得ない

ならこの定義をしても普通得はないので
普通こういう定義は使わない
0780132人目の素数さん
垢版 |
2019/02/10(日) 19:20:54.35ID:Qxxyq1iE
以下の定義により無限数mを定める
・無限数mは唯一つのスカラー値で、どのような実数rをとってもm<rとはできない
・mに対し乗算と除算は定義される。任意の複素数cに対し
m*c=m、c/m=0

問題: mは実数か、実数でないか、mの構成に矛盾があるか、判定せよ。
0784132人目の素数さん
垢版 |
2019/02/10(日) 21:23:20.46ID:5Gjdw9V6
x+2y=5 x>0,y>0を満たす実数x,yがある。
1/x+2/yの最小値はなんですか?
0788132人目の素数さん
垢版 |
2019/02/10(日) 22:47:06.19ID:Qxxyq1iE
>>771
法線が中心を通ること、接線が共通なこと、法線方向ベクトルと接線方向ベクトルが直交すること、を使えば解けるけど
でも当たり前に見える他の方針でやると、解けない四次方程式が出てきて困る。
例えば中心を(a,3)とでも置いて、放物線と円の方程式がただ1つの解を持つよう係数を定める、とかだと途端に解くのが困難になる
何で?原理的には必ず解にたどり着くはずなのに、かたや易しい四次方程式、かたや難しい四次方程式
0789132人目の素数さん
垢版 |
2019/02/10(日) 23:04:06.88ID:23ZsHF1j
>>771
この問題では 半径3の円の中心を求めることになるのだけど、なぜ 3 が出て来たのか?
一般に半径:r とおくと点Aのy座標は 
((8r-1)+√(16r+1))/8 となる。
ここで r=3 となっていれば 16r+1 が 平方数49 になる。

出題者は、入試問題作成の過程で 16r+1  をみつけ、よし! と思ったのかな。
0790132人目の素数さん
垢版 |
2019/02/10(日) 23:49:16.37ID:5Gjdw9V6
>>785
8/5になったのですが、、、
0791132人目の素数さん
垢版 |
2019/02/11(月) 00:09:18.11ID:BmnGKho7
>>784
y=z とすると
 x+y+z = 5, x>0, y>0, z>0
コーシーより
 (x+y+z)(1/x + 1/y + 1/z) - 9 = (x-y)^2 /(xy) + (y-z)^2 /(yz) + (z-x)^2 /(zx) ≧ 0,
 1/x + 1/y + 1/z ≧ 9/(x+y+z),
0794132人目の素数さん
垢版 |
2019/02/11(月) 06:35:54.73ID:S2/xVXIG
>>793相加平均相乗平均で
1/x+2/y≧2√1/x+2/yでやったら8/5になりました。
0797132人目の素数さん
垢版 |
2019/02/11(月) 11:56:02.88ID:pCkc4px4
下記問題では条件としてr>0を書くべきですか?
それとも、「r<0でもよいとすると軌跡自体は、0≦Θ<πの制約が
あろうがなかろうが同じになるので、条件0≦Θ<πが示されている
以上はr>0の意味である」と解釈できるからr>0の条件は書かなくても
よいと考えるべきでしょうか。

***************************************
点Aの座標を(10,0),極Oと点Aをを結ぶ線分を直径とする円Cの
周上の任意の点をQとする。点Qにおける円Cの接線に極Oから
垂線OPを下ろし、点Pの座標を(r,Θ)とするとき、その軌跡の
極方程式を求めよ。ただし、0≦Θ<πとする。

答 r=5+5cosΘ
0798132人目の素数さん
垢版 |
2019/02/11(月) 12:31:45.35ID:pCkc4px4
訂正
r>0  → r≧0
0799132人目の素数さん
垢版 |
2019/02/11(月) 17:10:15.90ID:2TYI3qAS
■フィボナッチ数列(英: Fibonacci sequence)

0, 1, 1, 2, 3, 5, 8, 13, 21, 34, 55, 89, 144, 233, 377, 610,
987, 1597, 2584, 4181, 6765, 10946, …

Fn=(1/sqrt(5))(((1+sqrt(5))/2)^n-((1-sqrt(5))/2)^n)

√5が式に含まれているのに
整数が出力されるのはなぜですか?
0803132人目の素数さん
垢版 |
2019/02/11(月) 18:45:29.79ID:ksx4Ldb0
つ【共役な無理数 で検索】

一般に a+b√k と a−b√k の累乗
(a, b, k は有理数)を比較すると
・b が偶数次の項:
√k が偶数乗で消え、係数が一致
・b が奇数次の項:
√k を持ち、係数は符号が逆で一致
であるから
・和は √k の項が消えて有理数
・差は √k の項のみが残り √k と有理数の積
となる

以上は(a±b)^nの展開の一般項から示せる
0805イナ ◆/7jUdUKiSM
垢版 |
2019/02/11(月) 22:57:14.33ID:pzMyTfdL
>>784
1/x+2/y=tとおくと、
辺々xyを掛けて、
y+2x=xyt――@
x+2y=5より、x=5-2yを@に代入すると、
y+2(5-2y)=(5-2y)yt
2ty^2-(5t+3)y+10=0
y(y>0)が実数解をもつ条件より、
判別式D=(5t+3)^2-4・2t・10≧0
25t^2+30t+9-80t≧0
25t^2-50t+9≧0
(5t-9)(5t-1)≧0
t≦1/5,9/5≦t
x>0,y>0よりt>0だから、
0<t≦1/5,9/5≦t
t=1/5という極値が気になるが、最小値となると、
9/5が妥当かと。
∴最小値は9/5
0807132人目の素数さん
垢版 |
2019/02/11(月) 23:18:56.68ID:W88qMu+8
:::::::::::::::::::::::::::::::::::::::::::::::::::::::::::::::::
:::::::::::::::::
::::::::::             ____
       ∧_∧    ||....___ |
      ( ´・ω・)   .|| └ヾ::::| |
    ┌/:::::::  l     ||__/._/_|
    | |:: |::::  | ┌━━ー┷┷┐

 ――  ┼     |  ‐┼   7   !ヽ  |__ 〃 |   ヽ
        |/´`ヽ  .l   __|   {   |  /´     |    |
(___  /|    し |   (__jヽ  ヽ_ノ  (__   し
0808132人目の素数さん
垢版 |
2019/02/11(月) 23:42:36.82ID:8NLIJgEx
あっちこっち突っ込みどころがありそうだけど、y>0と自分で書いておきながら
いきなりそれを無視してるあたりはどうにもならんな
0811132人目の素数さん
垢版 |
2019/02/11(月) 23:54:44.08ID:tLP64gX5
>>706
ありがとうございます。
0812132人目の素数さん
垢版 |
2019/02/11(月) 23:58:32.76ID:2TYI3qAS
(1/sqrt(17))(((1+sqrt(17))/2)^n-((1-sqrt(17))/2)^n)

で整数が出力できるのはなぜですか?
0814132人目の素数さん
垢版 |
2019/02/12(火) 00:19:31.44ID:1Z+g14Fu
>>809
(1+√5)^7
= 7C7(1)^7 + 7C6(1)^6(√5)^1 + 7C5(1)^5(√5)^2
+ 7C4(1)^4(√5)^3 + 7C3(1)^3(√5)^4 + 7C2(1)^2(√5)^5
+ 7C1(1)^1(√5)^6 + 7C0(√5)^7
= 1 + 7√5 + 105 + 175√5 + 875 + 525√5 + 875 + 125√5
=1856 + 832√5
0815132人目の素数さん
垢版 |
2019/02/12(火) 00:30:22.26ID:1Z+g14Fu
>>814
64(29 + 13√5)
0816132人目の素数さん
垢版 |
2019/02/12(火) 00:53:19.13ID:iZooXX6T
.
       ∧__∧
      ( ´・ω・)∧∧l||l
       /⌒ ,つ⌒ヽ ) <>>813
       (___  (  __)  
"''"" "'゙''` '゙ ゙゚' ''' '' ''' ゚`
0817イナ ◆/7jUdUKiSM
垢版 |
2019/02/12(火) 01:06:15.42ID:JvKRqtd4
>>805
>>809
(1+√5)^7=(1+√5)^2・(1+√5)^2・(1+√5)^2・(1+√5)=(6+2√5)^2・(6+2√5)(1+√5)
=2^3・(3+√5)^2・(3+√5)(1+√5)
=8(8+6√5)(8+4√5)
=64(4+3√5)(2+√5)
=64(8+15+10√5)
=64(23+10√5)
0818132人目の素数さん
垢版 |
2019/02/12(火) 07:29:43.67ID:S8Mm4aqg
Mは複素数を要素とする有限集合である。また、MおよびMの要素について以下が成り立つ。

(a)1∈M
(b)0∉M
(c)α,βがMの要素なら、αβ∈M

(1)Mのどの要素もその絶対値が1であることを示せ。

(2)Mがn個の要素を持つとする。このとき、それら全ての要素を決定せよ。

(3)条件(c)を「α,βがMの相異なる要素なら、αβ∈M」と置き換えたとき、問題(1)(2)の結論がどのようになるか、述べよ。ただしMは2つ以上の要素を持つとする。
0820132人目の素数さん
垢版 |
2019/02/12(火) 14:22:13.29ID:3KKy2ADD
>>752

|S| >|N(S)| とすると矛盾が起きるのはなぜですか?
0821132人目の素数さん
垢版 |
2019/02/12(火) 14:23:36.73ID:3KKy2ADD
|S| 個のグループを N(S) に対応するランクのカードだけで作れない

のはなぜですか?
0822132人目の素数さん
垢版 |
2019/02/12(火) 14:47:04.41ID:3KKy2ADD
|N(S)| = k とおく。

k種類のカードを使って作れるグループの数は最大で、 4*k / 4 = k グループ。
0823132人目の素数さん
垢版 |
2019/02/12(火) 15:05:47.24ID:9Fb6po5i
>>761 よくわからんけど。その条件満たすのなら他にもあるんじゃね?
(2,4,4,4,4,4,4,4,4,4,4,4,6)
(3,4,4,4,4,4,4,4,4,4,4,4,5)
(3,3,4,4,4,4,4,4,4,4,4,4,6)
0824132人目の素数さん
垢版 |
2019/02/12(火) 15:39:00.96ID:iZooXX6T
(1/sqrt(7))(((1+sqrt(7))/2)^n-((1-sqrt(7))/2)^n)

整数が出力できないのはなぜ?
0825132人目の素数さん
垢版 |
2019/02/12(火) 17:14:38.10ID:0s23s8jr
例えば、n=3のときのように分母に因数2が残ることがあるから

√7のところが√5なら出来るのは(1+√5)/2と(1-√5)/2はそれらをα、βとおくとα-βが√5であり、またそれぞれx^2=x+1の解であるから

α^2=α+1を使ってα^2、α^3、α^4、α^5……をそれぞれαの1次式で表すように計算していくとα^n-β^nはmを整数としてm√5という形になることもmがフィボナッチ数になることもわかる
0826132人目の素数さん
垢版 |
2019/02/12(火) 18:30:18.24ID:iZooXX6T
60836834554675200=(20!!)+17(22!!)+15(24!!)+16(26!!)+12(28!!)+(30!!)

別の表記法はありますか?
0827248
垢版 |
2019/02/12(火) 20:40:55.47ID:19tRtIio
y = a * {sinh(bx)}^c
を満たすxとyのペア(x1, y1), (x2, y2), …, (xn, yn)について、bの値が0.001〜0.1くらいであれば、せいぜい数万回の繰り返しで解けることを確認しました。
しかし、0.1以上の場合と0.001以下の場合では、現実的ではないほどの繰り返しが必要な事がわかりました。

例えばb=1となるようなxとyのペア(ここではa=5E-5, c=0.08)
(10, 1.05275E-4)
(20, 2.34293E-4)
(30, 5.21428E-4)
(40, 1.16046E-3)
(50, 2.58265E-3)
を、例えば初期値b0=0.1とした場合、時間がかかりすぎます。短時間(できれば1分以内)で求めることはできるのでしょうか。
できるのであれば、その方法を教えてください。
0828132人目の素数さん
垢版 |
2019/02/12(火) 21:11:00.64ID:2T9vevM1
b*x 〜 10 位では、
sinh(b*x)≒exp(b*x)/2
だから
y=p*exp(q*x)
へ落とせばいいだけじゃ?
0829132人目の素数さん
垢版 |
2019/02/12(火) 21:36:37.17ID:xlh+oIvr
確率計算の問題
ガチャガチャの景品3つを全て揃えたい
そのうち1つだけレアがある。レアの確率はp
他の2つは同じ確率で出るものとする
1-p、1-p÷2、pを使って全て揃えるための平均回数を求める

すいませんこれわからなくて出来れば助力お願いしたいです。
0830132人目の素数さん
垢版 |
2019/02/12(火) 21:44:24.78ID:xlh+oIvr
私ではとても難しくどうやっても解ける気がしません…
どうしてもこの問題を答えを知りたいのでよろしくお願いします
0832132人目の素数さん
垢版 |
2019/02/12(火) 21:59:06.83ID:xlh+oIvr
このパターンの公式がなかなか見つからなくて…
0833132人目の素数さん
垢版 |
2019/02/12(火) 22:17:03.16ID:FOlktkgq
正多面体について教えてください。
「1つの面の頂点の数」×「面の数」=「辺の数」×2
が成り立つと思うのですが,これは何かの定理なのでしょうか?
オイラーの定理を確認しているうちに,上記の等式も成り立つことに気がついたのですが。
よろしくお願いいたします。
0836132人目の素数さん
垢版 |
2019/02/12(火) 22:52:40.77ID:9HF2uqFd
以下の問題お願いします。

1時間に600立方メートルの海水を8mの高さに揚げるポンプを駆動するには、いくらの出力の電動機を必要とするか。
ポンプや電動機の総合効率は75%とする。管内などにおける水の受ける抵抗は、水頭に換算して4mに相当し、海水の密度は1025kg/立方メートルとする。
0837132人目の素数さん
垢版 |
2019/02/12(火) 22:58:32.90ID:hknqZ4jQ
>>833
「1つの面の頂点の数」は「1つの面の辺の数」と同じ
「1つの面の辺の数」×「面の数」は辺を2度ずつ数えていることになるから「辺の数」×2
0838132人目の素数さん
垢版 |
2019/02/12(火) 23:24:13.66ID:xlh+oIvr
>>829
やはり過去スレ見てもわかりませんでした…
どなたかわかる方いらっしゃらないでしょうか?
0839132人目の素数さん
垢版 |
2019/02/12(火) 23:37:13.36ID:jChjzacN
n≥3とする。
集合S={a_1,a_2,...,a_n}はn個の複素数を要素とし、どの2つの要素も相異なる。
Sは以下の3条件を同時に満たすと仮定する。

(a) 0∉S, 1∈S
(b) Sの3つの要素α,β,γをどのように選んでも(重複も可とする)、αβγ∈S
(c) Sの相異なる2つの要素α,βをどのように選んでも、αβ∉S

この3条件を同時に満たすSは存在するか。存在するならばnを用いて例を挙げ、存在しないならばその理由を述べよ。
0843132人目の素数さん
垢版 |
2019/02/13(水) 00:15:30.50ID:iJ6qT/QN
384 
53760 
8755200 
1805690880 
471092428800 
153043438141440 
60836834554675200

規則性を見つけてくれ〜(・ω・)ノ
0844132人目の素数さん
垢版 |
2019/02/13(水) 00:19:50.59ID:ot153MZ+
>>829
すいません。これってもうダメでしょうか…
0845311
垢版 |
2019/02/13(水) 00:23:59.07ID:Yp2uIwP1
>>829
>レアの確率はp
>他の2つは同じ確率で出るものとする

他の2つの確率が何と同じなの?
0846132人目の素数さん
垢版 |
2019/02/13(水) 00:42:10.38ID:KX7RYeRa
サイコロをn回振ったとき、出目の積がkの倍数になる確率をp(k、n)とする。

(1)kが7以上の素数の時、pを求めよ。
(2)k=n^nのとき、pを求めよ
(3)k=2^nのとき、pを求めよ

友達に出されました。(1)が0になりそうなのはわかりますが記述できないよ
0847132人目の素数さん
垢版 |
2019/02/13(水) 00:59:25.88ID:ts8JPILN
>>812 >>824

α+β = 1 αβ = μ より
特性多項式は tt-t+μ
漸化式は a_{n+1} = a_n - μa_{n-1},
となる。
√17 のとき μ = (1-17)/4 = -4,
√7 のとき μ = (1-7)/4 = -3/2,
√5 のとき μ = (1-5)/4 = -1,

ただし a_0=0, a_1=a_2=1 とした。
0848132人目の素数さん
垢版 |
2019/02/13(水) 01:06:51.31ID:+JuxKXbq
>>845
同じ確率とはpが10%の場合残り2つはどちらも45%で出ると言うことです
書き方が悪かったですね…すいません…
0849132人目の素数さん
垢版 |
2019/02/13(水) 01:08:40.10ID:+JuxKXbq
1-pは1回目のレア以外が当たる確率1-2÷pはもう一個のレア以外のものが当たる確率です
0851132人目の素数さん
垢版 |
2019/02/13(水) 01:21:56.88ID:QmdHK2/q
サイコロをn回振ったとき、出目の積がkの倍数になる確率をp(k、n)とする。

(1)kが7以上の素数の時、pを求めよ。
(2)k=n^nのとき、pを求めよ
(3)k=2^nのとき、pを求めよ

友達に出されました。(1)が0になりそうなのはわかりますが記述できないよ
0853132人目の素数さん
垢版 |
2019/02/13(水) 01:30:35.66ID:JiM/wUpa
どう考えてもわかりませんでした。よろしくお願いします。

「正方形は全ての角度に面がある。正論は「わたしの側の面」でしかなく、 人の数だけ正しい面が存在する。正しいを一つにしてる価値こそ間違いで正しいは存在しない。」

※とある芸人さんからの命題です
0855132人目の素数さん
垢版 |
2019/02/13(水) 03:58:50.41ID:LQ9m5hae
>>846
(1)kは7以上の素数だからkしか素因数を持たない。1〜6の出目で作れる素因数は2,3,5だから、これらがkとなることはない。
(2)サイコロをn回振ったときn^nの倍数⇒n=1,2,3,4,5,6,7以上,で場合分けが簡単
n=1⇒p=1
n=2⇒4の倍数⇒4が1回以上か、2か6が2回
n=3⇒27の倍数⇒3または6が合計3回
n=4⇒4回振って4^4の倍数になるには4が4連続で出るしかない
n=5⇒5が5連続しかありえない
n=6⇒6が6連続しかありえない
n≥7⇒どんなに頑張ってサイコロをn回振ってもn^nには届かないのでp=0
(3)2がa回出る⇒2^a。4は2^2だから、4がb回⇒(2^2)^b=2^(2b)。
したがって2がa回かつ4がb回出ると、2^(a+2b)の倍数が作れる。
あとはa≥0,b≥0,a+2b≥n,0<a+b≤n のもとで格子点の個数計算
0856132人目の素数さん
垢版 |
2019/02/13(水) 07:48:17.96ID:QmdHK2/q
>>855
ありがとうございます!
0857132人目の素数さん
垢版 |
2019/02/13(水) 09:57:00.44ID:+JuxKXbq
>>852
1-p、1-p÷2、pを使って式を作れないでしょうか?
0859132人目の素数さん
垢版 |
2019/02/13(水) 10:13:59.99ID:+JuxKXbq
>>858
すいません…それができないんです…
0860132人目の素数さん
垢版 |
2019/02/13(水) 12:31:19.30ID:UrPrPI7F
代入ができない奴に式を教えても意味ないなw
0861132人目の素数さん
垢版 |
2019/02/13(水) 13:13:49.37ID:nmTqlE7B
組み合わせ爆発ってよくいうけど指数関数a^xのような感じでかくとどうなるのでしょうか?
x^x ? a^x^2 ?
0865132人目の素数さん
垢版 |
2019/02/13(水) 17:17:45.39ID:Q3wR5xIZ
BottとTuの微分形式と代数トポロジーでは、
C={R^n上のC^∞関数全体}、R^n上の外積代数AをそれぞれR代数とみて、
R^nの微分形式全体をCテンソルA(R代数として)と定義しているのですが、
これの元が一意的にΣ Cの元*Aの元 というように書けるのはなぜですか?
0866132人目の素数さん
垢版 |
2019/02/13(水) 17:58:23.78ID:/RvSNZDt
>>865
その表示では一意的ではありません
Bott-Tuで書かれているのは、Ω*の基底を用いた表示が一意的である、ということです
この時点で読みづらいようであれば、Bott-Tuを読む前にまず可換環論を勉強することを勧めます
0867132人目の素数さん
垢版 |
2019/02/13(水) 18:08:41.88ID:ou/t+xEn
>>866
ありがとうございます
可換環論からやろうと思います

因みにΩ*の基底での表示が一意とは具体的にどういうことでしょうか?
R代数Ω*をC^∞(R^n)で係数拡大して考えるということですか?
0868132人目の素数さん
垢版 |
2019/02/13(水) 19:40:57.89ID:AQNfrj6o
連続な数の和が積分で、離散数の和が総和で対応してると思うんですけど
そのときに微分に対応するもの(離散数の微分みたいな、ただの差みたいなものだけど)は何て名前がついてるのでしょうか?
0872132人目の素数さん
垢版 |
2019/02/13(水) 23:11:19.91ID:LQ9m5hae
n次多項式f(x)で、
∫[a to b] f(x) dx = {1/(b-a)}*Σ[for i=a to b] f(i)
を満たす自然数a<bがとれるものを求めよ。
0873132人目の素数さん
垢版 |
2019/02/14(木) 00:33:15.73ID:jufxHPgM
横4行、縦4列、対角線2つの4つの数値の合計は
すべて34になるのはなぜ?

16 3 2 13
5 10 11 8
9  6 7 12
4 15 14 1
0874132人目の素数さん
垢版 |
2019/02/14(木) 06:38:11.69ID:/7nNoY1N
>>872
自作問題かな?
積分と対応する近似を意図しているようですが
右辺は自然数を使った 1 ごとの長方形近似なので
補正の 1/(b-a) は不要でしょう
シグマの上端も b-1 とすべきかも

このままでは
規則的でない無数の解がある
としかいえません
0875132人目の素数さん
垢版 |
2019/02/14(木) 06:49:57.68ID:/7nNoY1N
>>873
【魔方陣】で検索

小学生の宿題や自由研究であれば
自分の考えと予想を書いてから
作り方を調べて書き、
この作り方にしたがえば性質をみたす
並べ方になる、といえばよいでしょう
0876イナ ◆/7jUdUKiSM
垢版 |
2019/02/14(木) 11:56:00.70ID:EnaLKZon
>>873
1から16までの数を縦横4つずつの升目にたまたま合計が同じになるように並べたから。

ちなみにその合計は、
(1+16)(1/2)×4=34
∴示された。
0877132人目の素数さん
垢版 |
2019/02/14(木) 15:04:34.66ID:keXOjexL
>>855
答え合わせしたら(3)が違うと言われました。「6」も2を素因数として持ってる だそうです。でも、6が出た回数がc回だとすると、格子点の計算が3次元になるので求め方がわかりません
0878132人目の素数さん
垢版 |
2019/02/14(木) 15:33:23.97ID:jufxHPgM
n番目のフィボナッチ数をFnで表すと

F(0)=0,F(1)=1,

F(n+2)=F(n)+F(n+1),(n≧0)

これの一般項は

Fn=(1/sqrt(5))(((1+sqrt(5))/2)^n-((1-sqrt(5))/2)^n)

同じように

a(n)=a(n-1)+a(n-2)/((2n-1)(2n-3)),a(1)=0,a(2)=1/3

の一般項は何ですか?
0879132人目の素数さん
垢版 |
2019/02/14(木) 15:58:31.24ID:jDSHxGHh
>>877
すまん6忘れてた
3次元の格子点の求め方は積分と同じ
kを整数として平面z=kで切ると、切り口は三角形とか四角形になる
つまり2次元の場合に帰着できる
切る平面はx=kとかy=kとかでもいいな

あとはz=kの格子点の数S_kをΣ(k=d,...,e)S_kみたいな感じ
必要な範囲dからeまでS_kを足し合わせる
断面積の積分が体積になるのと同じ原理
0880132人目の素数さん
垢版 |
2019/02/14(木) 18:36:08.70ID:jDSHxGHh
nを正の整数とする。

(1)a,b,cがそれぞれ正の整数値をとるとき、a(b+c)≤nのもとで、a+b+cのとる最大値をnで表せ。

(2)(1)で求めた最大値をM(n)とするとき、M(n)とnの大小を比較せよ。

(3)不等式M(n)≤nを満たす(a,b,c)は何組あるか。
0881132人目の素数さん
垢版 |
2019/02/14(木) 20:44:54.78ID:keXOjexL
>>880
a=1、b+c=nとなるようにとれば、nが幾つでもM(n)>nがなりたつよ
0885132人目の素数さん
垢版 |
2019/02/15(金) 09:39:35.80ID:RA/qPohE
1:1.7:2
100:x:113
の時にxの値の求め方教えてください。
0886132人目の素数さん
垢版 |
2019/02/15(金) 10:00:54.41ID:yPWzmNsO
x^4+ax^3-(b-2)x^2-ax+1=0
xが0でないすべての実数をとるとき、上の等式をみたす実数(a,b)を全て求めよ。
0887イナ ◆/7jUdUKiSM
垢版 |
2019/02/15(金) 10:18:08.68ID:q0kuQ25d
>>876
>>885
109.1じゃないの?
13を七三に分けろってことでしょ?
100+13×0.7=100+9.1
=109.1
あってる。
0889132人目の素数さん
垢版 |
2019/02/15(金) 11:22:51.14ID:LNR5nYYM
>>886
両辺を x^2 で割って t = x - 1/x とおけば t はすべての実数値をとりうる。
t の2次方程式に対して判別式≧ 0
0891132人目の素数さん
垢版 |
2019/02/15(金) 14:03:51.84ID:af8RfF+1
>>873
 まず各数から1を引くと 0〜15 になる。

 これを4進数で表わせば、各桁の数字が

 4^1   4^0
 ----  ----
 3003  3210 
 1221  0123
 2112  0123
 0330  3210
 ----  ----

 これを2進数で表わせば、各桁の数字が

 2^3   2^2   2^1   2^0
 ----  ----  ----  ----
 1001  1001  1100  1010
 0110  1001  0011  0101
 1001  0110  0011  0101
 0110  0110  1100  1010
 ----  ----  ----  ----
0892イナ ◆/7jUdUKiSM
垢版 |
2019/02/15(金) 14:31:16.22ID:q0kuQ25d
>>887
>>886なるほど、そんなやり方があるのかぁ。
F(x)=x^4+ax^3-(b-2)x^2-ax+1=0とおくと、
F'(x)=4x^3+3ax^2-2(b-2)x-a=0
xの実数解c、d、e(c<d<e)について、F(c)<0、F(d)>0、F(e)<0だから、
F'(c)=4c^3+3ac^2-2(b-2)c-a=0
c^3=-(3/4)ac^2+{(b-2)/2}c+a/4――@
F'(d)=4d^3+3ad^2-2(b-2)d-a=0
d^3=-(3/4)ad^2+{(b-2)/2}d+a/4――A
F'(e)=4e^3+3ae^2-2(b-2)e-a=0
e^3=-(3/4)ae^2+{(b-2)/2}e+a/4――B
F(c)=c^4+ac^3-(b-2)c^2-ac+1>0――C
F(d)=d^4+ad^3-(b-2)d^2-ad+1<0――D
F(e)=e^4+ae^3-(b-2)e^2-ae+1>0――E
@をCに代入すると、
-(3/4)a(c+a)c^2+{(b-2)/2}(c+a)c+(c+a)a/4-(b-2)c^2-ac+1>0
-(3/4)ac^3+a^2・c^2+{(b-2)/2}(c+a)c+(c+a)a/4-(b-2)c^2-ac+1>0
c^3の項にさらに@を代入し、
-(3/4)a[-(3/4)ac^2+{(b-2)/2}c+a/4]+a^2・c^2+{(b-2)/2}(c+a)c+(c+a)a/4-(b-2)c^2-ac+1>0
(9/16)a^2・c^2-3{(b-2)/8}c-3a/16+a^2・c^2+{(b-2)/2}(c+a)c+(c+a)a/4-(b-2)c^2-ac+1>0
(9/16)a^2・c^2-3(b-2)c/8-3a/16+a^2・c^2+(b-2)c^2/2+a(b-2)c/2+ac/4+a^2/4-(b-2)c^2-ac+1>0
{(25/16)a^2-(b-2)/2}c^2-3(b-2)c/8-3a/16+a(b-2)c/2+ac/4+a^2/4-ac+1>0
{(25/16)a^2-(b-2)/2}c^2-3(b-2)c/8+a(b-2)c/2+ac/4-ac+a^2/4-3a/16+1>0
{(25/16)a^2-(b-2)/2}c^2-{(3/8+a/2)(b-2)-3a/4}c+a^2/4-3a/16+1>0
同様に、
{(25/16)a^2-(b-2)/2}d^2-{(3/8+a/2)(b-2)-3a/4}d+a^2/4-3a/16+1<0
{(25/16)a^2-(b-2)/2}e^2-{(3/8+a/2)(b-2)-3a/4}e+a^2/4-3a/16+1>0
一回目の考慮時間に入る。
0896132人目の素数さん
垢版 |
2019/02/15(金) 17:28:43.62ID:ax+2oDDv
>>894
何も知らないくずの分際で
クソみてえな友人が出した問題溶けねえとか言って
死にたいか
0898132人目の素数さん
垢版 |
2019/02/15(金) 20:31:34.22ID:7IBHz4en
>>889
あってますね、
s=x+1/x,t=x-1/xとおいて、さらにs,tが双曲線x^2-y^2=4上のx>0の点をみたすから三角関数でおいて…っていうのをやりたくて作ったんですが、たしかにあなたのやり方ででできますね…、お見事です。
0904132人目の素数さん
垢版 |
2019/02/17(日) 11:41:13.50ID:I6JOaRHH
ファイバー束や被覆空間で射影が全射じゃないが一般的に興味のある例ってありますか?
0906132人目の素数さん
垢版 |
2019/02/17(日) 19:40:58.79ID:CR4pm/Gs
P1st Q1st even
[1,] 0 0 1
[2,] 4 5 6
[3,] 26 27 13
[4,] 84 83 23
[5,] 203 197 35
[6,] 413 398 50
[7,] 751 722 67
[8,] 1259 1210 87
[9,] 1986 1910 109
[10,] 2986 2875 134
[11,] 4320 4165 161
[12,] 6054 5845 191
[13,] 8261 7987 223
[14,] 11019 10668 258
[15,] 14413 13972 295
[16,] 18533 17988 335
[17,] 23476 22812 377
[18,] 29344 28545 422
[19,] 36246 35295 469
[20,] 44296 43175 519
[21,] 53615 52305 571
[22,] 64329 62810 626
[23,] 76571 74822 683
[24,] 90479 88478 743
[25,] 106198 103922 805
[26,] 123878 121303 870
[27,] 143676 140777 937
[28,] 165754 162505 1007
[29,] 190281 186655 1079
[30,] 217431 213400 1154

宝3個のデータ表を作ってくれ〜(・ω・)ノ
0907132人目の素数さん
垢版 |
2019/02/17(日) 22:05:54.21ID:mw4dgZz0
nを自然数とする。
0<a(b+c)≤n
0<a+b+2c≤n
0<b^3≤n
をすべて満たす自然数(a,b,c)の組の個数をf(n)とおく。
例えばf(1)=f(2)=f(3)=0,f(4)=1,f(5)=2である。このときf(2019)を求めよ。
0908132人目の素数さん
垢版 |
2019/02/18(月) 12:24:23.16ID:mNEbP/XQ
fは実区間[a,b)からバナッハ空間Fへの連続写像であり、右側微分可能
右側微分係数は f_r’(x)のように表し
g(x) := norm( f(x) ) とします

この時、g は右側微分可能であり
| g_r’(x) | ≦ norm(f_r’(x)) (for x∈[a,b))
である事を示せ.

|g(x+h) - g(x)| = | norm( f(x)+ f_r’(x)h +o(h) ) - norm(f(x)) |
≦ norm(f(x)+ f_r’(x)h +o(h) - f(x))
≦ norm(f_r’(x)) h + norm(o(h))
∴ 0 ≦ sup[h→+0] | {g(x+h) - g(x)}/h | ≦ norm(f_r’(x))
ここまでは分かったのですが、
sup[h→+0] {g(x+h) - g(x)}/h = inf[h→+0] {g(x+h) - g(x)}/h
この示し方が分かりません.
0910132人目の素数さん
垢版 |
2019/02/18(月) 21:51:20.77ID:mNEbP/XQ
>>909
ありがとう。ちょっと違ったけど参考になりました。

g(x) = norm(f(x)) = | f | のように書くことにする.
任意の ε > 0 に対して k, h (ε> k > h > 0) が存在して
(sup... -ε) - (inf... + ε)
< {g(x+h) - g(x)}/h - {g(x+k) - g(x)}/k
= { g(x+h) k - g(x+k)h - g(x)(k - h) }/(hk)
= { | f + f’ h + o(h) | k - | f + f’ k + o(k) | h - | f | (k-h) }/(hk)
≦ { | f(x)(k - h) + o(h)k - o(k)h) | + | f | (h-k) }/(hk)
≦ |o(h)k - o(k)h| / (hk)
≦ |o(h)|/h + |o(k)|/k → 0 (as ε → +0)
∴ inf ... = sup ... = g_r’(x)
(inf... または sup... が非有界の場合も同様にして示せる)
0911132人目の素数さん
垢版 |
2019/02/19(火) 01:28:59.74ID:T+Kw+bmH
有界性は示されてるので
> (inf... または sup... が非有界の場合も同様にして示せる)
これはいらんかったわ
0913132人目の素数さん
垢版 |
2019/02/19(火) 05:23:01.88ID:OL3kpF0J
>>907 >>912

0 < b^3 ≦ 2019 より b = 1, 2, ・・・・, 12

b=1  12653
b=2  11645
b=3  10972
b=4  10469
b=5  10066
b=6   9731
b=7   9443
b=8   9192
b=9   8968
b=10  8768
b=11  8585
b=12  8418

f(2019) = 118910
0914132人目の素数さん
垢版 |
2019/02/19(火) 16:09:53.93ID:IDFPWNBX
3c2 5c2 7c2 9c2 11c2 13c2 15c2 17c2 の出力である

3 10 21 36 55 78 105 136 の総和を

Sumとchooseで表すとどうなりますか?
0918132人目の素数さん
垢版 |
2019/02/19(火) 18:38:00.24ID:gj7xTt1h
>>916
答え貼られたけどいちおうヒント

a_n が急激に 1 に近づくことに着目して
b_n = 1 - a_n
c_n = 1 / b_n
とおくと
c_(n+1) = 6(c_n)^2 + 6(c_n) + 2, c_1 = 2
と、分数を含まない漸化式にできる
0920132人目の素数さん
垢版 |
2019/02/19(火) 20:18:50.59ID:KeYirXLR
>>918
すみません、もう少しヒントほしいです
b_n = 1 - a_nを使えるように式を変形するところまでいけません
0922132人目の素数さん
垢版 |
2019/02/19(火) 21:24:57.32ID:UyPqaeuU
∫[0→π]dx∫[x→π]{y(sin(y))/y}dy
この累次積分を積分順序を交換することにより求めよ
この問題がどうやって積分したらいいかわかりません
0924132人目の素数さん
垢版 |
2019/02/20(水) 04:40:59.33ID:6LNxfr5k
積分領域をxy平面上で表すと「y軸と直線y=xと直線y=πで囲まれた領域」
この領域が等しくなるように積分順序を入れ替えると
∫[0→π]dx∫[x→π]{x(sin(y))/y}dy
= ∫[0→π]dy∫[0→y]{x(sin(y))/y}dx
後は計算するだけ
0925132人目の素数さん
垢版 |
2019/02/20(水) 09:30:27.81ID:JXz7qxEr
n^2+1とn^4+1とn^6+1がいずれも素数となるような自然数nを全て求めよ。
0926132人目の素数さん
垢版 |
2019/02/20(水) 09:31:42.80ID:FXvgw2du
微分可能多様体のある点における積分曲線の全体は、その点における接ベクトルの全体と一対一に対応しますか?
0929132人目の素数さん
垢版 |
2019/02/20(水) 21:02:52.54ID:Vg+FahS5
The lesson contained in Russell's Paradox and other similar examples is
that by merely defining a set we do not prove its existence.

この英文の意味は以下のような意味だと思いますが、この「by」の意味は何ですか?

ラッセルのパラドックスや他の似た例に含まれる教訓は、ただ単に集合を定義するだけで、その存在を証明しないということである。
0930132人目の素数さん
垢版 |
2019/02/20(水) 22:44:18.03ID:u2k/hpyq
>>929

ただ単に集合を定義しただけでは その存在を証明したことにはならない(そんな集合無いよ、ってこともある)、ということである。
0931132人目の素数さん
垢版 |
2019/02/21(木) 00:02:41.68ID:xX53gRp1
ものすごい初歩的な疑問で申し訳ないんだけど
超準数って正則性公理によって作れない筈の無限下降列が存在してしまうように思えるんだけど
どういうことなの?
0933132人目の素数さん
垢版 |
2019/02/21(木) 13:31:34.36ID:6M49srHc
[0,1]を定義域とする関数
f(x)=x(0≤x≤1/2), 1-x(1/2<x≤1)
を考える。
またxの関数g[n](x)を、
g[0](x)=f(x)
g[n+1](x)=f(g[n](x))
により定める。

(1)初期値x=1/4に対して、g[4](1/4)を求めよ。答えのみで良い。

(2)初期値x=a(0≤a≤1)に対して、g[n](a)を求めよ。またg[n](a)はn→∞としたときに収束するかどうか述べよ。
0934132人目の素数さん
垢版 |
2019/02/21(木) 18:44:17.01ID:aQlKHyg4
高専2年 重積分
立式はできましたが上手い解き方が思いつきませんでした。
極座標変換するのかなと思いましたが、上手く解けませんでした。解説お願いします。
https://i.imgur.com/bwrG2gY.jpg
0935132人目の素数さん
垢版 |
2019/02/21(木) 19:10:33.40ID:Zd/Ey4nq
齋藤正彦著『数学の基礎』を読んでいます。


R を集合 A 上の同値関係とする。互いに R 同値な A の元を全部あつめると A の部分集合ができる。


「互いに R 同値な A の元を全部あつめる」ってどういうことですか?

集合の記法で「互いに R 同値な A の元を全部あつめ」た部分集合を書くとどうなりますか?
0939132人目の素数さん
垢版 |
2019/02/21(木) 20:47:25.60ID:6C2nKqLD
>>935
一字一句そのままなら、齋藤の書き方がおかしい。

任意の元 a (∈A) について、 a とR 同値な A の元を全部あつめると A の部分集合 [a] ができる.
つまり [a] = { x ; x∈A, x〜a }
またAのR同値類は A/R = { [x] ; x ∈ A } と表せる.
0940132人目の素数さん
垢版 |
2019/02/21(木) 21:11:25.05ID:Zd/Ey4nq
>>939

他の本にも書いてあるその書き方なら分かりやすいですよね。

一字一句そのままです。
0941132人目の素数さん
垢版 |
2019/02/21(木) 21:14:59.49ID:Zd/Ey4nq
>>937

「互いに R 同値な A の元を全部あつめる」ってどういう操作なんですか?

「互いに R 同値な A の元を全部あつめる」というのがどういう操作なのか説明しないのはおかしいですよね。

質問は齋藤正彦さんの記述はどういうことを言っているのか?ということです。
0942132人目の素数さん
垢版 |
2019/02/21(木) 22:34:24.74ID:WGwuylYb
複利計算していて以下のような結果になりました
1.1^50 = 117.39
1.05^100 =131.50
1.025^200 =139.56
1.0125^400 =143.88
1.00625^800 =146.12
・・・
limx→∞(1+0.1*(1/2)^x)^(50*2^x)=148.413・・・=e^5
なんでeの5乗なのこれ
0943132人目の素数さん
垢版 |
2019/02/21(木) 23:27:31.48ID:gd1NFM30
いつもの松坂君に続々と大量に釣られてやがる
これはまた粘着のモチベ与えちゃったな
0946132人目の素数さん
垢版 |
2019/02/22(金) 06:44:52.04ID:J9u5inRW
https://imgur.com/HlTUQmh.jpg

↑の赤い線を引いた箇所が分かりません。
0947132人目の素数さん
垢版 |
2019/02/22(金) 08:54:13.46ID:2w9rS1IK
わからないんですね
0948132人目の素数さん
垢版 |
2019/02/22(金) 09:45:39.78ID:pFJ0+U3Y
神界より上の世界より上の世界より上の世界より・・・・・(これが無限に続く。)
究極の本当にもうこの上ない絶頂世界はありますか?
あったとしたらそれはどんな世界ですか?
また、そこに行くにはどうすれば良いのでしょうか?
また、その世界は我々が存在しているこの世界と繋がっていますか?
0951132人目の素数さん
垢版 |
2019/02/22(金) 11:44:15.34ID:Xz/D0dBy
上方てどっちやねん
0952132人目の素数さん
垢版 |
2019/02/22(金) 15:14:37.27ID:M0+stptT
[0,1]を定義域とする関数
f(x)=4x(0≤x≤1/2), 4-4x(1/2<x≤1)
を考える。
またxの関数g[n](x)を、
g[0](x)=f(x)
g[n+1](x)=f(g[n](x))
により定める。

(1)初期値x=1/4に対して、g[4](1/4)を求めよ。答えのみで良い。

(2)初期値x=a(0≤a≤1)に対して、g[n](a)を求めよ。またg[n](a)はn→∞としたときに収束するかどうか述べよ。
0954132人目の素数さん
垢版 |
2019/02/23(土) 02:16:47.01ID:7Jcl7DYV
>>948
俺の次に絶頂
0955132人目の素数さん
垢版 |
2019/02/23(土) 04:12:52.50ID:mliwcdpK
>>954
真面目に教えてください。お願いします。
0957132人目の素数さん
垢版 |
2019/02/23(土) 15:48:42.57ID:A9cGGQwo
[0,1]を定義域とする関数
f(x)=2x(0≤x≤1), 4-2x(1<x≤2)
を考える。
またxの関数g[n](x)を、
g[0](x)=f(x)
g[n+1](x)=f(g[n](x))
により定める。

(1)初期値x=1/4に対して、g[4](1/4)を求めよ。答えのみで良い。

(2)初期値x=a(0≤a≤2)に対して、g[n](a)がn→∞としたときに収束するかどうか述べよ。
0959132人目の素数さん
垢版 |
2019/02/23(土) 17:36:24.37ID:Z72qpGXM
y''-4y'+3y=(8/x^3)+(13/x^2)+9logx
の解き方がわかりません
特にlog xをどうやって未定係数を決めればいいか分かりにくいません
0960132人目の素数さん
垢版 |
2019/02/23(土) 18:03:24.25ID:M9p9l/x6
y = a・log(x) + b/x を与式に入れて、係数を比べる。
a=3, b=4.
あとは、斉次方程式の解 c_1・e^x + c_3・e^(3x) をたす。
0962132人目の素数さん
垢版 |
2019/02/24(日) 01:47:43.17ID:i2zd9bcI
>>934 >>950

 xy平面より上方にあり,放物面 y^2 + z^2 = 4ax と円柱 x^2 + y^2 = 2ax とで囲まれる部分の体積を求めよ。
ただし,a>0 とする。
0966132人目の素数さん
垢版 |
2019/02/24(日) 02:37:15.27ID:zws9pgVd
完成版です

[0,2]を定義域とする関数
f(x)=2x(0≤x≤1), 4-2x(1<x≤2)
を考える。
またxの関数g[n](x)を、
g[0](x)=f(x)
g[n+1](x)=f(g[n](x))
により定める。

(1)初期値x=1/3に対して、g[4](1/3)を求めよ。答えのみで良い。

(2)初期値x=a(0≤a≤2)に対して、g[n](a)がn→∞としたときに収束するかどうか述べよ。
0967132人目の素数さん
垢版 |
2019/02/24(日) 02:50:45.97ID:cjGQ9Ao2
最高裁長官はどれくらい数学ができますか?
0969132人目の素数さん
垢版 |
2019/02/24(日) 05:45:58.56ID:11wA0XDA
>>964
ありがとうございます。
私も同じように立式しましたが、どうすれば解けるのでしょうか?
0970132人目の素数さん
垢版 |
2019/02/24(日) 07:58:34.45ID:tbaAoo1o
V = 2∫ [x:0,2a]dx ∫ [y:0,√(2ax-xx)]dy √(4ax-yy)
= 2∫ dx ∫ d(y/√(4ax)) 4ax √( 1 - (y/√(4ax))^2 )
= 4a ∫ dx x { asin(s) + s √(1-ss) } (∵ @A)
= 64 a^3 ∫ [s:0,1/√2]ds (s-2s^3){ asin(s) + s√(1-ss) }
= 32 a^3 ∫ d{ (ss-s^4)(asin(s) + s√(1-ss)) } - ∫ ds 2(ss-s^4)√(1-ss)
= 32 a^3 { (π/16 + 1/8) - (1/8)(π/4 + 1/3) } (∵ D)
= a^3 (π + 8/3)
0971132人目の素数さん
垢版 |
2019/02/24(日) 08:00:09.40ID:tbaAoo1o
@∫ dx √(1-xx)
= ∫ d{sinθ}cosθ
= ∫ dθ cosθ^2 = ∫ dθ (1- sinθ^2)
= θ + sinθcosθ - ∫ dθ cosθ^2
= (1/2)( asin(x) + x√(1-xx) )

As = √{(2ax-xx)/(4ax)}
x = 2a (1- 2ss), 2xdx = d{xx} = -32aa (s - 2s^3) ds

B∫ ds ss (1-ss)^{1/2}
= -(1/3) s(1-ss)^{3/2} + (1/3)∫ ds (1-ss)^{3/2}
= -(1/4) s(1-ss)^{3/2} + (1/4)∫ ds (1-ss)^{1/2}
= -(1/4) s(1-ss)^{3/2} + (1/8){ asin(s) + s√(1-ss) } (∵@)
0972132人目の素数さん
垢版 |
2019/02/24(日) 08:01:25.92ID:tbaAoo1o
C∫ ds (1-ss)^{3/2}
= ∫ ds (1-ss)(1-ss)^{1/2}
= (1/2)( asin(s) + s√(1-ss) ) - ∫ ds ss(1-ss)^{1/2} (∵@)
= (3/8)( asin(s) + s√(1-ss) ) + (1/4)s(1-ss)^{3/2} (∵B)

D∫ ds (ss-s^4)√(1-ss) = ∫ ds ss(1-ss)^{3/2}
= -(1/5)s(1-ss)^{5/2} + (1/5)∫ ds (1-ss)^{5/2}
= -(1/6)s(1-ss)^{5/2} + (1/6)∫ ds (1-ss)^{3/2}
= (1/16)(asin(s) + s√(1-ss)) + (1/24)s(1-ss)^{3/2} -(1/6)s(1-ss)^{5/2} (∵@C)
(s=1/√2)
= (1/16)(π/4 + 1/2) + (1/24)(1/4) - (1/6)(1/8)
= (1/16)(π/4 + 1/3)

END
0974132人目の素数さん
垢版 |
2019/02/24(日) 08:45:21.61ID:U/bGXjV7
「任意の無限集合は、必ず可算集合を部分集合として含む。」

この命題を証明するには、選択公理が必要であるそうですが、なぜ帰納法だけでは証明できないのでしょうか?



「可算集合の無限部分集合は可算である。」

同じ著者が、この命題の証明では、選択公理を使っていません。
証明は、 Z^+ の無限部分集合が可算であることを示せば十分であるとして、
Z^+ の部分集合には最小元があるということを使っています。
この命題の証明では、なぜ選択公理が不要なのでしょうか?
0975132人目の素数さん
垢版 |
2019/02/24(日) 11:53:32.89ID:ClFQ9YsZ
空でない正の整数の部分集合からその元を選択する関数として、最小元を取る関数が存在するからじゃね?
0977132人目の素数さん
垢版 |
2019/02/24(日) 13:41:55.02ID:U/bGXjV7
以下の証明はどこがダメなのでしょうか?

「任意の無限集合は、必ず可算集合を部分集合として含む。」

証明:

M を任意の無限集合とする。


M ≠ φ だから、
∃x ∈ M
a_1 := x とする。

M - {a_1} ≠ φ だから、
∃x ∈ M - {a_1}
a_2 := x とする。

M - {a_1, a_2} ≠ φ だから、
∃x ∈ M - {a_1, a_2}
a_3 := x とする。

a_n まで決まったら、

M - {a_1, a_2, …, a_n} ≠ φ だから、
∃x ∈ M - {a_1, a_2}
a_{n+1} := x とする。

と a_{n+1} を決めることができる。

帰納法により、すべての n ∈ N に足して、 a_n が決まる。

よって、 a : N → M は単射である。

よって M は加算部分集合を含む。
と a_n を決めていく。
0978132人目の素数さん
垢版 |
2019/02/24(日) 13:42:38.51ID:U/bGXjV7
訂正します:

以下の証明はどこがダメなのでしょうか?

「任意の無限集合は、必ず可算集合を部分集合として含む。」

証明:

M を任意の無限集合とする。


M ≠ φ だから、
∃x ∈ M
a_1 := x とする。

M - {a_1} ≠ φ だから、
∃x ∈ M - {a_1}
a_2 := x とする。

M - {a_1, a_2} ≠ φ だから、
∃x ∈ M - {a_1, a_2}
a_3 := x とする。

a_n まで決まったら、

M - {a_1, a_2, …, a_n} ≠ φ だから、
∃x ∈ M - {a_1, a_2, …, a_n}
a_{n+1} := x とする。

と a_{n+1} を決めることができる。

帰納法により、すべての n ∈ N に足して、 a_n が決まる。

よって、 a : N → M は単射である。

よって M は加算部分集合を含む。
と a_n を決めていく。
0979132人目の素数さん
垢版 |
2019/02/24(日) 13:43:11.61ID:U/bGXjV7
訂正します:

以下の証明はどこがダメなのでしょうか?

「任意の無限集合は、必ず可算集合を部分集合として含む。」

証明:

M を任意の無限集合とする。


M ≠ φ だから、
∃x ∈ M
a_1 := x とする。

M - {a_1} ≠ φ だから、
∃x ∈ M - {a_1}
a_2 := x とする。

M - {a_1, a_2} ≠ φ だから、
∃x ∈ M - {a_1, a_2}
a_3 := x とする。

a_n まで決まったら、

M - {a_1, a_2, …, a_n} ≠ φ だから、
∃x ∈ M - {a_1, a_2, …, a_n}
a_{n+1} := x とする。

と a_{n+1} を決めることができる。

帰納法により、すべての n ∈ N に対して、 a_n が決まる。

よって、 a : N → M は単射である。

よって M は加算部分集合を含む。
と a_n を決めていく。
0980132人目の素数さん
垢版 |
2019/02/24(日) 13:44:08.38ID:U/bGXjV7
訂正します:

以下の証明はどこがダメなのでしょうか?

「任意の無限集合は、必ず可算集合を部分集合として含む。」

証明:

M を任意の無限集合とする。


M ≠ φ だから、
∃x ∈ M
a_1 := x とする。

M - {a_1} ≠ φ だから、
∃x ∈ M - {a_1}
a_2 := x とする。

M - {a_1, a_2} ≠ φ だから、
∃x ∈ M - {a_1, a_2}
a_3 := x とする。

a_n まで決まったら、

M - {a_1, a_2, …, a_n} ≠ φ だから、
∃x ∈ M - {a_1, a_2, …, a_n}
a_{n+1} := x とする。

と a_{n+1} を決めることができる。

帰納法により、すべての n ∈ N に対して、 a_n が決まる。

以上より、 単射 a : N → M が存在する。

よって M は加算部分集合を含む。
0981132人目の素数さん
垢版 |
2019/02/24(日) 15:01:10.33ID:hPxxY52P
ID:U/bGXjV7のレスを読んでいます。

この方は、教科書に誤植や勘違いがあると著者を「いい加減な人ですね。」と過剰に責め立てるのに、自分の発言は何度も訂正しています。

他人に厳しく自分には激甘な、ゴミのような人間ですね。
0982132人目の素数さん
垢版 |
2019/02/24(日) 15:02:22.65ID:hPxxY52P
訂正します:

ID:U/bGXjV7のレスを読んでいます。

この方は、教科書に誤植や勘違いがあると著者を「いい加減な人ですね。」と過剰に責め立てるのに、自分の発言は何度も訂正しています。

他人に厳しく自分には激甘な、ゴミ以下の存在価値しかない人間ですね。
0985132人目の素数さん
垢版 |
2019/02/24(日) 17:37:25.75ID:ClFQ9YsZ
いくらでも長い列が存在するのと無限に長い列が存在するのは違う。
0991132人目の素数さん
垢版 |
2019/02/25(月) 02:12:23.49ID:/mxaunxg
>>981-982
 本で収入を得ている人は本の中身に責任を持たないと。
 信用無くしたら飯が食えねぇ...
0992132人目の素数さん
垢版 |
2019/02/25(月) 09:29:09.24ID:savhGCke
>>987
この一般式になるのって、どうやったら証明できるのでしょうか?
帰納法でやろうにも全然先へ進めませんでした。
0993132人目の素数さん
垢版 |
2019/02/25(月) 10:39:43.12ID:IjvstYh2
右半平面と上半平面の合併上の微分一形式(-ydx+xdy)/x^2+y^2のポテンシャル関数の求め方を教えてください

計算してみて
tan(y/x) (0<x, 0<y)
π/2-tan(x/y) (x<=0, 0<y)
あたりがポテンシャル関数になりそうだと思いましたがx=0での微分可能性が示せません
0995992
垢版 |
2019/02/25(月) 11:19:01.34ID:savhGCke
>>987
自己相似的なノコギリ波形が倍々で増えてくので直感的にそうなるのは分かるのですが
うまく数式証明できずに悩んでいます。
0996992
垢版 |
2019/02/25(月) 14:35:37.30ID:savhGCke
証明できました.

0)準備
f(x) = (x<1) ? 2x : 4-2x = (x<1) ? 2x : 2-2(x-1)
f(2-x) = (2-x<1) ? 2(2-x) : 4 -2(2-x) = (1<x) ? 4-2x : 2x = f(x)
つまりグラフは直線 x=1に関して対称
ff(x) = (x<1) ? f(2{x}) : f(2-2{x-1})
= (x<1) ? f(2{x}) : f(2{x-1}) = f(2{x})

1) n=1 の時
g[1](x) = ff(x) = f(2{x}) = f(2{2^(1-1).x}) (n=1 で成立)

2) g[n](x) = f(2{2^(n-1).x}) を仮定する
g[n+1](x) = f(g[n](x))
= ff(2{2^(n-1).x}) = f(2{2{2^(n-1).x}) = f(2{2^(n).x})
( ∵ 2{2{t}} = t<1 ? 2{2t} : 2{2(t-1)} = 2{2t} )
帰納法より g[n](x) = f(2{2^(n-1).x}) (n=1,2,...)
0997992
垢版 |
2019/02/25(月) 16:23:05.34ID:savhGCke
ついでに >>966 (2) の解答
(2進表記にて)
f(x) = (x<1) ? x<<1 : 100. - (x << 1)
f(a.bcde...) = (a==0) ? b.cdef... : B.CDEF...
(大文字はビット反転を表す)
∵ ab.cdef... + AB.CDEF... =11.1111... = 100. ≡ 0 (mod 4)

x=a[0].a[1]a[2]a[3]... と置くと
2{2^(n-1).x} = 2{ a[n-1].a[n]a[n+1]a[n+2]... } = a[n].a[n+1]a[n+2]...

g[n](x) = f(a[n].a[n+1]a[n+2]...)
= (a[n]==0) ? a[n+1].a[n+2]... : A[n+1].A[n+2]...

よって、ある n ≧ 0 で
・a[n].a[n+1]a[n+2]... = 0.00000... = 0
・a[n].a[n+1]a[n+2]... = 1.01010... = 4/3
どちらかになる事が収束の条件である.
つまり x = (N+δ/3)/2^k と表せる値で
δ=0 なら 0 に収束
δ=1 なら 4/3 に収束
どちらも有限回で収束値に達する. 他の値では収束しない.
0998132人目の素数さん
垢版 |
2019/02/25(月) 16:57:57.28ID:OteJGTQP
₁₂₃₄₅₆Ⓒ₈₉ɔ₁₀₁₁₁₂₁₃₁₄₁₅
1000132人目の素数さん
垢版 |
2019/02/25(月) 17:09:06.51ID:OteJGTQP
₁₂₃₄₅₆Ⓒ₈₉ɔ₁₀₁₁₁₂₁₃₁₄₁₅ⓒ
10011001
垢版 |
Over 1000Thread
このスレッドは1000を超えました。
新しいスレッドを立ててください。
life time: 57日 8時間 9分 2秒
10021002
垢版 |
Over 1000Thread
5ちゃんねるの運営はプレミアム会員の皆さまに支えられています。
運営にご協力お願いいたします。


───────────────────
《プレミアム会員の主な特典》
★ 5ちゃんねる専用ブラウザからの広告除去
★ 5ちゃんねるの過去ログを取得
★ 書き込み規制の緩和
───────────────────

会員登録には個人情報は一切必要ありません。
月300円から匿名でご購入いただけます。

▼ プレミアム会員登録はこちら ▼
https://premium.5ch.net/

▼ 浪人ログインはこちら ▼
https://login.5ch.net/login.php
レス数が1000を超えています。これ以上書き込みはできません。

ニューススポーツなんでも実況